Random Specialities Facts + Past exam Qs Flashcards

1
Q

Uterus palpable abdominally at how many weeks

A

Uterus palpable abdominally at 12-14 weeks

How well did you know this?
1
Not at all
2
3
4
5
Perfectly
2
Q

Engagement =

A

If 2/5 head palpable abdominally, then more than half has entered the pelvis and so the head
must be engaged

How well did you know this?
1
Not at all
2
3
4
5
Perfectly
3
Q

What is used to date pregnancies under 14w?

A

NICE guidelines: should date women using crown rump length if <14 weeks

How well did you know this?
1
Not at all
2
3
4
5
Perfectly
4
Q

Causes of raised AFP

A

Alpha fetoprotein
o Produced by fetal liver
o Open neural tube defect or abnormalities such as gastrochisis - increased maternal AFP
o Indicates risk of third trimester complications
o Seldom used as USS more accurate

How well did you know this?
1
Not at all
2
3
4
5
Perfectly
5
Q
 Low PAPP-A (1st tri)
 High B-hCG (1st/2nd)
 Low AFP (1st/2nd)
 Low oestriol (2nd)
 High inhibin (2nd)
A

DOWNS

How well did you know this?
1
Not at all
2
3
4
5
Perfectly
6
Q

Absence of cranium
 Frog Eye appearance on USS
 Incompatible with life

A

Anencephaly

How well did you know this?
1
Not at all
2
3
4
5
Perfectly
7
Q

 Partial extrusion of abdominal contents in peritoneal sac
 50% have chromosomal problem
 Isolated, small defects have good prognosis

A

Exomphalos

How well did you know this?
1
Not at all
2
3
4
5
Perfectly
8
Q

 Free loops of bowel in amniotic cavity
 Rarely associated with other abnormalities
 Common if mother young
 >90% survive – requires postnatal surgery

A

Gastroschissis

How well did you know this?
1
Not at all
2
3
4
5
Perfectly
9
Q

 Dependent on systemic vascular resistance and cardiac output
 Falls to a minimum in second trimester by 30/15mmHg due to  SVR
 By term, BP is at pre-pregnant levels
 HTN due to PET is largely due to  SVR
 Protein excretion in pregnancy is increased, but in absence of underlying renal disease, should be
<0.3g/24h

A

Normal BP changes in Pregnancy

How well did you know this?
1
Not at all
2
3
4
5
Perfectly
10
Q

Normal BP changes in Pregnancy

A

 Dependent on systemic vascular resistance and cardiac output
 Falls to a minimum in second trimester by 30/15mmHg due to  SVR
 By term, BP is at pre-pregnant levels
 HTN due to PET is largely due to increased SVR
 Protein excretion in pregnancy is increased, but in absence of underlying renal disease, should be
<0.3g/24h

How well did you know this?
1
Not at all
2
3
4
5
Perfectly
11
Q

Simple classification of PET

A

Mild (140/90-149/99mmHg) Moderate (150/100-159/99mmHg) Severe (>160/110mmHg)

How well did you know this?
1
Not at all
2
3
4
5
Perfectly
12
Q

PCR level in PET

A

Level of 30mg/nmol = 0.3g/24h protein excretion

How well did you know this?
1
Not at all
2
3
4
5
Perfectly
13
Q

PCR level in PET

A

Level of 30mg/nmol = 0.3g/24h protein excretion

How well did you know this?
1
Not at all
2
3
4
5
Perfectly
14
Q

Sx of magnesium toxicity

A

Respiratory depression and hypotension

Preceded by loss of patellar reflexes, which are tested regularly

How well did you know this?
1
Not at all
2
3
4
5
Perfectly
15
Q

ECG changes in pregnancy

A

ECG changes = LAD and T wave inversion

How well did you know this?
1
Not at all
2
3
4
5
Perfectly
16
Q

Thromboprophylaxis in pregnancy?

A

LMWH

How well did you know this?
1
Not at all
2
3
4
5
Perfectly
17
Q

What drug is given alongside anti-epileptics from week 36?

A

10mg vit K given from 36 weeks onwards

How well did you know this?
1
Not at all
2
3
4
5
Perfectly
18
Q

 Acute hepatorenal failure, DIC and hypoglycaemia  high maternal and fetal mortality
 Extensive fatty change
 Malaise, vomiting, jaundice and vague epigastric pain (first sx = thirst)
 Early diagnosis and promt delivery essential
 Correction of clotting defects and hypoglycaemia required first
 Tx: supportive, dextrose, fluid balance, dialysis

A

Acute Fatty Liver

How well did you know this?
1
Not at all
2
3
4
5
Perfectly
19
Q

lupus anticoagulant and/or anticardiolipin antibodies

A

Antiphospholipid Syndrome

How well did you know this?
1
Not at all
2
3
4
5
Perfectly
20
Q

Criteria for APLS

A

Diagnosis

1+ clinical criteria
o Vascular thrombosis
o 1+ death of fetus >10 weeks
o PET or IUGR requiring delivery <34 weeks
o 3+ fetal loss <10 weeks, otherwise unexplained

With laboratory criteria measured twice >3 months apart
o Lupus anticoagulant
o High anticardiolipin antibodies
o Anti-B2 Glp I ab

How well did you know this?
1
Not at all
2
3
4
5
Perfectly
21
Q

Diagnosis

1+ clinical criteria
o Vascular thrombosis
o 1+ death of fetus >10 weeks
o PET or IUGR requiring delivery <34 weeks
o 3+ fetal loss <10 weeks, otherwise unexplained

With laboratory criteria measured twice >3 months apart
o Lupus anticoagulant
o High anticardiolipin antibodies
o Anti-B2 Glp I ab

A

Criteria for APLS

How well did you know this?
1
Not at all
2
3
4
5
Perfectly
22
Q

Preg SE of paroxetine?

A

Paroxetine -> cardiac defects

How well did you know this?
1
Not at all
2
3
4
5
Perfectly
23
Q

 Not teratogenic
 Use associated with PTD, IUGR, stillbirth, SIDS and developmental delay

Which recreational drug?

A

Opiates

How well did you know this?
1
Not at all
2
3
4
5
Perfectly
24
Q

Which recreational drug?

 Teratogenic
  Risk cardiac defects and gastroschicosis
 Pregnancy complications are similar to cocaine

A

Ecstasy

How well did you know this?
1
Not at all
2
3
4
5
Perfectly
25
Q

Which recreational drug?

 Associated with facial clefs
 Cause neonatal hypotonia and withdrawal sx

A

BDZ

How well did you know this?
1
Not at all
2
3
4
5
Perfectly
26
Q

Which recreational drug?

 Associated with other drug use makes it higher risk
 May cause IUGR and affect child development

A

Cannabis

How well did you know this?
1
Not at all
2
3
4
5
Perfectly
27
Q
o Facial abnormalities
o IUGR
o Small or abnormal brain
o Developmental dlay
o Usually >18 units/day alcohol exposure

Which recreational drug?

A

Alcohol

How well did you know this?
1
Not at all
2
3
4
5
Perfectly
28
Q

 Dose-response manner   risk miscarriage, IUGR, PTD, placental abruption, stillbirth and SIDS
 Associated with childhood illness
 PET is more severe if assoc.
 Consider high risk if women do not stop

Which recreational drug?

A

Tobacco

How well did you know this?
1
Not at all
2
3
4
5
Perfectly
29
Q

Blood volume increases by how much in pregnancy?

A

40% increase in blood volume   red cell mass and net fall in Hb concentration

How well did you know this?
1
Not at all
2
3
4
5
Perfectly
30
Q

AntiD dose 28w

A

AntiD (1500IU) should be given to all Rh –ve women at 28 weeks

How well did you know this?
1
Not at all
2
3
4
5
Perfectly
31
Q

Bleeding from the genital tract after 24 weeks gestation

A

Antepartum Haemorrhage

How well did you know this?
1
Not at all
2
3
4
5
Perfectly
32
Q

Delivery MUST be via (elec) CS?

A

Placentae praaevia

How well did you know this?
1
Not at all
2
3
4
5
Perfectly
33
Q

SGA definition

A

Weight of fetus is <10th centile for gestation

How well did you know this?
1
Not at all
2
3
4
5
Perfectly
34
Q

Weight of fetus is <10th centile for gestation (at term = <2.7kg)

A

SGA

How well did you know this?
1
Not at all
2
3
4
5
Perfectly
35
Q

How long should the first stage of labour be?

A

<12hrs

How well did you know this?
1
Not at all
2
3
4
5
Perfectly
36
Q

How long should the second stage of labour be?

A

40 mins - nulliparous
20 mins - multiparous
>1hr likely to need assistance

How well did you know this?
1
Not at all
2
3
4
5
Perfectly
37
Q

Do frogs in canada ride in a pink limo?

A
Descent 
Flexion
Internal rotation
Crowning
Restitution 
Internal rotation of the shoulders
Anterior shoulder
Posterior shoulder
Lateral flexion
How well did you know this?
1
Not at all
2
3
4
5
Perfectly
38
Q

Average time stage 3 of labour

A

15 mins

How well did you know this?
1
Not at all
2
3
4
5
Perfectly
39
Q

What is mendelsons syndrome?

A

Aspiration of stomach contents under GA

How well did you know this?
1
Not at all
2
3
4
5
Perfectly
40
Q

Keilland’s forceps

A

Occipito-posterior position

How well did you know this?
1
Not at all
2
3
4
5
Perfectly
41
Q

Scalp pH <7.20 (capillary) indicates…

A

…significant hypoxia

How well did you know this?
1
Not at all
2
3
4
5
Perfectly
42
Q

Name a systemic opiate used during delivery

A
 Pethidine
 Meptid
 Occasionally
diamorphine
 Given as IM
injections

 Can be self-
administered

 Can cause
sedation,
confusion and
feeling ‘out of
control’
 Antiemetics also
needed
 Causes respiratory
depression in
newborn – requires
reversal with
naloxone
How well did you know this?
1
Not at all
2
3
4
5
Perfectly
43
Q

3 methods of anaesthesia in labour

A
Spinal
 Injection through dura mata into
CSF
 Produces short-lasting but effect
analgesia

 Method of choice for CS or mid-
cavity instrumental delivery if no

epidural in situ
 Complications = hypotension, total
spinal analgesia and respiratory
paralysis
Pudendal Nerve Block
 Local anaesthetic injected
bilaterally around pudendal nerve
where is passes ischial spine
 Low cavity instrumental deliveries
Epidural Anaesthesia
 Injection of local anaesthetic via
epidural catheter into epidural
space
 Between L3 and 4
 Local anaesthetic infused
continuously or to ‘top up’
intermittently
 Complete sensory and partial
motor blockade is the norm
How well did you know this?
1
Not at all
2
3
4
5
Perfectly
44
Q

Complications of epidural

A

o Spinal tap affects 0.5%
o Puncture of dura mata  leakage of CSF and severe headache
o Pain worse when sitting up
o Treated with analgesia
o ‘Blood patch’ to seal the leak
o Inadvertent IV injection  convulsions and cardiac arrest
o Inadvertent injection of local anaesthetic into CSF + progression up SC  total spinal
paraesthesia

Must also monitor urination - no sensation to bladder. Encourage frequent urination

How well did you know this?
1
Not at all
2
3
4
5
Perfectly
45
Q

Absolute contraindications to induction of labour?

A
 Acute fetal compromise
 Abnormal lie
 Placenta praevia
 Pelvic obstruction e.g. mass or deformity
 cephalopelvic disproportion
 >1 CS
How well did you know this?
1
Not at all
2
3
4
5
Perfectly
46
Q

VBAC contraindications

A

 Usual contraindications for CS
 Vertical uterine scar
 Multiple previous caesarean
 After two CS, seldom attempt VD

How well did you know this?
1
Not at all
2
3
4
5
Perfectly
47
Q

Zanavelli manoeuvre

A

replacement of the head and caesarean section - dystocia

How well did you know this?
1
Not at all
2
3
4
5
Perfectly
48
Q

replacement of the head and caesarean section

A

Zanavelli manoeuvre:

How well did you know this?
1
Not at all
2
3
4
5
Perfectly
49
Q

pressure on anterior and posterior shoulder to rotate shoulders from
transverse position

A

Wood’s screw manoeuvre

How well did you know this?
1
Not at all
2
3
4
5
Perfectly
50
Q

Wood’s screw manoeuvre

A

pressure on anterior and posterior shoulder to rotate shoulders from
transverse position

How well did you know this?
1
Not at all
2
3
4
5
Perfectly
51
Q

How long is lochia normal?

A

Lochia (uterine discharge) may be blood-stained for 4 weeks, but is thereafter yellow or white
 Menstruation is usually delayed by lactation (6 weeks after if not lactating)

How well did you know this?
1
Not at all
2
3
4
5
Perfectly
52
Q

What two hormones control milk secretion?

A

 Prolactin from anterior pituitary stimulates milk secretion
o High at birth
o Rapid decline in E2 and P4  milk secreted
o Prolactin antagonised by E2/P4
 Oxytocin from posterior pituitary stimulates ejection

How well did you know this?
1
Not at all
2
3
4
5
Perfectly
53
Q

What is colostrum?

A

Colostrum – yellow fluid containing fat-laden cells, proteins (IgA) and minerals
o Passed for first 3 days before milk

How well did you know this?
1
Not at all
2
3
4
5
Perfectly
54
Q

Mild depression, how many core + bio?

A

MILD: 2 core + 2 bio

How well did you know this?
1
Not at all
2
3
4
5
Perfectly
55
Q

Mod depression, how many core + bio?

A

MODERATE: 2 core + 6 bio

How well did you know this?
1
Not at all
2
3
4
5
Perfectly
56
Q

Severe depression, how many core + bio?

A

SEVERE: ≥8 symptoms, inc all 3 core

How well did you know this?
1
Not at all
2
3
4
5
Perfectly
57
Q

Organic / physical causes of depression?

A

Physical causes: hypothyroid, head injury, cancer, quiet delirium, meds etc

Adjustment disorder: following a life event, but not as severe
Normal sadness: part of life!
Bereavement: becomes a concern when grief (numbness, pining, depression, recovery) is
extremely intense, prolonged (>6/12) or delayed

How well did you know this?
1
Not at all
2
3
4
5
Perfectly
58
Q

Can you stop antidepressants immediately?

A

Antidepressants cannot be stopped suddenly, wean over weeks, SSRIs increased risk of
bleeding, especially in older people (consider prescribing gastroprotective in older people
who are also taking NSAIDs or aspirin)

How well did you know this?
1
Not at all
2
3
4
5
Perfectly
59
Q

What distinguishes type 1+2 bipolar?

A

*Classification: Type I – manic episodes interspersed with depressive episodes,
Type II- mainly recurrent depressive episodes, less prominent hypomanic episodes,
Rapid cycling BPAD - ≥4 affective episodes in a year, women, respond better to
valproate
*MIXED EPISODE: manic/hypomanic + depressive symptoms in a single episode,
present every day for at least a week (ICD-10)
*Ultra-rapid cycling: fluctuations over days or even hours

How well did you know this?
1
Not at all
2
3
4
5
Perfectly
60
Q

Support groups for bipolar?

A
Bipolar UK
MIND
SANE
RETHINK
Samaritans
How well did you know this?
1
Not at all
2
3
4
5
Perfectly
61
Q

Delusion definition

A

Delusion – false, fixed belief that the patient has and is convinced by contrary to the
evidence and rational argument that cannot be explained by the patient’s cultural,
religious or educational background.

How well did you know this?
1
Not at all
2
3
4
5
Perfectly
62
Q

Neurodevelopmental aetiology of schizophrenia?

A

Neurodevelopmental: enlarged ventricles, overall smaller/lighter brains, no gliosis
( changes before adulthood). Further evidence: low pre-morbid IQ, poor
learning/mem/executive function. Early brain damage/abnormalities imperceptible
at first, more pronounced as brain matures through ongoing myelination and
synaptic pruning.

How well did you know this?
1
Not at all
2
3
4
5
Perfectly
63
Q

What is the dopamine hypothesis in schizophrenia?

A

Neurotransmitters: dopamine hypothesis –POSITIVE symptoms from EXCESS
dopamine in MESOLIMBIC tracts, NEGATIVE symptoms from REDUCED dopamine in
MESOCORTICAL tracts. Serotonin hyperactivity, gluatamate dyfunction too

How well did you know this?
1
Not at all
2
3
4
5
Perfectly
64
Q

List the subtypes of schizophrenia

A

Paranoid: delusion & hallucination
Hebephrenic: Affective changes (extension of prodrome) Disorganised speech
behaviour (silly/shallow), flat/inappropriate affect
Catatonic: Psychomotor disturbance (treat with BZDs)
Undifferentiated: Meets criteria but no specific dominant symptom
Post-schizophrenic depression: Some residual symptoms, but depression mainly
Residual: Previous +ve symptoms decreased, - ve symptoms prominent
Simple: No delusions/hallucinations, ‘defect state’ (-ve) gradually arises without an
acute episode

How well did you know this?
1
Not at all
2
3
4
5
Perfectly
65
Q

List typical antipsychotic SEs

A

 Typical antipsychotics Extrapyramidal side effects
o Acute dystonia: early onset (hours), involuntary, painful sustained muscle
spasm eg torticolllis, oculogyric crisis; tongue and sternocledomastoid mostly
 tx with anticholinergic iv procyclidine
o Akathisia: hours –weeks, unpleasant restlessness of usually lower limbs 
change/↓ dose; add propranolol or BDZ
o Parkinsonism: days-weeks, triad: resting tremor, rigidity, bradykinesia 
change/↓ dose, add anticholingergic (procyclidine iv)
o Tardive dyskinesia: months-years; rhythmic involuntary movemetns,
continuous slow writhing movements, esp oral-lingual/limbs, tend to be
irreversible  /stop antipsychotic, avoid anticholinergic (worse), atypical
SSRI/clozapine
 ! Neuroleptic malignant syndrome: muscle stiffness and rigidity, altered
consciousness, disturbance of autonomic (feve, tachy, labile BP), raised CK and WCC,
acute renal failure secondary to rhabdomyolysis can  death. Normally when
changing/increasing dose of drug. Tx: immediately stop antipsychotic, medical ward,
hydration & oxygen, monitoring, dantrolene and bromocriptine

How well did you know this?
1
Not at all
2
3
4
5
Perfectly
66
Q

List the atypical antipsychotics SEs

A

Atypical antipsychotics (metabolic syndrome, weight gain) :
Olanzapine – helps with
positive symptoms ( but causes weight gain)
Quietiapine – qt prolongation – need
ECG
Risperidone – may increase prolactina and aggression
Aripiprazole – expensive
but no side effects (use in patients with metabolic syndrome)

How well did you know this?
1
Not at all
2
3
4
5
Perfectly
67
Q

Clozapine MoA?

A

Clozapine
 Blocsk D1 and D4 receptors, superiority due to added blockade of 5HT2 receptors and
increased GABA turnover
 Anticholinergic, antihistaminic, anti-adrenergic side effects: constipation, fever, BP
derangement, sedation, seizures, weight gain etc

How well did you know this?
1
Not at all
2
3
4
5
Perfectly
68
Q

SEs of clozapine

A

Anticholinergic, antihistaminic, anti-adrenergic side effects: constipation, fever, BP
derangement, sedation, seizures, weight gain etc

Many interactions: lithium (↑seizure risk, anticholinesterase inhibitors, smoking
increases clearance  ↓ plasma conc, plasma conc ↑by caffeine
 C/I: previous/current neutropenia or blood dyscrasias, previous
MI/pericarditis.cardiomyopathty, liver disease
 Greatest worry is fatal agranulocytosis- leukopenia, eosinophilia, leucocytosis 
regular blood tests for WCC (weekly for 18 weeks  fortnightly until 1 year 
monthly indefinitely) . Fatal myocarditis/cardiomyopathy/pulmonoary embolism aslo
a worry.
 Evidence is that it REDUCES mortality in schizo by ↓ suicide!

How well did you know this?
1
Not at all
2
3
4
5
Perfectly
69
Q

Define schizoaffective disorder?

A

Both features of schizophrenia and affective disorder, 50:50

How well did you know this?
1
Not at all
2
3
4
5
Perfectly
70
Q

Define SCHIZOTYPCAL DISORDER

A

‘ Partial expression’ of the schizophrenia phenotype. Classified along with
schizophrenia in ICD-10, Cluster A with odd-eccentric personality disorder in DSM-IV.
No hallucinations and delusions.

How well did you know this?
1
Not at all
2
3
4
5
Perfectly
71
Q

Define SCHIZOPHRENIFORM DISORDER

A

Schizophrenia-like psychosis that fails to fulfil duration criterion for schizophrenia in DSM-IV

How well did you know this?
1
Not at all
2
3
4
5
Perfectly
72
Q

Transient ‘state of shock’ lasting minutes-hours,
max 1-3 days. Anxious but may seem dazed, may experience amnesia,
depersonalisation and derealisation, may stop talking)

A

Acute stress reaction

How well did you know this?
1
Not at all
2
3
4
5
Perfectly
73
Q

Associated with persistent fear & prominent
avoidance of the feared situation, anticipatory anxiety attacks and
insight that the fear is irrational and disproportionate to the risk. Think
of impact on daily functioning. May be so severe that it induces panic
attack

A

Phobic anxiety disorders

How well did you know this?
1
Not at all
2
3
4
5
Perfectly
74
Q

Fear of being unable to easily escape to a
safe place (e.g. home). May manifest in open spaces, or
confined spaces that are difficult to leave without
attracting attention. 95% have current or past diagnosis of
panic disorder. Onset in mid twenties or thirties

A

Agoraphobia

How well did you know this?
1
Not at all
2
3
4
5
Perfectly
75
Q

Fear of being criticised or scrutinised; worry
that they will be embarrassed in public. Will tolerate an
anonymous crowd (unlike agoraphobics) but small groups
(e.g. meetings/dinner parties) are intimidating. May have
specific worries e.g. eating in public. Self-medication with
alcohol or drugs serves as avoidance and therefore
perpetuates problem.

A

Social phobia

How well did you know this?
1
Not at all
2
3
4
5
Perfectly
76
Q

‘episodic paroxysmal anxiety’. Not restricted to
certain situations (i.e. not phobic) or objective danger. Patients may
develop fear of having further attacks – ‘anticipatory anxiety’. Many
also have agoraphobia. ‘Panic’ can persist until patients receives
reassurance or reverts to ‘safety behaviours’: actions to avoid
catastrophe, e.g. calling ambulance, taking aspirin.

A

Panic disorder

How well did you know this?
1
Not at all
2
3
4
5
Perfectly
77
Q

Transient ‘state of shock’ lasting minutes-hours,
max 1-3 days. Anxious but may seem dazed, may experience amnesia,
depersonalisation and derealisation, may stop talking)

A

Acute stress reaction

How well did you know this?
1
Not at all
2
3
4
5
Perfectly
78
Q

Abnormal psychological changes to adversity.
May follow common life changes.
o Onset within weeks and lasts less than 6mths
o Symptoms of anxiety and depression, without the biological
symptoms of depression. None of the symptoms should be of
sufficient severity or prominence in its own right to justify a
more specific diagnosis.

A

Adjustment disorder

How well did you know this?
1
Not at all
2
3
4
5
Perfectly
79
Q

Organic DDx to rule out in anxiety disorders?

A
For anxiety, rule out endocrine – phaeo (urinary catecholamines),
hyperthyroid (TFTs); neuro – b12 deficiency; metabolic – hypoglycaemia.
Porphyria; cardio- arythmia, AF, mitral valve prolapse; substance misuse –
alcohol withdrawal (LFT – GGT), smoking withdrawal
How well did you know this?
1
Not at all
2
3
4
5
Perfectly
80
Q

1st line Rx for PTSD

A

First line treatment is
trauma-focussed CBT
and EMDR

____________

Medicate if there is severe
ongoing threat, or if
patient is too distressed
for psychotherapy, or if
psychotherapy fails
SSRIs (paroxetine,
sertraline)
Sleep disturbance –
mirtazapine,
levomepromazine, Anxiety/hyperarousal –
BDZ (clonazepam),
busiprone,
antidepressants,
propranolol
Intrusive thoughts –
carbamazepine, valproate,
topiramate, lithium
Psychosis – olanzapine,
risperidone, quetiapine,
clozapine, aripiprazole
How well did you know this?
1
Not at all
2
3
4
5
Perfectly
81
Q

Anxiety-producing obsessions which they try
to relieve with rituals (compulsions). These must cause distress or
interfere with the person’s social or individual functioning (usually by
wasting time) and should not be the result of another psychiatric
disorder.

A

OCD (duration >2w)

Obsessions – involuntary thoughts, images or impulses which are:
1. Recurrent and intrusive (unpleasant/distressing)
2. Enter mind against conscious resistance
3. Patients recognise obsessions as being the product of their
own mind even though they are involuntary and often
repugnant

Compulsions – repetitive mental operations or physical acts with the
following characteristics:
1. Feel compelled to perform them in response to their own
obsessions or irrationally defined rules
2. Performed to reduce anxiety through belief they will prevent a
dreaded event, even though they are not realistically
connected to the event

How well did you know this?
1
Not at all
2
3
4
5
Perfectly
82
Q

What is a compulsion?

A

Compulsions – repetitive mental operations or physical acts with the
following characteristics:
1. Feel compelled to perform them in response to their own
obsessions or irrationally defined rules
2. Performed to reduce anxiety through belief they will prevent a
dreaded event, even though they are not realistically
connected to the event

How well did you know this?
1
Not at all
2
3
4
5
Perfectly
83
Q

Define an obsession?

A

Obsessions – involuntary thoughts, images or impulses which are:
1. Recurrent and intrusive (unpleasant/distressing)
2. Enter mind against conscious resistance
3. Patients recognise obsessions as being the product of their
own mind even though they are involuntary and often
repugnant

How well did you know this?
1
Not at all
2
3
4
5
Perfectly
84
Q

What conditions is OCD associated with?

A

BIO
Affected by illnesses in
which risk of OCD is increased: Sydenhem’s chorea, encephalitis
lethargica, Tourette’s syndrome. Strep throat infection may
produce anti-basal ganglia antibodies (c.f. streptococcal infection
causing Sydenham’s chorea). Neuroimaging has shown deficit in
frontal-lobe inhibition – intrusive/ritualistic thoughts might be
harder to suppress in OCD.

PSYCH
1. Anankastic personality traits: rigidity, orderliness. ¼ of OCD
patients have premorbid anankastic personality traits.
2. Stress: may precipitate OCD symptoms

How well did you know this?
1
Not at all
2
3
4
5
Perfectly
85
Q

Syndrome characterised by
acute onset of fluctuating cognitive impairment (or deterioration in
pre existing cognitive impairment) associated with behavioural
abnormalities.

A

Delerium

How well did you know this?
1
Not at all
2
3
4
5
Perfectly
86
Q

Causes of delirium?

A
  1. Infective: UTI, chest infection, abscess, cellulitis, subacute
    bacterial endocarditis
  2. Metabolic: anaemia, electrolyte disturbance, hepatic
    encephalopathy, uraemia, cardiac failure, hypothermia
  3. Intracranial: CVA, head injury, encephalitis, primary or metastatic
    tumour, raised ICP
  4. Endocrine: pituitary, thyroid, parathyroid, adrenal disease,
    hypoglycaemia, DM, vitamin deficiencies
  5. Substances: intoxication or withdrawal of alcohol, BDZ,
    anticholinergics, psychotropics, lithium, antihypertensives,
    diuretics, anticonvulsants, digoxin, steroids, NSAIDs

Always consider the ‘great cerebral masqueraders’: TB, neurosyphilis,
AIDS

How well did you know this?
1
Not at all
2
3
4
5
Perfectly
87
Q

DDx for delirium?

A

Wernicke’s encephalopathy – which is a medical emergency 

Korsakoff’s psychosis
 Mood disorder
 Functional psychiatric conditions (mania, depression, late onset
schizophrenia)
 Responses to major stress, dissociative disorders
 Dementia (hard to differentiate as people with established
dementia are vulnerable to delirium)

How well did you know this?
1
Not at all
2
3
4
5
Perfectly
88
Q

Different seizure types

A

Generalised: involve whole cortex and lead to LOC
Focal: involve one area of cortex and may become secondarily
generalised. May be subclassified as simple partial or complex partial
Simple partial: localised motor/ sensory features ± LOC or memory
loss
Complex partial: ± aura/automatism, and associated changes in
conscious level

How well did you know this?
1
Not at all
2
3
4
5
Perfectly
89
Q

Biological sources of epilepsy

A
  1. Cerebrovascular damage
  2. Cerebral tumours
  3. Alcohol related seizures
  4. Post traumatic seizures
How well did you know this?
1
Not at all
2
3
4
5
Perfectly
90
Q

Cognitive decline, choreiform involuntary movements and personality change

A

Huntington

How well did you know this?
1
Not at all
2
3
4
5
Perfectly
91
Q

What chromosome is Huntington’s gene on?

A

Autosomal dominant gene on Chr 4

How well did you know this?
1
Not at all
2
3
4
5
Perfectly
92
Q

Classic triad: chorea, dementia and FH of HD

 Chorea: initial jerks, tics, gross involuntary movements of all parts
of the body, grimacing, dysarthria. Increased one with rigidity and
stiffness, positive primitive reflexes, abnormal eye movements
 Cognitive impairments  subcortical dementia
o Mental slowing
o Impaired executive function
o Speech deteriorates faster than comprehension
 Psychiatric disturbances
o Common in those with HD
o Changes in behaviour/ personality
o Affective disorders
o Schiphreniform psychoses
o These disturbances are not related to the severity of HD

A

HUNTINGTON’S DISEASE

How well did you know this?
1
Not at all
2
3
4
5
Perfectly
93
Q

Alcohol MoA

A

Alcohol MOA: non-specific effects on neuronal cell wall fluidity and permeability,
as well as enhancement of GABA-A transmission (anxiolytic), release of dopamine
in mesolimbic system (‘reward’), inhibition of NMDA-mediated glutaminergic
transmission (amnesic)

How well did you know this?
1
Not at all
2
3
4
5
Perfectly
94
Q

Profound short-term memory loss characterised by confabulation

A

Korsakoffs

How well did you know this?
1
Not at all
2
3
4
5
Perfectly
95
Q

a. Most dramatic neuropsychiatric complication
b. Thiamine deficiency  mammillary body damage  ataxia,
nystagmus, ophthalmoplegia, acute confusion

A

Wernickes

How well did you know this?
1
Not at all
2
3
4
5
Perfectly
96
Q

3 stages of alcohol withdrawal (severity)

A

Uncomplicated alcohol withdrawal syndrome: 4-12hrs after last drink.
Coarse tremor, sweating, insomnia, tachycardia, nausea and vomiting,
psychomotor agitation, anxiety. ± hallucinations (transitory visual, tactile or
auditory), craving for alcohol. Symptoms prolonged in heavier drinkers.
 Alcohol withdrawal syndrome with seizures: 5-15% of withdrawals
complicated by grand mal seizures 6-48hrs after last drink.
 Delirium tremens: 1-7 days after last drink with peak incidence @ 48hrs.
Severe dependence, comorbid infection and pre-existing liver damage
increase risk. Features of withdrawal and additionally: clouding of
consciousness, disorientation, amnesia for recent events, psychomotor
agitation, visual, auditory and tactile hallucinations (Lilliputian hallucinations
of diminutive people or animals), marked hour by hour fluctuations (worse at
night), if severe there is heavy sweating, fear, paranoid delusions, agitation,
suggestibility, temperature and sudden CV collapse. Reported mortality 5-
10%

How well did you know this?
1
Not at all
2
3
4
5
Perfectly
97
Q

BDZ used in alcohol detox?

A

Chlordiazepoxide (lowest abuse potential)

How well did you know this?
1
Not at all
2
3
4
5
Perfectly
98
Q

Lady has a diagnostic laparotomy. She has suprapubic pain that not even IV paracetamol is helping.
What is the likely reason?

A

Urinary retention

How well did you know this?
1
Not at all
2
3
4
5
Perfectly
99
Q

Somali lady comes in doesn’t speak much English says she’s 42 weeks. Examination of abdomen suggests
a 32 week uterus.

Admit her to ward for ANC tomorrow
do HIV test as
HIV babies tend to be small, Induce her labour now,
do a transabdominal scan

A

do a transabdominal scan

How well did you know this?
1
Not at all
2
3
4
5
Perfectly
100
Q

Leading cause of maternal death in UK

A

Heart disease remains the leading cause of women dying during pregnancy or up to six weeks after giving birth, followed by blood clots. Maternal suicide is the fifth most common cause of women’s deaths during pregnancy and its immediate aftermath, but is the leading cause of death over the first year after pregnancy.

However, there are striking inequalities: black women are five times and Asian women two times more likely to die as a result of complication

How well did you know this?
1
Not at all
2
3
4
5
Perfectly
101
Q

Management of 4 year old with enuresis - dry by day.

A

Ans: reassure. (Enuresis is a problem over the age

of 5)

How well did you know this?
1
Not at all
2
3
4
5
Perfectly
102
Q

Child with Downs has an NJ tube at home. It comes out and needs re-inserting. Who should put it back in? [Community paeds nurse; Community Paediatrician; Hospital Nursing staff; GP, healthworker]

A

Hospital Nursing staff

How well did you know this?
1
Not at all
2
3
4
5
Perfectly
103
Q

You’re the duty GP. Mum calls about child with fever/non-blanching rash. What do you do/advise? [Go to their home with you bag to give IM benpen; Arrange an emergency ambulance to take em hospital; Tell them to go to A&E; Tell them to make an appointment with GP)

A

Arrange an emergency ambulance to take em hospital

How well did you know this?
1
Not at all
2
3
4
5
Perfectly
104
Q

Child diagnosed with functional abdo pain. Associated with school. What’s the best management plan? [Encourage her to go to school and come back when her tummy starts to hurt; Have her work sent home whilst she still having pains; Change school; Make her go to school regardless of tummy pains, give her home tutoring]

A

Make her go to school regardless of tummy pains

How well did you know this?
1
Not at all
2
3
4
5
Perfectly
105
Q

Child is slow to get dressed. Likes to arrange his toys in a particular way. [ASD; OCD; Oppositional defiant disorder etc]

A

ASD

How well did you know this?
1
Not at all
2
3
4
5
Perfectly
106
Q

Child is an arsonist. Gets in fights. Attacks teachers. What is he likely to have at age 20? [Conduct disorder; ADHD, Dissocial personality disorder; Oppositional defiant disorder]

A

Conduct

How well did you know this?
1
Not at all
2
3
4
5
Perfectly
107
Q

Neonate. Ortolani test positive. What you do? [Hip ultrasound at 12 weeks; Hip ultrasound at 6 months; Hip x-ray at 12 weeks; Hip ultrasound at 6 months]

A

12 = best answer

Before 6 months of age, ultrasound is preferred over radiographs for evaluation of DDH due to insufficient ossification of the hip. In the absence of clinical findings, ultrasound should be delayed until about 6 weeks of age to reduce false positive results.

How well did you know this?
1
Not at all
2
3
4
5
Perfectly
108
Q

Child with downs in residence. Short term history of attacking people, anger outbursts etc. Best management? [Give antipsychotics; Move her to new home; Send her for neuropsychiatric assessment/therapy; Give antidepressant]

A

Send her for neuropsychiatric assessment/therapy

How well did you know this?
1
Not at all
2
3
4
5
Perfectly
109
Q

Asthmatic child in A&E. Mother has been giving two puffs of salbutamol with spacer every 4 hours. Description given correlated to severe asthma. What your management? [Inhaled nebs; inhaled adrenaline, Discharge; 10 puffs of salbutamol through spacer]

A

Inhaled nebs

How well did you know this?
1
Not at all
2
3
4
5
Perfectly
110
Q

Child has inspiratory stridor (doesn’t explicitly state, but gives a description implying this) since birth. Likely cause?

A

[Laryngomalacia;]

How well did you know this?
1
Not at all
2
3
4
5
Perfectly
111
Q

A child with short stature. Estimated mid parental height is on 25th centile. His measurements are plotted on the chart (which was printed extremely faint. You could only make out the crosses and some of the centiles. It appeared the child was crossing multiple centiles since young age). What is the cause? [Constitutional delay; Malabsorption; Hypothyroidism; Familial short stature]

A

Malabsorption;

How well did you know this?
1
Not at all
2
3
4
5
Perfectly
112
Q

Hyperactive child at home. Doesn’t pay attention at school. Diagnosis?

A

ADHD

How well did you know this?
1
Not at all
2
3
4
5
Perfectly
113
Q

Child takes 4 tablets of grandmothers benzodiazepines 4 hours ago. She’s currently asleep, but was fully
awake with GCS 15. Your next step? [Activate charcoal; Gastric Lavage; IV flumenazil; admit for
monitoring]

A

admit for
monitoring

Only give with rest depression

How well did you know this?
1
Not at all
2
3
4
5
Perfectly
114
Q

Afebrile child with D+V. Description alludes to shock. She is 15kg. What do you give her initially?

A

[300ml bolus of 0.9% saline]

How well did you know this?
1
Not at all
2
3
4
5
Perfectly
115
Q

What defines a mild learning disability? [IQ: <80/<70/<60/<50/<30]

A

<70

How well did you know this?
1
Not at all
2
3
4
5
Perfectly
116
Q

Which of the following is not an RF for suicide? [FHx of depression; FHx of suicide; Prev suicide attempt;
Male; Heroin-use]

A

FHx of depression

How well did you know this?
1
Not at all
2
3
4
5
Perfectly
117
Q

75 yr lady bought in with daughter. “She” (not sure who it was referring to) says is anxious, being
increasingly forgetful lately, thinks she has dementia. Diagnosis?

A

A: Depression.

How well did you know this?
1
Not at all
2
3
4
5
Perfectly
118
Q

You’re a GP. Elderly person presents with a history of the classical signs of Alzheimer’s. What do you do
next? [Prescribe donazepil; refer to memory clinic; MRI head; reassure]

A

memory clinic

How well did you know this?
1
Not at all
2
3
4
5
Perfectly
119
Q

What bloods would you test for, to monitor Lithium toxicity? [U&E/Lithium levels/TFTs, Lithium levels,
TFTs, Nothing unless symptomatic, U&E/Lithium, TFT/Lithium]

A

U&E/Lithium levels/TFTs

How well did you know this?
1
Not at all
2
3
4
5
Perfectly
120
Q

A person with a diagnosis of Bulimia nervosa. BMI 22. Wants to get help. Management? [CBT; CBT+
Fluoxetine; Fluoxetine; Psychoanalytic therapy]

A

CBT

How well did you know this?
1
Not at all
2
3
4
5
Perfectly
121
Q

A teenage girl takes an OD of paracetamol after being “dumped” by boyfriend. Which feature is likely to
suggest further suicide risk? [She got dumped for another girl; She took the OD in front of him saying she wanted to teach him a lesson; N-acetyl cysteine was required; she felt hopeless]

A

she felt hopeless

How well did you know this?
1
Not at all
2
3
4
5
Perfectly
122
Q

Elderly lady consented for operation by GP. Day of op decides not to have it but son insists she should go ahead. What should the doctor do. Options: [act in best interests; take word of the son as consent; assess the patient’s capacity;]

A

assess the patient’s capacity

How well did you know this?
1
Not at all
2
3
4
5
Perfectly
123
Q

When interviewing a recovered Schizophrenic, what on MSE would make you think they were getting relapse? [Visual hallucination; being withdrawn;]

A

Visual hallucination

How well did you know this?
1
Not at all
2
3
4
5
Perfectly
124
Q

Lady with social phobia. What feature would typically be associated with it? [Not liking big trains, people noticing you blush]

A

people noticing you blush

How well did you know this?
1
Not at all
2
3
4
5
Perfectly
125
Q
  1. Paeds derm treatment – also featured in 2014 paper. Options were a load of medicated creams/lotions + no treatment required.
    a. Nappy rash with satellite lesions
    b. Nappy rash with flexural sparing
    c. Chicken Pox
    d. Scabies
    e. Impetigo
A

a. Nappy rash with satellite lesions [A: Clotrimazole]
b. Nappy rash with flexural sparing [A: Zinc barrier cream]
c. Chicken Pox [A: Do nothing?]
d. Scabies [A: Permethrin]
e. Impetigo [A: Fusidic Acid]

How well did you know this?
1
Not at all
2
3
4
5
Perfectly
126
Q
  1. Mental Health Act
    [Options: Section 2; Section 3; Section 4; Section 5(2); Section 5(3); Deprivation of Liberty Safeguards (DoLS);
    Mental capacity act; Section 135 (even explained what it was! – taking someone to a place of safety from private property); No detention required]
    a. Inpatient with Mania detained under section 2, reaching the end of the term. He is still not showing signs of improvement and poses a risk to himself/others.
    b. Neighbours concerned about a man who they haven’t seen in a while. Psych nurse goes to visit and finds him confused, aggressive etc. She calls paramedics who give an IV infusion of saline.
    c. Doctor in a medical ward. Patient with some psych condition.
A

a) section 3
b) no detention
c) 5(2)

How well did you know this?
1
Not at all
2
3
4
5
Perfectly
127
Q
  1. Multiple pregnancies – [Options: Monochorionic; Monoamniotic; Dichorionic; Molar pregnancy; Twin-twin transfusion;]

Two fetal poles, one gestational sac

Snowstorm appearance

A

a) monochorionic

b) molar

How well did you know this?
1
Not at all
2
3
4
5
Perfectly
128
Q
  1. Paeds
    joint problems

a. Child bought in by grandmother. Said to have knocked knee in cupboard which has become swollen. His brother had swelling having banged head when he was young.
b. Child with posterior rib fractures and some other patterns of injuries
c. Girl with painful hot swollen knee joints. PMH of hot/swollen/painful wrist.
d. Girl not able to weight bear. Recently recovered from viral illness.
e. Girl not able to weight bear. Some joint(s) hurt. Mum has rheumatoid arthritis.

A

a) [Haemophilia]
b) [NAI]
c) [Septic arthritis]
d) [Post-viral synovitis]
e) [JIA]

How well did you know this?
1
Not at all
2
3
4
5
Perfectly
129
Q
  1. Paeds milestones - what age would you expect most children to have achieved these?

a. Smiling
b. Sitting unsupported -
c. Walking -
d. 2-3 word sentences
e. Pincer Grip

A

a) 6 weeks
b) 7/8months Sits without support (Refer at 12 months)
c) 13-15 months Walks unsupported (Refer at 18 months)
d) 3 years = 3-5 word sentences
e) 12 month Good pincer grip, early 9 months

How well did you know this?
1
Not at all
2
3
4
5
Perfectly
130
Q
  1. Jaundice
    [The usual options: ABO incompatibility; Biliary atresia; Physiological; Sepsis; Criggler-najjar etc.]

a. 28 day child with pale stools.
b. Child is blood group O. Mother is blood group AB.
c. Mum had group B strep.
d. A child who had jaundice for a few days (day 2-7). He’s fine now.

A

a) biliary atresia
b) ABO incompatibility
c) Sepsis
d) physiological

How well did you know this?
1
Not at all
2
3
4
5
Perfectly
131
Q
  1. Child with funny turns [Options: Infantile spasms; Reflex anoxic seizures; Breath holding attacks; Absence seizures ]

a. ~8 month who keeps clenching fists and bringing arms out towards parents.
b. Child falls. Parents pick him but has a tonic-clonic seizure. Is complete well afterwards.
c. Child at school. Complains of unusual smell. Then seems to ‘space out’. Afterwards she’s very drowsy
and sleeps for a few hours in the school nurses office. She has no recollection of what happened.

A

a) infantile spasms
Wests = developmental delay, infantile spasms, hypsarrythmia

b) reflex anoxic
c) absence

How well did you know this?
1
Not at all
2
3
4
5
Perfectly
132
Q
  1. Diarrhoea [options: toddlers diarrhoea; constipation with overflow; Crohn’s; Ulcerative colitis; …. ]
    a. Redcurrent jelly stools
A

Intussusception

How well did you know this?
1
Not at all
2
3
4
5
Perfectly
133
Q
  1. Psych

a. Student who recently finished exams found disorientated and slurring his words or similar - basically

A

alcohol excess

How well did you know this?
1
Not at all
2
3
4
5
Perfectly
134
Q
  1. Kids development
    a. MINECRAFT! 7 year old who refuses to do his homework and stays up late playing Minecraft on his iPad.
    Parents are worried about him.
A

Normal behaviour

How well did you know this?
1
Not at all
2
3
4
5
Perfectly
135
Q

Gardasil - what 4 does it protect against. Name the one from the list: HIV, HSV1, HSV2, HPV11, HPV29?

A

6,11,16,18

How well did you know this?
1
Not at all
2
3
4
5
Perfectly
136
Q

UTI in first trimester of pregnancy. What would be the safest and most effective treatment? Trimethoprim, Ciprofloxacin, Doxycycline, Cefalexin, Meropenem

A

Cefalexin

How well did you know this?
1
Not at all
2
3
4
5
Perfectly
137
Q

Amenorrhoea for 4 months - what do u do?

A

Pregnancy test, FSH, need a withdrawal if PCOS

How well did you know this?
1
Not at all
2
3
4
5
Perfectly
138
Q

19 year old abdo swelling, weight gain, irregular periods usually, can’t remember when last period was, denies being sexually active. What is the first test you would do?

A

Preg test

How well did you know this?
1
Not at all
2
3
4
5
Perfectly
139
Q

Girl with cystic ovaries on US and something else. What other symptom would be the best indicator of her having polycystic ovaries? Dysmenorrhoea, Hirsutism, Obesity, Acne.

A

Hirsutism

How well did you know this?
1
Not at all
2
3
4
5
Perfectly
140
Q

Woman with PCOS. Best medication to increase fertility.

A

Clomifene

How well did you know this?
1
Not at all
2
3
4
5
Perfectly
141
Q

Woman with signs of premature ovarian failure. What test would be best to confirm this diagnosis: Oestradiol, Testosterone, FSH, LH

A

FSH

How well did you know this?
1
Not at all
2
3
4
5
Perfectly
142
Q

Man with azoospermia. What would be the most likely cause? Hx of mumps, hx of testicular torsion, Varicocele

A

Varicocele

How well did you know this?
1
Not at all
2
3
4
5
Perfectly
143
Q

Post menopausal woman with a PV bleed. What ix should you do? Laparotomy and hysteroscopy, outpatient US with endometrial biopsy, CT, MRI…

A

outpatient US with endometrial biopsy

How well did you know this?
1
Not at all
2
3
4
5
Perfectly
144
Q

Asymptomatic woman, nulliparous, found to have a 5.4cm unilocular ovarian cyst on US, no fhx. Mgmt/Ix? USS guided cyst aspiration, laparoscopic removal of cyst, discharge and safety-net, rearrange USS in 3 months and Ca12.5 follow-up, yearly follow up

A

If pre menopausal – yearly follow up as over 5cm…
Women with simple ovarian cysts of 50–70 mm in diameter should have yearly ultrasound follow-up and those with larger simple cysts should be considered for either further imaging (MRI) or surgical intervention.

How well did you know this?
1
Not at all
2
3
4
5
Perfectly
145
Q

Lady with cyclic pain 1 week before her period starts, trying for a baby for one year Diagnosis?

A

endometriosis

How well did you know this?
1
Not at all
2
3
4
5
Perfectly
146
Q

Woman 8 weeks after normal vaginal delivery and second degree tear, still bleeding and mild lower pelvic pain. Diagnosis? Normal menstruation, PID, endometritis, lochia.. (lochia for 2-6weeks, period returns 6-8 weeks)

A

endometritis

How well did you know this?
1
Not at all
2
3
4
5
Perfectly
147
Q

What is a 23 week USS useful for? Nuchal thickness for Down’s, congenital heart disease identification, predicting position of placenta at term..

A

congenital heart disease identification

NORMALLY 18-20 weeks

How well did you know this?
1
Not at all
2
3
4
5
Perfectly
148
Q

What causes increased urinary volume and frequency in the first trimester? Increased GFR, pressure of uterus on bladder, glycosuria.

A

pressure of uterus on bladder

How well did you know this?
1
Not at all
2
3
4
5
Perfectly
149
Q

Effect of taking fluoxetine during pregnancy on baby?

A

Persistent pulmonary hypertension of the newborn,

How well did you know this?
1
Not at all
2
3
4
5
Perfectly
150
Q

Woman with Nexplanon. Most likely reason for wanting to change contraceptive? Weight gain, acne, mood swings,

A

irregular bleeding

How well did you know this?
1
Not at all
2
3
4
5
Perfectly
151
Q

Pregnant lady being domestically abused by husband and scared to go home. What do you do in GP? Send her home and ask her to come back with husband, give her a leaflet about domestic abuse, call and arrange emergency accommodation

A

call and arrange emergency accommodation

How well did you know this?
1
Not at all
2
3
4
5
Perfectly
152
Q

Woman with a slow growing painless lesion on labia.

A

Genital warts

How well did you know this?
1
Not at all
2
3
4
5
Perfectly
153
Q

Woman with tender lump inside her vagina.

A

Bartholins cyst

How well did you know this?
1
Not at all
2
3
4
5
Perfectly
154
Q

Young woman, pain during sex, ‘Strawberry cervix’ on examination

A

TV

How well did you know this?
1
Not at all
2
3
4
5
Perfectly
155
Q

Woman with cheesy white discharge

A

candida

How well did you know this?
1
Not at all
2
3
4
5
Perfectly
156
Q

Old woman complaining of superficial dyspareunia

A

atrophic vagina

How well did you know this?
1
Not at all
2
3
4
5
Perfectly
157
Q

HRT options: COCP, Oral Oestrogen, Oral Progesterone, Transdermal oestrogen, Transdermal progesterone, all have continuous or cyclical, Bisphosphonates, Testosterone.

Women who has gone through menopause, had a hysterectomy. Wants HRT mainly to prevent osteoporosis and treat her hot flushes. Doesn’t want to take tablets.

Swimmer wants to treat hot flushes, doesn’t want a patch.

Woman with premature ovarian failure, wants to have periods.

Woman who just wants to treat osteoporosis, 62.

A

Transdermal oestrogen

Oral both (either cyclical or continuous depending on last period )

COCP or cyclical combined HRT (likely to have a bleed but not guaranteed)

bisphosphonates

How well did you know this?
1
Not at all
2
3
4
5
Perfectly
158
Q

Breast
Option: mastitis, intraductal papilloma, malignancy, abscess, genital warts, Bartholins cyst

Lady high White cell count, fever, lump in breast

General breast tenderness - left sided, breastfeeding

Small slow growing lesion on labia, pregnant lady

Small painful lump inside vagina of sexually active woman

A

abscess

mastitis

genital warts

bartholins

How well did you know this?
1
Not at all
2
3
4
5
Perfectly
159
Q

Bacteria causing acne?

A

Propionibacteria acnes

How well did you know this?
1
Not at all
2
3
4
5
Perfectly
160
Q

Kid with history of anal fissure - what is your first cause of action? - inspect anal region, do DRE with little finger

A

inspect anal region

How well did you know this?
1
Not at all
2
3
4
5
Perfectly
161
Q

A level student has recently come back from nigeria, with symptoms of jaundice, mild anaemia and fever with malaise,arthralgia.

A

Malaria

How well did you know this?
1
Not at all
2
3
4
5
Perfectly
162
Q

Kid has pain in outer ear, ear was protruding outwards, and there was a lump behind his ear.

A

Mastoiditis

How well did you know this?
1
Not at all
2
3
4
5
Perfectly
163
Q

Kid with temp of 39, cap refill 6s, generally unwell + bulging fontanelles, no description of rash. Options: Meningococcal septicamia, uti, pnemonia

A

Meningococcal septicamia

How well did you know this?
1
Not at all
2
3
4
5
Perfectly
164
Q

4yo kid having acute asthma attack, given iv salbutamol and hydrocortisone. Sats still low, no chest sounds on auscultation. What do you do/give next? IM adrenaline, call for senior help, start atrovent

A

call for senior help

How well did you know this?
1
Not at all
2
3
4
5
Perfectly
165
Q

Kid with cervical lymphadenopathy, fever, sore throat, red tongue with white spots. What does she have? Scarlet Fever, measles, chicken pox.

A

measles

How well did you know this?
1
Not at all
2
3
4
5
Perfectly
166
Q

Child who had bulimia. What gives it away? (Options were dental enamel caries, striae, lacerations on wrist…)
bronchopulmonary dysplasia

A

dental enamel

How well did you know this?
1
Not at all
2
3
4
5
Perfectly
167
Q

4 month old, about to have 3 batch of primary vaccinations. Which would be a complete contraindication to having the vaccine? Confirmed history of pertussis as a baby, currently ill with a fever of 38.5, got a rash at site of last vaccination, severe cow’s milk allergy,

A

currently ill with a fever of 38.5

How well did you know this?
1
Not at all
2
3
4
5
Perfectly
168
Q

Perianal itching especially at night. What’s the best treatment option? (Options: Mebendazole, Cotrimazole)

A

Mebendazole

How well did you know this?
1
Not at all
2
3
4
5
Perfectly
169
Q

5 month old with cough, runny nose, fever. Examination of chest you hear wheeze. What’s the main pathogen that causes this RTI? Options: Streptococcus Pneumoniae, Respiratory Syncytial Virus, Mycoplasma pneumoniae

A

Respiratory Syncytial Virus

How well did you know this?
1
Not at all
2
3
4
5
Perfectly
170
Q

Respiratory Disease. Options: Meconium Aspiration, Group B Strep infection, PCP pneumonia, transient tachypnoea of newborn, Surfactant deficiency, , diaphragmatic hernia.

a) Heart sound not heard, scaphoid chest.
b) Baby was born at 41 weeks via emergency C section due to foetal distress. Needed ventilation straight away. X ray showed hyper inflated lungs with areas of consolidation. What does he have?
c) Baby born at 37 weeks, via forceps. Showing signs of resp distress. CXR shows areas of consolidation throughout.
d) Prem baby, resp distress, CXR looks like ground glass.

A

Diaphragmatic hernia

Meconium aspiration (mature fetus, distress caused gasping breath in) patchy opacities, hyperinflation, no air bronchograms )

GBS

Surfactant deficiency

How well did you know this?
1
Not at all
2
3
4
5
Perfectly
171
Q

Treatments of Paed Rashes. Options: Hydrocortisone, Permethrin, Zinc…, Anti-fungal, Histamine cream, Nothing,

Nappy rash flexure sparing

Nappy rash with satellite lesions

Anal itch, worst at night

Scabies

Chickenpox

A

zinc

antifungal

mebendazole

permethrin

nothing (can use histamine cream) OR topical calamine

How well did you know this?
1
Not at all
2
3
4
5
Perfectly
172
Q

Joint problems (same as 2015)

Options: haemophilia, JIA, post-viral synovitis, septic arthritis, NAI

Grandmother brings boy in with swollen knee. Boy’s brother died young of a minor head injury.

Posterior rib fractures

Unwell and not able to weight bear, fever

Recent URTI - not able to weight bear

A

Haemophilia

Nai

septic arthritis

Recent URTI - not able to weight bear post viral synovitis

How well did you know this?
1
Not at all
2
3
4
5
Perfectly
173
Q

Suicide risk. What is the highest predictor of doing it again? bad relationship with mum, feeling hopeless about future, previous self harm

A

feeling hopeless about future

How well did you know this?
1
Not at all
2
3
4
5
Perfectly
174
Q

80 year old man with new onset dementia, needs MRI but refuses. You need someone to make decision for him. - (options: talk to family with his permission, ask independent mental health advocate)

A

talk to family with his permission

How well did you know this?
1
Not at all
2
3
4
5
Perfectly
175
Q

Guy who has visual hallucination and likely to fall over. but he is not giving consent to help regarding his falls and delirium. He wants to leave hosp. Difficult to understand what he is saying
Q: what is a big reason that makes you think he does not have consent? (Options: due to lack of understanding, due to lack of processing, due to his visual hallucination, due to him not communicating properly)

A

due to him not communicating properly

How well did you know this?
1
Not at all
2
3
4
5
Perfectly
176
Q

Woman on SSRI, wanted to get pregnant, was wondering what risk it may have on her baby: Stillbirth, Pulmonary htn of the newborn, delayed labour, hypoglycemia at birth.

A

Pulmonary htn of the newborn

How well did you know this?
1
Not at all
2
3
4
5
Perfectly
177
Q

Woman wants to know what risk her baby has of getting schizophrenia, since the baby’s father has it. 7-9%, 12-15%, 20-25%, 1%…

A

should be 10% (48% if both parents have it)

How well did you know this?
1
Not at all
2
3
4
5
Perfectly
178
Q

Woman 5 weeks postpartum feels very sad, unable to cope, teary. Dx?

A

Postnatal depression (baby blues usually pnly for 1-2 weeks)

How well did you know this?
1
Not at all
2
3
4
5
Perfectly
179
Q

What blood test should be frequently done if someone is on Lithium? Thyroid function, liver function, adrenal function, FBC,

A

Thyroid function

How well did you know this?
1
Not at all
2
3
4
5
Perfectly
180
Q

Guy on medication for schizophrenia get muscle rigidity, altered consciousness, high blood pressure, tachycardia. Mgmt?

A

stop antipsychotic and dantrolene and bromocriptine

How well did you know this?
1
Not at all
2
3
4
5
Perfectly
181
Q

60 year old woman, short term memory loss, struggling to complete her normal daily tasks, ataxia and dysphagia. What would you see on MRI?

A

NPH most likely

How well did you know this?
1
Not at all
2
3
4
5
Perfectly
182
Q

40 year old man with moderate learning disability. He has a Hb of ~6 (below the normal range). Refuses blood transfusion but happy to take oral iron therapy. What law do you use to assess his decision (or something along those lines). [Options: Common Law; Mental Capacity Act; Mental Health Act; Disability Discrimination Act; ?European Convention on Human Rights]

A

MCA

Learning disability is NOT a mental health DISORDER

How well did you know this?
1
Not at all
2
3
4
5
Perfectly
183
Q

Man who had come into GP for peeling of skin on his hands. Excessive hand washing 6 times daily which has got worse since his elderly father died 3 months ago following a wound infection(?) post operatively. (Options: Adjustment Disorder (lasts 6 months), OCD)

A

OCD

How well did you know this?
1
Not at all
2
3
4
5
Perfectly
184
Q

Man with treatment resistant schizophrenia on clozapine, recently stopped smoking. High level of clozapine now. Most likely consequence? Agranulocytosis, seizures.. (seizures found in agranulocytosis)

A

Agranulocytosis

It is well documented that cigarette smoke can induce cytochrome P450 (CYP) isoenzymes, specifically CYP1A1, CYP1A2, and CYP2E1. Because clozapine is primarily metabolized by CYP1A2 (approximately 70%), smoking can induce clozapine metabolism and abruptly stopping smoking can increase clozapinelevels.

How well did you know this?
1
Not at all
2
3
4
5
Perfectly
185
Q

Definitions
Options: Illusion, formication, micropsia, pseudo hallucination

Person sees flower on wallpaper - sees them as moving snakes:

Person sees things smaller

Definition of feel insects under skin.

A

Illusion

micropsia

fornication

How well did you know this?
1
Not at all
2
3
4
5
Perfectly
186
Q

Agitated psych patient threating violence - what do you do? (Options: talk to consultant, call the police, talk to pt)

A

talk to pt

How well did you know this?
1
Not at all
2
3
4
5
Perfectly
187
Q

Diagnosis:
Options: Somatic syndrome, malignancy, Munchausens, Borderline Personality Disorder

50 y woman ,constant stomach pain, had many ix eg multiple laprascopies with nothing found. Comes to A&E saying she needs another laproscopy.

Young girl who has come into A&E multiple times with different presentations, nothing found for any of them.

Woman with abdominal pain, weight loss, lethargy and feeling low.

Man who drinks and takes some drugs. Split up with girlfriends, cutting himself.

A

Maunchausens

Somatic

Malignancy

BPD

How well did you know this?
1
Not at all
2
3
4
5
Perfectly
188
Q
  1. 14 year old primary amenorrhoea + ejection systolic murmur
A

Turner’s / coarction

How well did you know this?
1
Not at all
2
3
4
5
Perfectly
189
Q
  1. What gives you macrosomia
A

gestational diabetes

How well did you know this?
1
Not at all
2
3
4
5
Perfectly
190
Q
  1. Cyclical pain, no heavy menstrual bleed, never sexually active
A

endometriosis

How well did you know this?
1
Not at all
2
3
4
5
Perfectly
191
Q
  1. Ethics – 14 year old pregnant, comes with sister, wants a TOP. What should you do?
A

Persuade her to tell her parents if she refuses and gillick competent, you can give her TOP.

How well did you know this?
1
Not at all
2
3
4
5
Perfectly
192
Q
  1. What situation would you use donor eggs?
A

POF

How well did you know this?
1
Not at all
2
3
4
5
Perfectly
193
Q
  1. What do you test for in Hep B infection antenatally?
A

HBsAg

How well did you know this?
1
Not at all
2
3
4
5
Perfectly
194
Q
  1. What is not a risk factor for primary PPH?

a. B thalassaemia trait
b. Retained products
c. Sepsis
d. Vaginal tear
e. Multiparity

A

a. B thalassaemia trait

How well did you know this?
1
Not at all
2
3
4
5
Perfectly
195
Q
  1. At antenatal check, woman with BP 150/90, what would you do?
A

Admit and assess

How well did you know this?
1
Not at all
2
3
4
5
Perfectly
196
Q
  1. Urogynae – leak urine when laughing and going up stairs, initial management?
A

Pelvic floor exercise

How well did you know this?
1
Not at all
2
3
4
5
Perfectly
197
Q
  1. What is the treatment for a bartholian abscess
A

malsupialisation

How well did you know this?
1
Not at all
2
3
4
5
Perfectly
198
Q
  1. Women with previous GDM what is the best way to investigation her blood glucose? OGTT at 28w, OGTT as soon as possible after booking (later at 28 weeks if normal)
A

OGTT as soon as possible after booking (later at 28 weeks if normal)

How well did you know this?
1
Not at all
2
3
4
5
Perfectly
199
Q
  1. Women with APH (spotting) otherwise well, what is the most important thing to rule out? Ectopic pregnancy, placenta praevia
A

Ectopic pregnancy

How well did you know this?
1
Not at all
2
3
4
5
Perfectly
200
Q
  1. What type of contraception can increase risk of osteoporosis?
A

Depot injections

How well did you know this?
1
Not at all
2
3
4
5
Perfectly
201
Q
  1. Women with excessive vomiting, under what circumstance would you admit her?
A

Ketonuria

How well did you know this?
1
Not at all
2
3
4
5
Perfectly
202
Q
  1. What is likely to cause this man’s azoospermia?
A

Varicocele, Mumps orchitis

How well did you know this?
1
Not at all
2
3
4
5
Perfectly
203
Q
  1. Which drug is likely to be teratogenic? [antiepileptics]
A

Sodium valproate

How well did you know this?
1
Not at all
2
3
4
5
Perfectly
204
Q

Someone comes in at 28w with a Hb of 10.5 what would you do?

A

Simple advice - leafy Haemoglobin levels outside the normal UK range for pregnancy (that is, 11 g/100 ml at first contact and 10.5 g/100 ml at 28 weeks) should be investigated and iron supplementation considered if indicated.

How well did you know this?
1
Not at all
2
3
4
5
Perfectly
205
Q

Menopause treatment – what HRT would you give

a. Premature menopause and wants a bleed
b. Someone want something for their bones
c. Women with menopausal symptoms, hysterectomised, does not want to take oral tablets
d. Perimenopausal women with menopausal symptoms, irregular menstruation, does swimming and does not patch
e. Women with menopausal symptoms and has eczema

A

cyclical HRT or cocp

bisphosphonate

transdermal oestrogen HRT

cyclical oral HRT

oral combined HRT

How well did you know this?
1
Not at all
2
3
4
5
Perfectly
206
Q
  1. STIs

a. Clue cells
b. Strawberry cervix
c. Painful multiple lesions on labia
d. Lichen sclerosis
e. Curdy white-yellow discharge
f. Blue dots on cervix

A

BV

Trichomonas and wet slide

herpes

thin vulval epithelium, white plaques

candidiasis

endometriosis (blue/ dark dots) nathobian cyst – yellow dots

How well did you know this?
1
Not at all
2
3
4
5
Perfectly
207
Q

a. Mother with stillborn baby, generalized oedematous when born, mother had fever at 18w with rash on trunk

A

parvovirus

How well did you know this?
1
Not at all
2
3
4
5
Perfectly
208
Q
  1. What is the likely diagnosis
    a. Smear comes back as moderate dyskaroysis
    b. 76 year old had a single brown discharge
    c. Intermittent pain + vomiting
A

CIN2

atrophic vaginitis

ovarian torsion

How well did you know this?
1
Not at all
2
3
4
5
Perfectly
209
Q
  1. Gynae Management
    a. 47yo women with menorrhagia + dysmenorrhea, US showed multiple fibroids –

b. 32 yo has 2 children done with family, had COCP before
c. Women going away for holiday and would like to delay her periods.

A

hysterectomy

mirena

Norethisterone

How well did you know this?
1
Not at all
2
3
4
5
Perfectly
210
Q
  1. Obstetric complications
    a. Mother being prepared for CS, sudden tingling around her mouth?

b. Mother being prepared for CS and has a regional block, sudden tachycardia, SOB, difficulty in breathing
c. Mother had previous CS, sudden abdo pain and abnormal CTG
d. Another anaphylaxis one

A

Spinal block

anaphylaxis

uterine rupture

How well did you know this?
1
Not at all
2
3
4
5
Perfectly
211
Q
  1. Kid with fever of >39 what to do next
A

Septic screen

How well did you know this?
1
Not at all
2
3
4
5
Perfectly
212
Q

When disappear?

a. Moro reflex newborns
b. Asymmetrical neck reflex newborns
c. Palmar and planter grasp newborn
d. Rooting newborn

A

3/4 months (drop extend arms)

3months (turn and extend arm)

5/6months

4months

How well did you know this?
1
Not at all
2
3
4
5
Perfectly
213
Q
  1. Mother with girl who doesn’t speak
A

refer for hearing assessment

How well did you know this?
1
Not at all
2
3
4
5
Perfectly
214
Q
  1. Kid fitting for over 5mins, normal glucose, what do you give?
A

Rectal diazepam/ bucal midazolam/IV lorazepam if you have access

How well did you know this?
1
Not at all
2
3
4
5
Perfectly
215
Q
  1. Dehydration + ill kid – weight 15kg what do you give initially?
A

300ml bolus IV

How well did you know this?
1
Not at all
2
3
4
5
Perfectly
216
Q
  1. Aspiration pneumonia what would you do 1st?
A

ABx

How well did you know this?
1
Not at all
2
3
4
5
Perfectly
217
Q
  1. Strawberry tongue
A

scarlet fever

How well did you know this?
1
Not at all
2
3
4
5
Perfectly
218
Q
  1. What is the purpose of debriefing after a child’s death in resuscitation?
A

To address emotional needs of everyone in the team.

How well did you know this?
1
Not at all
2
3
4
5
Perfectly
219
Q
  1. Child who pass stool every few days, when he does go, stools are pellet like and smelly, what is the likely diagnosis?
A

Overflow constipation

How well did you know this?
1
Not at all
2
3
4
5
Perfectly
220
Q
  1. Child with pruritus ani worse at night, what would you give to treat?
A

Mebendazole

How well did you know this?
1
Not at all
2
3
4
5
Perfectly
221
Q
  1. Skin condition treatment wtf
    a. Impetigo
    b. Rash involving flexures
    c. Rash not involving flexures
    d. Scapies
    e. Chickenpox
A

a. fusidic acid (first line)/ oral flucloxacillin
b. imidazole
c. zinc
d. permethrin
e. calmamine lotion

How well did you know this?
1
Not at all
2
3
4
5
Perfectly
222
Q
  1. By WHAT AGE would you refer the following kids if they haven’t achieve the following milestones
    a. Sit without support

b. Walk
c. Hops on one leg
d. Pincer grip
e. Smiles

A
  1. By WHAT AGE would you refer the following kids if they haven’t achieve the following milestones
    a. Sit without support – normally by 6m with round back, refer by 8m (limit age 9 months)
    b. Walk – normally by 15m, refer by 18m (limit age 18 months)
    c. Hops on one leg – normally by 4y, refer by 5y (5y was the only upper limit answer)
    d. Pincer grip – normally by 10m, refer by 12m (limit age 12 months)
    e. Smiles – normally by 6w, refer by 8w (limit age 8 weeks)
How well did you know this?
1
Not at all
2
3
4
5
Perfectly
223
Q
  1. Poisons + investigations
    a. Drunk/ intoxicated kid
    b. Dehydrated, seizures, mother been giving some herbal oral rehydration fluid or something
    c. Sick kid with fever, vomiting, photophobia
A

a. Drunk/ intoxicated kid – urine drug screen
b. Dehydrated, seizures, mother been giving some herbal oral rehydration fluid or something – check plasma sodium
c. Sick kid with fever, vomiting, photophobia - LP

How well did you know this?
1
Not at all
2
3
4
5
Perfectly
224
Q
  1. Respiratory problems
    a. Neonate getting progressively worse at breathing over first 3h with opacities
    b. Still needs oxygen
    c. Ground grass appearance
    d. Nitrogen washout test
    e. Meconium aspiration
A
  1. Respiratory problems
    a. Neonate getting progressively worse at breathing over first 3h with opacities - Group B strep pneumonia
    b. Still needs oxygen - Bronchopulmonary dysplasia
    c. Ground grass appearance - Respiratory distress syndrome/ primary surfactant deficiency
    d. Nitrogen washout test – congenital heart disease (after all that revision this was the only question that came up. Great)
    e. Meconium aspiration – asymmetrical patchy, opacities
How well did you know this?
1
Not at all
2
3
4
5
Perfectly
225
Q
  1. Childhood malignancies presentation
    a. Nephroblastoma, same as Wilms
    b. Retinoblastoma
    c. Osteoid sarcoma
    d. ALL
    e. Posterior fossa tumour
A

a. Nephroblastoma, same as Wilms – before 5, large abdo mass
b. Retinoblastoma – white pupillary reflex and a squint. (chromosome 13)
c. Osteoid sarcoma – painful bone
d. infection, anaemia, limp, bruising
e. Posterior fossa tumour – medulloblastoma

How well did you know this?
1
Not at all
2
3
4
5
Perfectly
226
Q
  1. Which virus caused the following conditions
    a. Kid with sore throat, cervical lymphademopathy, been given antibiotics, rash comes on.
    b. Rash that started from behind the ears and spread to trunk, parents are vegetarians and kid goes to school in north London
A

a. Kid with sore throat, cervical lymphademopathy, been given antibiotics, rash comes on. EBV
b. Rash that started from behind the ears and spread to trunk, parents are vegetarians and kid goes to school in north London – measles

How well did you know this?
1
Not at all
2
3
4
5
Perfectly
227
Q
  1. Gastroinestional
    a. Intermittent pain, dehydrated, vomited 3 times
    b. Sudden abdo pain, well child, something indentable on the L lower quadrant
    c. Scaphoid abdomen (never heard of that term before until the exam!)
A

a. Intermittent pain, dehydrated, vomited 3 times - Intussesception
b. Sudden abdo pain, well child, something indentable on the L lower quadrant – constipation
c. Scaphoid abdomen (never heard of that term before until the exam!) – diaphragmatic hernia

How well did you know this?
1
Not at all
2
3
4
5
Perfectly
228
Q
  1. Ethics – 78 year old fell and sustained a fractured NoF and refuse surgery, son is a lawyer and says his mother does not have capacity, what should you do?
A

Access capacity

How well did you know this?
1
Not at all
2
3
4
5
Perfectly
229
Q
  1. Someone who just started on an antipsychotic and becomes tachycardia, hyperthermia, sweating, urine drug screen negative.
A

Neuroleptic malignancy syndrome

How well did you know this?
1
Not at all
2
3
4
5
Perfectly
230
Q
  1. Which of the following condition would a kid’s twin brother has if he was diagnosed with it?
A

ADHD

How well did you know this?
1
Not at all
2
3
4
5
Perfectly
231
Q
  1. Diagnosed bipolar disorder – sudden renal failure, what do you check? Lithium levels, U&Es, Thyroid
A

Lithium levels

How well did you know this?
1
Not at all
2
3
4
5
Perfectly
232
Q
  1. Someone is brought in by their mother to AE and appears psychotic what section would you put them under?
A

Section 2 (in emerhency could do 4)

How well did you know this?
1
Not at all
2
3
4
5
Perfectly
233
Q
  1. Someone with acute dystonia what would you give?
A

Procyclidine

How well did you know this?
1
Not at all
2
3
4
5
Perfectly
234
Q
  1. Someone with bulimia, BMI 22, well and keen to get treatment.
A

CBT

How well did you know this?
1
Not at all
2
3
4
5
Perfectly
235
Q
  1. Out of the following people, who is likely to commit suicide?
    a. Women
    b. People who work in managerial roles
    c. People who are 40-50
    d. People who are married
A

c. People who are 40-50

How well did you know this?
1
Not at all
2
3
4
5
Perfectly
236
Q
  1. If all these children have learning disability, who is most likely to have mental disorder later on?
    a. Child with epilepsy
    b. Child with long term illness
    c. Child with malignancy
A

b. Child with long term illness

How well did you know this?
1
Not at all
2
3
4
5
Perfectly
237
Q
  1. Women with mild depression what is your management
A

CBT

How well did you know this?
1
Not at all
2
3
4
5
Perfectly
238
Q
  1. Women with sudden confusion what would you do first?
A

Urine dip

How well did you know this?
1
Not at all
2
3
4
5
Perfectly
239
Q
  1. One of the criteria for learning disability?
A

IQ <70

How well did you know this?
1
Not at all
2
3
4
5
Perfectly
240
Q
  1. Psychiatry drugs
    a. Failed 2 antipsychotics
    b. Someone who was started on haloperidol and has acute muscle spasm (acute dystonic)
    c. Someone who is aggressive and needs rapid tranqulisation
    d. Moderate Depression in young adult
A

a. Failed 2 antipsychotics – clozapine
b. Someone who was started on haloperidol and has acute muscle spasm (acute dystonic) – procyclidine
c. Someone who is aggressive and needs rapid tranqulisation – non psychotic: oral lorazepam, psychotic: lorazepam + haloperidol IM: lorazepam + haloperidol
d. Moderate Depression in young adult – CBT + medication (she said citalopram)

How well did you know this?
1
Not at all
2
3
4
5
Perfectly
241
Q
  1. Substance misuse

a. Something that works on 5HT receptors?

A

MDMA/ ectasy

How well did you know this?
1
Not at all
2
3
4
5
Perfectly
242
Q
  1. Psychiatry services who to refer to?
    a. Who would you refer a schizophrenic patient to if they are being discharged and needs support in the community regarding treatment?
    b. Someone with schizo and need help organizing their activities after they are being discharged?
    c. Girl who overdose in front of her boyfriend after a row, no previous suicide/ self-harm attempts. Good health. Only did it because she wanted attention from her boyfriend.
A
  1. Psychiatry services who to refer to?
    a. Who would you refer a schizophrenic patient to if they are being discharged and needs support in the community regarding treatment? Community psychiatric nurse (she said) HTT
    b. Someone with schizo and need help organizing their activities after they are being discharged? Occupational therapist
    c. Girl who overdose in front of her boyfriend after a row, no previous suicide/ self-harm attempts. Good health. Only did it because she wanted attention from her boyfriend. – GP
How well did you know this?
1
Not at all
2
3
4
5
Perfectly
243
Q

40 year old premature ovarian failure and wanted medication to deal with the symptoms of menopause. What would you prescribe her?

A

HRT

How well did you know this?
1
Not at all
2
3
4
5
Perfectly
244
Q

Woman is pregnant and HIV negative at booking but her partner is HIV positive - what do you do?

A

Nothing

How well did you know this?
1
Not at all
2
3
4
5
Perfectly
245
Q

What signifies onset of active labour?

A

Regular contractions

How well did you know this?
1
Not at all
2
3
4
5
Perfectly
246
Q

Woman with white discharge and itch. Which treatment?

A

Oral Fluconazole, Topical Clotrimazole

How well did you know this?
1
Not at all
2
3
4
5
Perfectly
247
Q

Pregnant woman with itchy feet, what investigation?

A

Liver function tests 

How well did you know this?
1
Not at all
2
3
4
5
Perfectly
248
Q

Results show azoospermia. How should they be managed? IVF, egg donation, ICSI, IUI

A

ICSI

How well did you know this?
1
Not at all
2
3
4
5
Perfectly
249
Q

Ovarian Cysts:
Options: Mature teratoma, Dysgerminoma, serous cystadenoma, endometroima
● Cyst contains hair and teeth.
● Ground glass appearance
● Woman has pain before periods and has been subfertile, cyst found on ovary.

A

● Cyst contains hair and teeth. - Teratoma
● Ground glass appearance - Endometrioma
● Woman has pain before periods and has been subfertile, cyst found on ovary - Endometrioma

How well did you know this?
1
Not at all
2
3
4
5
Perfectly
250
Q

Gynae Cancer - endometiral cancer, VIN, vulval cancer, cervical cancer

  • Post menopausal woman has bleeding
  • 70 year old lady has some spotting and says she uses some steroid cream for a “rash down there”
  • Woman had some bleeding and curettage of the endometrium had carcinomatous change (it genuinely said this or something like this!)
  • Lady is on Tamoxifen, which cancer does this increase the risk of? Endometrial cancer
A
  • endometiral cancer
  • lichen sclerosis
  • Endometrial cancer
  • Endometrial cancer
How well did you know this?
1
Not at all
2
3
4
5
Perfectly
251
Q

STIs

  • Green discharge
  • White discharge
A

STIs

  • Green discharge- trichomonas
  • White discharge- candid
How well did you know this?
1
Not at all
2
3
4
5
Perfectly
252
Q

Heart Disease:
A newborn appears to be in severe respiratory distress and appears blue. Despite being given high flow O2, his saturations remain at 65%.
What is the next best step to take with regards to his management?
● Chest X-Ray
● Infusion of Prostaglandin
● Surgery
● Indomethacin

A

● Infusion of Prostaglandin

How well did you know this?
1
Not at all
2
3
4
5
Perfectly
253
Q

3 months old baby with signs of HF, systolic murmur that radiates over the praecordium

A

VSD3

How well did you know this?
1
Not at all
2
3
4
5
Perfectly
254
Q

What is the most important thing to look at in follow up of HSP? ESR, FBC, urine protein and RBCs, platelets

A

urine protein and RBCs

How well did you know this?
1
Not at all
2
3
4
5
Perfectly
255
Q

6 year old child with 24 hour history of left peri-orbital swelling. Had an upper respiratory tract infection last week. Left proptosis, visual acuity was normal and had a fever of 38.9. What is the best diagnostic investigation? CT of nasal orbits, USS of nasal orbits, nasal endoscopy, intraocular pressure measurement, plain x-ray of nasal sinus

A

CT of nasal orbits

How well did you know this?
1
Not at all
2
3
4
5
Perfectly
256
Q

Boy with itchy bottom, what do you prescribe? Miconazole cream, mebendazole solution

A

mebendazole

How well did you know this?
1
Not at all
2
3
4
5
Perfectly
257
Q

Child with 6m of loose stools. Passed one hard blood streaked stool 10 days ago. What investigation? Colonoscopy, stool mc&s, anti TTG, DO NOTHING

A

DO NOTHING

How well did you know this?
1
Not at all
2
3
4
5
Perfectly
258
Q

Infant with episodes of throwing arms forward with fists clenched. Febrile seizure, focal seizure, infantile spasms, partial seizure

A

infantile spasms

How well did you know this?
1
Not at all
2
3
4
5
Perfectly
259
Q

Mother worried about 2.5 or 3? year old child’s bed wetting. Dry by day, wets bed at night. What do?

A

Reassure

How well did you know this?
1
Not at all
2
3
4
5
Perfectly
260
Q

Child with fever of 39, high resp rate, nurse says chest is clear, what investigation do you do? CXR, urinalysis

A

urinalysis

How well did you know this?
1
Not at all
2
3
4
5
Perfectly
261
Q

Child with fever, white exudate on one tonsil, diagnosis? Tonsillitis, Quinsy, diphtheria

A

Quinsy

How well did you know this?
1
Not at all
2
3
4
5
Perfectly
262
Q

Child with episodes of smelling strange things, hard to communicate with during these episodes, falls asleep for an hour after and doesn’t remember anything. Diagnosis? Focal seizure, absence seizure, tonic clonic, narcolepsy

A

Focal seizure

How well did you know this?
1
Not at all
2
3
4
5
Perfectly
263
Q

Mother complains her young child is a fussy eater. She eats soft foods and drinks a lot of milk. Also has been feeling tired recently. Diagnosis?

A

Iron deficiency anaemia

How well did you know this?
1
Not at all
2
3
4
5
Perfectly
264
Q

Baby is almost a month old and jaundiced. Parents say has been jaundiced since day 2. Stools are grey or white. Diagnosis?

A

Biliary atresia

How well did you know this?
1
Not at all
2
3
4
5
Perfectly
265
Q

Rashes:
Please match up the following pictures to the scenario described
Options (Photos of): Mongolian Blue Spot, Non-blanching rash with glass test (N. Meningitidis), Periorbital cellulitis, Molluscum contagiosum, Blanching rash on the trunk, Eczema on the face

● Child suffers from fever. The fever disappears but she has now developed a rash. She subsequently has febrile convulsions.

● Child appears severely unwell with a non-blanching rash…?

● Mother with cold sores has been kissing her child who has a background of eczema?

● Child has an URTI 2 weeks ago. Has now developed a rash over the back of the legs along with joint and abdominal pain.

A

● Child suffers from fever. The fever disappears but she has now developed a rash. She subsequently has febrile convulsions. - HHV6 - Roseola Infantum, blanching rash

● Child appears severely unwell with a non-blanching rash…? - Non-blanching rash with glass test (N. Meningitidis)

● Mother with cold sores has been kissing her child who has a background of eczema? Eczema on the face

● Child has an URTI 2 weeks ago. Has now developed a rash over the back of the legs along with joint and abdominal pain. HSP

How well did you know this?
1
Not at all
2
3
4
5
Perfectly
266
Q
Treatment for skin lesions
Fusidic acid, zinc and castor oil barrier cream, 1% hydrocortisone, no treatment needed
●	Nappy rash with satellite lesions 
●	Nappy rash sparing flexures 
●	Impetigo measuring 8mm 
●	Chicken pox
A

Treatment for skin lesions
Fusidic acid, zinc and castor oil barrier cream, 1% hydrocortisone, no treatment needed
● Nappy rash with satellite lesions fluconazole
● Nappy rash sparing flexures zinc and castor oil
● Impetigo measuring 8mm fusidic acid
● Chicken pox do nothing

How well did you know this?
1
Not at all
2
3
4
5
Perfectly
267
Q

ADHD:
Child comes in
Rx?

A

● Methylphenidate

How well did you know this?
1
Not at all
2
3
4
5
Perfectly
268
Q

16 y o having sex with a 12 year old - what is your next action?

A

(call the police, safeguarding…)

How well did you know this?
1
Not at all
2
3
4
5
Perfectly
269
Q

Boy always playing video games from teenage years, not interested in other people threatening to kill himself? Histrionic, Narcissistic Schizoid personality disorder

A

Schizoid

How well did you know this?
1
Not at all
2
3
4
5
Perfectly
270
Q

Kid with spiral fracture. What do you do?

A

Admit

How well did you know this?
1
Not at all
2
3
4
5
Perfectly
271
Q

Kid with petechial rashes, low RBC and raised WBC with a limp and I think and sick.

A

ALL

How well did you know this?
1
Not at all
2
3
4
5
Perfectly
272
Q

Foreign kid that is drooling and something about not being vaccinated.

A

Epiglottitis

How well did you know this?
1
Not at all
2
3
4
5
Perfectly
273
Q

Kid given dexamethasone for croup 12 hours ago by GP, was stable and well with good sats but still mild stridor. What else do you give? Repeat steroids, nebulised adrenaline, Inhaled salbutamol, oxygen

A

Repeat steroids,

How well did you know this?
1
Not at all
2
3
4
5
Perfectly
274
Q

Kid with delayed milestones in language, GP clicked his fingers and she turned to look, what’s the next step - refer for hearing assessment, refer to SALT, refer for developmental assessment

A

refer for hearing assessment

may be unilateral

How well did you know this?
1
Not at all
2
3
4
5
Perfectly
275
Q

Kid with globally delayed milestones, started to walk at like 18 months, saying 2 word phrases at 3 years, 50 word vocabulary at 3 years or something and some other stuff. What would be the most useful investigation? Detailed development history, MRI, hearing test, etc.

A

Detailed development history

How well did you know this?
1
Not at all
2
3
4
5
Perfectly
276
Q

Another kid with a strawberry tongue, what was the likely diagnosis?

A

scarlet fever or Kawasaki

How well did you know this?
1
Not at all
2
3
4
5
Perfectly
277
Q

Kid with anal itch, what do you give?

A

Mebendazole cream

How well did you know this?
1
Not at all
2
3
4
5
Perfectly
278
Q

Kid with episodes of stiffening of hands and limbs, accompanied by screaming and sweating. Kid with impaired taste stuff, then awareness and then goes to sleep for like an hour and back to normal. No memory of event. Focal seizure, atypical migraine, Absence seizure, Epilepsy, narcolepsy

A

Focal seizure

How well did you know this?
1
Not at all
2
3
4
5
Perfectly
279
Q

Another kid that would fall down and scream and stuff but was completely fine afterwards.

A

Temper tantrum

How well did you know this?
1
Not at all
2
3
4
5
Perfectly
280
Q

Kid with yellow and grey stools and was like 4 weeks old or something. What do you test for? Conjugated bilirubin levels (always do bilirubin first), G6PD, Coombs Test

A

Conjugated bilirubin levels (always do bilirubin first)

How well did you know this?
1
Not at all
2
3
4
5
Perfectly
281
Q

Kid with bouts of crying and episodes where they flex their knees and hips and red stool.

A

intussusception

How well did you know this?
1
Not at all
2
3
4
5
Perfectly
282
Q

Hypochloraemic hypokalaemic pH shown, with some clinical information. What is the initial management for it? Correct electrolyte imbalance, surgical consult, abdominal USS

A

Correct electrolyte imbalance

How well did you know this?
1
Not at all
2
3
4
5
Perfectly
283
Q

Premature kid that was born distended abdomen, vomiting, episode of blood in stool

A

NEC

How well did you know this?
1
Not at all
2
3
4
5
Perfectly
284
Q

Question on a kid who had bloods that demonstrated: low platelets, normal white cells and normal red cells.

A

ITP

How well did you know this?
1
Not at all
2
3
4
5
Perfectly
285
Q

7 year old kid headache and secondary nocturnal enuresis. He’s lost 1.5kg. Urine dipstick normal (i.e. no glucose, proteins, blood. Specific gravity ?1.010 to 1.030) What is the likely diagnosis? Urinalysis nil (plasma osmolality not given?). Diabetes insipidus, diabetes mellitus, behavioral enuresis, constipation

A

Diabetes insipidus

How well did you know this?
1
Not at all
2
3
4
5
Perfectly
286
Q

Voraciously hungry kid, hypotonia and almond eyes what was the diagnosis? Initial problems feeding, and almond eyes. Down’s, Edwards , Patau’s, Prader-Willi

A

Prader-Willi

How well did you know this?
1
Not at all
2
3
4
5
Perfectly
287
Q

7 year old kid has an accident and needs to have his leg amputated below knee. He says no and wants to wait for his mum to approve first but she’s on a business trip, dad says go for it. What do you do: Apply to Court to get amputation in best interests, Proceed with the dad’s consent (assuming dad is biological and still with the wife), wait for mum to give permission, kid is gillick competent - don’t operate.

A

Proceed with the dad’s consent (assuming dad is biological and still with the wife)

How well did you know this?
1
Not at all
2
3
4
5
Perfectly
288
Q

Kid with nocturnal enuresis where behavioural therapy and enuresis alarm hasn’t worked. He’s going to friends for a sleepover. What is next management? Desmopressin, restrict fluids

A

Desmopressin

How well did you know this?
1
Not at all
2
3
4
5
Perfectly
289
Q

Kid soiling his pants at school, something along those lines. What was the cause for it?

A

Constipation

How well did you know this?
1
Not at all
2
3
4
5
Perfectly
290
Q

3 year old kid with unilateral nasal discharge with bleeding and crust or something like that, What was the most likely cause. Foreign body insertion, nasal polyp, cancer

A

Foreign body insertion

How well did you know this?
1
Not at all
2
3
4
5
Perfectly
291
Q

Another case with an unwell child where chest was clear, had a fever. Lost weight recently. What do you do next? Glucose. CXR, urine dip, ABG and some other stuff

A

Glucose

always go with ABC…DEFG approach that’s what you do FIRST

How well did you know this?
1
Not at all
2
3
4
5
Perfectly
292
Q

Neonate with some cardio problem. Systolic murmur loudest at the left sternal edge 2/6. What was it, PDA, ASD, VSD, tetralogy of Fallot.

A

tetralogy of Fallot

How well did you know this?
1
Not at all
2
3
4
5
Perfectly
293
Q

Kid who basically had ADHD. What is the management? CBT, Parental training,
methylphenidate.

A

Parental training

How well did you know this?
1
Not at all
2
3
4
5
Perfectly
294
Q

Child who has a hx of very dry skin, rash over arms, getting worse & spreading to ?extensor surfaces. Sister has itchy rash on ankles and wrists. (Sounds like Eczema) What would be the management? Fusidic acid, emollients+1% hydrocortisone, permethrin cream

A

emollients+1% hydrocortisone

How well did you know this?
1
Not at all
2
3
4
5
Perfectly
295
Q

Hip pain on exercise and climbing stairs. Prolonged history, otherwise well. Perthes disease, osgood-schlater, septic arthritis

A

Perthes disease,

How well did you know this?
1
Not at all
2
3
4
5
Perfectly
296
Q

3 year old Kid with hypochromic microcytic anaemia and low ferritin. What could be the cause? Folate deficiency, Thalassaemia, coeliacs, fussy eater

A

fussy eater (fussy eater common in this age, they take less iron and get anaemia, which explained the hypochromic microcytic anaemia blood film)

How well did you know this?
1
Not at all
2
3
4
5
Perfectly
297
Q

Cerebral Palsy (described hemiplegic weakness with brisk reflexes), what area of the brain is affected? Motor cortex, basal ganglia, pyramidal tracts, cerebellum, internal capsule

A

Motor cortex

How well did you know this?
1
Not at all
2
3
4
5
Perfectly
298
Q

Kid having 1st set of primary vaccinations, what would stop you giving it? Fever of >38.5, rash from previous vaccine, if her brother had a reaction to it

A

Fever of >38.5

How well did you know this?
1
Not at all
2
3
4
5
Perfectly
299
Q

Kid with rough (i.e. sandpaper) rash on face & trunk, flushed face. No rash around mouth - scarlet fever, parvovirus

A

scarlet fever

How well did you know this?
1
Not at all
2
3
4
5
Perfectly
300
Q

Kid with 2cm x 2cm neck mass (inframandibular) on L side, painful, neck mass, reactive neutrophils - blood film shows: toxic left shift with reactive neutrophilia
Glandular fever, thyroglossal cyst, mump, Lymphadenitis, lymphoma, mumps

A

Lymphadenitis

How well did you know this?
1
Not at all
2
3
4
5
Perfectly
301
Q

Girl with sickle cell, has 0 reticulocytes - parvovirus, acute chest syndrome, stroke, diactylitis

A

parvovirus

How well did you know this?
1
Not at all
2
3
4
5
Perfectly
302
Q

Graph of paracetamol ‘overdose’, it had been 7 hours - decide what to give (was given a chart with a treatment line and a measure of something - it was below treatment line)? Activated charcoal, N-acetylcysteine, Gastric lavage, Active monitoring

A

All patients with a timed plasma paracetamol level on or above a single treatment line joining points of 100mg/L at 4 hours and 15mg/L at 15 hours after ingestion should receive acetylcysteine (Parvolex or generics) based on a new treatment nomogram, regardless of risk factors (see figure 1 below)

How well did you know this?
1
Not at all
2
3
4
5
Perfectly
303
Q

Newborn with purple spot on face [Sturg-Weber], what is the next best approach? Discharge to Gp follow up, Medical photography, Clotting studies, Send urgently to A&E, MRI/CT

A

MRI/CT

How well did you know this?
1
Not at all
2
3
4
5
Perfectly
304
Q

Kid needs fluids, but you can’t get standard IV access. Where do you go? Jugular, brachial, carotid, Intraosseous

A

Intraosseous

How well did you know this?
1
Not at all
2
3
4
5
Perfectly
305
Q

Kid with symptoms of nephrotic syndrome - 1st line treatment? ?Steroids, albumin solution

A

Steroids

How well did you know this?
1
Not at all
2
3
4
5
Perfectly
306
Q

HIV with undetectable viral load. what is contraindicated in labour? Forceps, ventouse, foetal blood sampling, c section and some other stuff.

A

foetal blood sampling

How well did you know this?
1
Not at all
2
3
4
5
Perfectly
307
Q

Standard chicken pox question. Doesn’t remember she had it. What do you do? Check IgG antibodies, check IgM antibodies, give her varicella Ig, give her aciclovir

A

Check IgG antibodies

How well did you know this?
1
Not at all
2
3
4
5
Perfectly
308
Q

Woman with BMI 40, abdo distension, urinary symptoms, bowel symptoms and weight loss. What could be it? Ovarian cancer, colorectal cancer, endometrial cancer etc.

A

Ovarian cancer

How well did you know this?
1
Not at all
2
3
4
5
Perfectly
309
Q

60 year old woman with PMB and superficial dyspareunia, what is the MOST LIKELY diagnosis?

A

Atrophic vaginitis

How well did you know this?
1
Not at all
2
3
4
5
Perfectly
310
Q

Woman had an implant inserted but she’s getting it removed. What is the most likely reason why? Weight gain, irregular bleeding , hirsutism, etc

A

Irregular bleeding

How well did you know this?
1
Not at all
2
3
4
5
Perfectly
311
Q

What cancer are you at increased risk at with HRT? Ovarian, endometrial, cervical, bowel, breast, etc.

A

breast

How well did you know this?
1
Not at all
2
3
4
5
Perfectly
312
Q

Cervical os is open in a young woman early pregnancy. What is it?

A

Inevitable miscarriage

How well did you know this?
1
Not at all
2
3
4
5
Perfectly
313
Q

Woman with pre-eclampsia, what drug do you give her first-line? Labetalol, Nifedipine, Methyldopa, and some others

A

Labetalol

How well did you know this?
1
Not at all
2
3
4
5
Perfectly
314
Q

Mother had rupture of membranes at like 32 weeks. What do you give her? IM dexamethasone, magnesium sulphate, something that began with c

A

IM dexamethasone

How well did you know this?
1
Not at all
2
3
4
5
Perfectly
315
Q

Intermittent pain in a young woman not pregnant? Ovarian torsion, primary dysmenorrhoea, endometriosis

A

endometriosis

1 in ten women endometriosis vs 1/100000 torsion

How well did you know this?
1
Not at all
2
3
4
5
Perfectly
316
Q

17yr old girl wanting Emergency contraception more than 5 days after unprotected sex. Copper IUD, Levonestrogel, IUS, COCP

A

Copper IUD

How well did you know this?
1
Not at all
2
3
4
5
Perfectly
317
Q

Babies head comes out but it kinda goes back in. What is the cause?

Turtle neck

A

Shoulder dystocia

318
Q

Some girl with painful periods in the first 2 days of her period ever since menarche, she has heavy bleeding. What is it? Primary dysmenorrhoea, endometriosis, endometrial cyst

A

Primary dysmenorrhoea

319
Q

Pregnant woman in early pregnancy with depression and anxiety on sertraline. What do you do? Stop sertraline, carry it on, prescribe high-dose folic acid, reduce the dose

A

Carry it on

320
Q

Post partum haemorrhage with high BP - carboprost, misoprostol, ergometrine, syntocinon

A

syntocinon

321
Q

What’s the management for DVT risk in a pregnant woman or something like that who’s coming in for a planned cesarean? LMWH, LMHW and Ted stockings, Ted stockings, Warfarin or something, Aspirin, Aspirin and Ted stockings

A

LMHW and Ted stockings

322
Q

Some pregnant woman with some itch. What tests do you do?

A

Bile acids

323
Q

Woman with blocked tubes, blocked tubes on hysterosalpingogram, what treatment should you do for fertility? IVF, IUI, ICSI.

A

IVF

324
Q

Some smear question she is 47 y/o. It was borderline HPV negative, what do you do? Repeat in 6w, repeat 6m, repeat 1y, discharge to routine (2y), discharge to routine (3y)

A

discharge to routine (3y)

325
Q

Down’s syndrome person gets pregnant. Mum is like allow that and wants termination for her and Down’s person wants to keep it. What should you do? Assess patient’s mental capacity without mum present (everyone assumed to have capacity until proven otherwise so asses it)

A

Assess capacity

326
Q

Heavy periods in a girl not sexually active. What do you give her? COCP, mefenamic acid, tranexamic acid

A

tranexamic acid

327
Q

Woman with high BMI (28) who basically had stress incontinence. What’s the first line management? Pelvic floor exercises, oxybutinin, bladder training (was this an option?, weight loss and all that

A

Pelvic floor

328
Q

What do you measure at booking for hepatitis b?

A

Hep B surface antigen

329
Q

What procedure is contra-indicated in HIV pregnant woman? Foetal blood sampling, C section, forceps, ventousse

A

Foetal blood sampling

330
Q

Man with azospermia - what is the most common cause? (think this was a repeat)
varicocele, mumps orchitis

A

varicocele

331
Q

Woman with offensive smelly lochia d2 post-partum, had had some high vaginal swabs? Reassure and discharge, await swab results, give antibiotics, abdo USS

A

give antibiotics

332
Q

Woman at term has just SROM’d, transverse lie clear liquid but fetal distress, why? Cord prolapse, vasa previa, placenta previa, placental abruption, uterine rupture

A

Cord prolapse

333
Q

Large mum (BMI >40), head comes out then goes back in, chin not visible. Rupture, short cord, shoulder dystocia, normal mechanism of labour

A

shoulder dystocia

334
Q

70yo w/ 2 pmbs, TVUS shows 6mm endometrium and 3 cm simple ovarian cyst. Next step? Ca125, diagnostic laparoscopy, pipelle,

A

Ca125

  • all PM women with >1cm cyst = require RMI determination

RMI I combines three presurgical features: serum CA-125 (CA-125); menopausal status (M); and ultrasound score (U).The RMI is a product of the ultrasound scan score, the menopausal status and the serum CA-125 level (IU/ml) as follows:
RMI = U x M x CA-125.

● The ultrasound result is scored 1 point for each of the following characteristics: multilocular cysts, solid areas, metastases, ascites and bilateral lesions. U = 0 (for an ultrasound score of 0), U = 1 (for an ultrasound score of 1), U = 3 (for an ultrasound score of 2–5).
● The menopausal status is scored as 1 = premenopausal and 3 = postmenopausal.
● Postmenopausal can be defined as women who have had no period for more than one year or women over
the age of 50 who have had a hysterectomy.
● Serum CA-125 is measured in IU/ml and can vary between zero to hundreds or even thousands of units.

335
Q

How to calculate RMI?

A

RMI I combines three presurgical features: serum CA-125 (CA-125); menopausal status (M); and ultrasound score (U).The RMI is a product of the ultrasound scan score, the menopausal status and the serum CA-125 level (IU/ml) as follows:
RMI = U x M x CA-125.

● The ultrasound result is scored 1 point for each of the following characteristics: multilocular cysts, solid areas, metastases, ascites and bilateral lesions. U = 0 (for an ultrasound score of 0), U = 1 (for an ultrasound score of 1), U = 3 (for an ultrasound score of 2–5).
● The menopausal status is scored as 1 = premenopausal and 3 = postmenopausal.
● Postmenopausal can be defined as women who have had no period for more than one year or women over
the age of 50 who have had a hysterectomy.
● Serum CA-125 is measured in IU/ml and can vary between zero to hundreds or even thousands of units.

336
Q

The very last question was a bunch of tests for a woman who couldn’t conceive- all test normal and prolactin raised by like 2 points above normal value what’s most likely cause?

A

Unknown

337
Q

Drug to give for depression in a guy who had an MI.

A

Sertraline

338
Q

Old age boxer rips out cannula, attacks staff what do you do? Calm him with verbal de-escalation, make him some tea LOL, debrief the nurses, start iv Abx

A

verbal de-escalation

339
Q

Guy on haloperidol gets muscle problems

A

give IM procyclidine

340
Q

Woman experienced trauma 6 months ago/has PTSD best management?- trauma focused cbt wasn’t option, EMDR , (cognitive analytical therapy i think was on there)

A

EMDR

341
Q

Guy works as an IT engineer, doesn’t like sex, or people

A

schizoid

342
Q

Guy worries about running kids over so has to check the road or something everytime

A

OCD

343
Q

Woman basically worries about blushing in public (buzzword)-

A

social phobia

344
Q

Guy has a fear of public speaking, worried about embarrassing himself, hasn’t seen his personal tutor, hasn’t left the house in a few? weeks, gets sick thinking about it. social phobia, agoraphobia, avoidant personality disorder (or something like that)

A

Social

345
Q

Some schizo making a scene in public and police want to detain him, which MHA?

A

136

346
Q

Alcoholic wants to quit and wants something for reducing cravings (acamprosate, naltrexone, disulfiram)

A

Naltrexone safe if they haven’t already stopped

347
Q

Guy on antipsychotic comes in with tremor- most likely drug

A

haloperidol

348
Q

Child has classic symptoms of ADHD, how do you manage? Methlyephenidate, Family Therapy or CBT

A

Family Therapy

349
Q

Guy writes letters to the PM, think he’s being spied on by the government for the past 10 years, shows no other symptoms

A

Delusional disorder,

350
Q

Man with history of paranoid schizophrenia, his Ix and tests are strongly suggestive of an MI, needs treatment. He has capacity and refuses treatment - what should you do? Treat him under DoLs, treat under MCA, treat under MHA, respect his wishes

A

respect his wishes

351
Q

Lithium - what do you monitor?

A

Lithium, TFTs, U&E

352
Q

Patient on ward becomes aggressive, verbal de-escalation hasn’t worked. Tx? Oral haloperidol, oral lorazepam, IM lorazepam, IM haloperidol, IM haloperidol & lorazepam

A

IM lorazepam

353
Q

50y lady fell over gardening, comes in with paralysis of leg, no medical cause found? Conversion disorder

A

Conversion disorder

354
Q
  • 70 year old lady has some spotting and says she uses some steroid cream for a “rash down there” – lichen sclerosis –
A

steroid treatment

 Lichen sclerosis is the commonest cause of vulval itching. There is severe itchiness, especially at night. Inflammatory adhesions can form, with potential labial fusion and stenosis of introitus.
 Leucoplakia (white plaques) and atrophic changes
 Treatment = ultra-potent topical steroids

355
Q
  • Lady is 47 y/o. It was borderline dyskaryosis and HPV negative, what do you do? Repeat in 6w, repeat 6m, repeat 1y, discharge to routine (2y), discharge to routine (3y)
A

discharge to routine (3y)

356
Q
  • Gardasil – what does it protect against. Name the one from the list: HIV, HSV1, HSV2, HPV11, HPV29?
A

HPV11

HPV 16/18 are implicated mostly in cervical cancer

357
Q
  • Woman = BMI 40, abdominal distension, urinary symptoms, bowel symptoms and weight loss. What could be it? Ovarian cancer, colorectal cancer, endometrial cancer etc.
A

Ovarian cancer

358
Q
  • Asymptomatic woman, nulliparous, found to have a 5.4cm unilocular ovarian cyst on US, no FHx. What is the next appropriate step? USS guided cyst aspiration, laparoscopic removal of cyst, discharge and safety-net, rearrange USS in 3 months and Ca125 follow-up
A

rearrange USS in 3 months and Ca125 follow-up

359
Q
  • 60-year-old woman with PMB and superficial dysuria and dyspareunia, what is the most likely diagnosis?
A

Atrophic vaginitis
 This is vaginal irritation caused by thinning of the epithelium. It is due to reduction of circulating oestrogen levels. Caused by menopause and prolonged lactation.
 O/E = pale, thin vaginal walls with cracks and fissures. Loss of rugal folds.
 Swabs can be done for any superimposed infection risk.
 Manage with topical oestrogens or systemic HRT

360
Q
  • What is the commonest type of endometrial cancer?
A

ADENOCARCINOMA
Other types are stromal sarcoma, myometrial sarcoma (a.k.a. leimyosarcoma), atypical myometrial tumours (e.g. leimyoblastoma, clear cell leiomyoma).

361
Q
  • 72-year-old woman with PMB and endometrial thickness seen on USS, what’s the diagnosis?
A

Endometrial hyperplasia/cancer

Endometrial hyperplasia is the precursor to cancer. It can be simple or complex (atypical). Treat this with progesterone for its regression, and redo the biopsy 3 months later. This can be repeated up to one year. If it is still abnormal for 1 year, perform a hysterectomy.
The surgery done is a total abdominal hysterectomy and bilateral salpingo-oopherectomy.

362
Q
  • Post-menopausal woman with a PV bleed. What ix should you do?
    Laparotomy and hysteroscopy, outpatient USS with endometrial biopsy, CT, MRI
A

outpatient USS with endometrial biopsy

363
Q
  • 70-year-old women with 2 PMBs, TVUSS shows 6mm endometrium and 3cm simple ovarian cyst. What is the next appropriate step? Ca125, diagnostic laparscopy, pipelle
A

Ca125,

  • * THIS IS THE NEXT STEP – NOT INVASIVE YET. FIRST WE HAVE TO CHECK CA125 LEVELS!!!!!!***
364
Q
  • A 47 year old woman with a 2 cm simple ovarian cyst seen on USS, as well as 12mm endometrium. What do you do next?
    (a) Repeat USS
    (b) Ca 125 levels
    (c) Hysteroscopy
    (d) Pipelle biopsy
    (e) Reassure & discharge
A

Pipelle biopsy

365
Q
  • Woman BP 120/80, smokes 5/day, what contraception for her?
A

COCP – no absolute contraindications.

366
Q
  • Woman on COCP, has missed 6th and 7th day pill, had unprotected sex two days ago, her urine pregnancy is clear. What to do? Repeat in a week, reassure and discharge, emergency contraception
A

emergency contraception

367
Q
  • Woman had a Nexplanon implant inserted but she’s getting it removed. What is the most likely reason why? Weight gain, irregular bleeding, hirsutism [REPEAT]
A

irregular bleeding

368
Q
  • What type of contraception can increase risk of osteoporosis?
A

Depot injections

369
Q
  • 32 year old has completed family. Had COCP before, what contraception now?
A

Mirena coil

370
Q
  • What cancer are you at increased risk at with HRT? Ovarian, endometrial, cervical, bowel, breast, etc.
A

breast

 Hormone replacement therapy (HRT) includes the use of a small dose of oestrogen (with progesterone in women with a uterus) to help alleviate menopausal symptoms
 Side effects: nausea, breast tenderness, fluid retention, weight gain
 Complications: increased risk of breast cancer (higher risk when combined preparations of oestrogen AND progesterone), increased risk of endometrial cancer (LOWER risk when combined with progesterone), VTE, stroke, ischaemic heart disease (if taken >10 years after the menopause)

371
Q
  • HRT question for a menopausal woman with flushes and stuff, last period ?10m ago (less than a year). What do you give her? Cyclical combined HRT, oestrogen only HRT, topical HRT, continuous HRT -
A

Cyclical combined HRT

They called cyclical, sequential in the exam

372
Q
  • Woman who swims wants to treat hot flushes, doesn’t want a patch and has irregular menstruation.
A

Cyclical oral combined HRT or COCP depending on if she has bled in past 12 months and wants to have periods/get pregnant

373
Q
  • Woman with premature ovarian failure, wants to have periods.
A

COCP

374
Q
  • Woman who just wants to treat osteoporosis.
A

Bisphosphonates

Treatment for osteoporosis: bisphosphonates, strontium ranelate, SERMS e.g. raloxifene

Or denosumab ;)

375
Q
  • 40 year old premature ovarian failure and wanted medication to deal with the symptoms of menopause. What would you prescribe her?
A

Cyclical combined HRT (oestrogen and progesterone) as she still has a uterus to decrease risk of endometrial cancer

376
Q

Hormonal profiles: GH, LH, FSH, prolactin, free androgens

  • Anorexia =
A

hypothalamic hypogonadism. High GH, low LH, low FSH, normal prolactin

377
Q

Hormonal profiles: LH, FSH, prolactin,

  • Prolactinoma =
A

high prolactin (causes low GnRH release) therefore low LH and FSH. Treat with dopamine agonists e.g. bromocriptine to restore ovulation as DA inhibits prolactin release.

378
Q

Hormonal profiles: LH, FSH,

  • Menopause =
A

high LH, high FSH, low anti-mullerian hormone, low oestradiol and low inhibin

379
Q

Hormonal profiles: GH, LH, FSH, prolactin, free androgens

  • PCOS =
A

raised LH:FSH ratio (>2.5), high free androgens

380
Q

presents as sharp unilateral pain immediately following intercourse or strenuous exercise. Bimanual examination in non-severe cases is generally unremarkable but the lower abdomen is tender. Ultrasound shows free fluid in the pelvic cavity.

A

Ruptured ovarian cyst

381
Q

can present with sharp unilateral pain with nausea and vomiting. There is a tender palpable adnexal mass on bimanual exam. USS: enlarged, oedematous ovary with impaired blood flow, whirlpool sign. Doppler scan may show little or no ovarian venous flow. USS whirlpool sign can also be seen when bowel twists on itself causing a volvulus.

A

Ovarian or adnexal torsion

382
Q

mid-cycle pain. Usually goes away over 24-48 hours.

A

Mittelschmerz

383
Q

Hx: A 28-year-old woman complains of a two year history of bad period pains which are not controlled by NSAIDs or the combined contraceptive pill. She also reports significant pains during intercourse

A

USS can show free-fluid. Laparoscopy will show endometriosis lesions.

Endometriosis

384
Q

bilateral lower abdo pain associated with vaginal discharge. Dysuria may also be present. Peri-hepatic inflammation secondary to chlamydia = Fitz Hugh Curtis syndrome. Fever is present and amylase can also be slightly raised. Treat the cause. Hysterosalpingography may show bilaterally blocked fallopian tubes.

A

Pelvic inflammatory disease –

385
Q

Cervical excitation on examination =

A

PID or ectopic pregnancy

386
Q
  • Woman with really heavy periods and doesn’t want to get pregnant. What’s first line management?
    (a) COCP
    (b) Implant
    (c) IUS (if no desire for kids, give her the IUS, if wants kids then tranexamic acid)
    (d) IUD
    (e) Tranexamic Acid
    (f) Mefenamic Acid
A

(c) IUS (if no desire for kids, give her the IUS, if wants kids then tranexamic acid)

387
Q
  • Heavy periods in a girl not sexually active. What do you give her?
A

Tranexamic acid

388
Q
  • Some girl with painful periods in the first 2 days of her period ever since menarche, she has heavy bleeding. What is it? Primary dysmenorrhoea, endometriosis, endometrial cyst
A

Primary dysmenorrhoea,

389
Q
  • Woman with PCOS. Best medication to increase fertility.
A

Clomiphene

390
Q
  • Woman presents with cystic ovaries on USS and irregular periods. What other symptom would be the best indicator of her having polycystic ovaries? Dysmenorrhoea, Hirsutism, Obesity, Acne.
A

Hirsutism

391
Q
  • Subfertile woman, all tests normal and prolactin raised by 2 points above normal value. What’s most likely cause?
A
  • Subfertile woman, all tests normal and prolactin raised by 2 points above normal value. What’s most likely cause? Unknown (accounts for like 30% of subfertility), prolactinoma (prolactin would be in 1000s), raised testosterone suggests PCOS and LH:FSH ratio>2:1
392
Q
  • Woman with blocked tubes, blocked tubes on hysterosalpingogram, what treatment should you do for fertility? IVF, IUI, ICSI.
A

IVF

393
Q
  • Amenorrhoea for 4 months - what test do you do?
A

B-HCG / preg test

394
Q
  • 19-year-old abdominal swelling, weight gain, irregular periods, can’t remember LMP and denies being sexually active. What is the first test you would do?
A

Pregnancy test,

395
Q
  • Woman with cyclic pain 1 week before her period starts, trying for a baby for one year. What is the likely diagnosis?
A

endometriosis

396
Q
  • Woman with signs of premature ovarian failure. What test would be best to confirm this diagnosis: Oestradiol, Testosterone, FSH
A

FSH

397
Q
  • Male presents with azoospermia - what is the most common cause? [REPEAT 2016+2018]
    (a) Varicocele
    (b) History of mumps orchitis
    (c) History of testicular torsion
A

(a) Varicocele (accounts for 25% of male subfertility)

398
Q
  • Results show azoospermia. How should they be managed? IVF, egg donation, ICSI, IUI
A

ICSI

399
Q
  • Woman with BMI 28 (i.e. overweight) leaks urine when laughing and going up the stars. What’s the first line management?
A
Stress incontinence  pelvic floor exercises. 2nd line is surgery – retropubic mid-urethral tape
Urge incontinence (a.k.a. overactive bladder)  bladder training (at least 6 weeks). 2nd line = anti-muscarinics e.g. oxybutynin, tolterodine. Oxybutynin should be avoided in old people.
Overflow incontinence – due to bladder outlet obstruction e.g. prostate enlargement
400
Q
  • 47-year-old woman with menorrhagia, dysmenorrhoea and USS with multiple fibroids. Doesn’t want any more children. What is good treatment?
A

Hysterectomy

401
Q
  • 14 year old girl wants TOP, can’t/will not inform parents. What do you do?
A

Persuade her to tell her parents if she refuses and gillick competent, you can give her TOP

402
Q
  • 14 year old with primary amenorrhoea and ejection systolic murmur. What is Dx?
A

Turner’s syndrome

403
Q

EMQ

(a) Mastitis
(b) intraductal papilloma
(c) malignancy
(d) abscess
(e) genital warts
(f) Bartholins cyst

  • Lady high WCC, fever, lump in breast –
  • General breast tenderness, left sided, breastfeeding –
  • Small slow growing lesion on labia, pregnant lady –
    Small painful lump inside vagina of sexually active woman –
    What is the treatment for Bartholin’s cyst –
A
  • Lady high WCC, fever, lump in breast – abscess
  • General breast tenderness, left sided, breastfeeding – mastitis
  • Small slow growing lesion on labia, pregnant lady – Genital Warts (pregnancy causes them to grow)
    Small painful lump inside vagina of sexually active woman – Bartholin’s Cyst
    What is the treatment for Bartholin’s cyst – marsupialisation
404
Q

Pessary for prolapse?

A

Urogynae – types of pessary (ring, shelf, hodgson). Uterine prolapse = ring pessary 

405
Q
  • Woman with a slow growing painless lesion on labia -
A

Genital warts, also known as condylomata accuminata are common. Caused by many varieties of HPV (especially 6 and 11). Small, fleshy protuberances which are slightly pigmented. May bleed or itch. Treatment = topical podophyllum or cryotherapy are first-line. Multiple, non-keratinised warts respond better with topical agents; solitary, keratinised warts respond better to cryotherapy. Imiquimod is a topical cream used second line. These grow during pregnancy.

406
Q
  • Woman with tender lump inside her vagina –
A

Bartolin’s cyst

With infection = abscess. Rx = antibiotics, or marsupialisation (insertion of a word catheter)

407
Q
  • Old woman complaining of superficial dyspareunia –
A

atrophic vaginitis

408
Q
  • Painful multiple lesions/genital ulcerations on labia – herpes  Rx
A

= oral acyclovir

409
Q
  • Blue dots on cervix –
A

Nathobian cyst

410
Q
  • Woman with cheesy, curd-like white discharge –
A

candidiasis

411
Q
  • What is the appropriate treatment for candida?
A

Oral fluconazole, topical clotrimazole

DUO

412
Q
  • Young woman, dyspareunia, ‘strawberry cervix’
A

= Trichomonas vaginalis

413
Q
  • Yellow/green (frothy) discharge

-

A

= Trichomonas vaginalis

  • Pinpoint areas of exudation may also be present. Treated with oral metronidazole
414
Q
  • Clue cells =
A

bacterial vaginosis

  • Rx = oral metronidazole for 5-7 days
    Caused by Gardnerella vaginalis. Raised vaginal pH >4.5 and a positive whiff test. Relapse rate is >50% within 3 months. Alternatives to oral metronidazole are topical metronidazole or topical clindamycin.
415
Q

Gram-negative diplococcus bacterial infection with vaginal discharge –>

A

Neisseria gonorrhoea. Rx: single dose of IM ceftriaxone 500mg and oral azithromycin 1g. These act synergistically and is useful for also killing co-existent Chlamydia [usually treated with azithromycin or doxycycline].

416
Q
  • Cyst contains hair and teeth –
A

mature cystic teratoma (a.k.a. dermoid cyst)

417
Q
  • Ground glass appearance –
A

endometrioma

418
Q
  • Woman has pain before periods and has been sub-fertile
A

– chocolate cyst on ovary

419
Q
  • What procedure is contra-indicated in HIV pregnant woman?
    (a) Foetal blood sampling
    (b) C-section
    (c) Forceps
    (d) ventouse
A

(a) Foetal blood sampling

420
Q
  • Woman is pregnant and HIV -ve at booking but her partner is HIV +ve. What do you do?
A

Nothing

421
Q
  • Standard chicken pox question. Doesn’t remember she had it. What do you do first?
    (a) Check IgG antibodies
    (b) Check IgM antibodies
    (c) Give her varicella Ig
    (d) Give her acyclovir
A

(a) Check IgG antibodies - first line. IgG = memory response mounted versus VZV.

422
Q
  • Pregnant woman with itchiness, what test do you do? Serum bile acids, uric acids, FBCs, bilirubin, transaminases, or liver function tests?
A

Serum bile acids

423
Q
  • UTI in first trimester of pregnancy. What would be the safest and most effective treatment? Trimethoprim, Ciprofloxacin, Doxycycline, Cefalexin, Meropenem
A

Cefalexin

 Trimethoprim is a folate antagonist and it is therefore teratogenic in the first trimester of pregnancy. Its use is also avoided throughout pregnancy
 In pregnant women, nitrofurantoin can be used instead for a UTI. But nitrofurantoin should be avoided in women at full-term due to risk of neonatal haemolysis.
 Tetracyclines (doxycycline), aminoglycosides, sulphonamides, trimethoprim, meropenem and quinolones (ciprofloxacin) are contraindicated in pregnancy.

424
Q
  • Pregnant woman with gram negative bacilli in urine microscopy. Number was a bit low and on repeat was still low. What do you do? Watch and wait, give amoxicillin, urgent referral, some scans
A

give amoxicillin

Asymptomatic bacteriuria in pregnancy  immediate Abx: nitrofurantoin (avoid at term), or amoxicillin (3 times a day for 7 days), or cefalexin.

425
Q
  • What is the leading cause of mortality (maternal) in the UK?
A

Cardiovascular disease

426
Q
  • Down’s syndrome person gets pregnant. Mum is like allow that and wants termination for her and Down’s person wants to keep it. What should you do?
A

Assess patient’s mental capacity without mum present (everyone assumed to have capacity until proven otherwise so assess it)

427
Q
  • Woman 38/40 has come in for her elective c/s, ECV failed, what do you do now?
A

(b) Carry on with the C-Section as planned

428
Q
  • Woman previously had an abortion due to severe spina bifida. No other clinical information given. What do you advise her on folic acid doses for her next pregnancy?
A

5mg of folic acid before conception until week 12 pregnancy

This dose should be taken by all women at higher risk of conceiving a child with NTD - criteria:
 Either partner has NTD
 Previous pregnancy affected by NTD
 Family history of NTD
 Woman is taking antiepileptics, or has coeliac disease, diabetes, or thalassaemia trait
 Woman is obese i.e. BMI >30
All other healthy women should take 400mcg of folic acid until week 12 pregnancy.

429
Q
  • Large mum (BMI >40), head comes out then goes back in, chin not visible. What is the cause? Uterine rupture, short cord, shoulder dystocia, normal mechanism of labour.
A

shoulder dystocia

Turtle head (not a poo)

430
Q
  • Poorly controlled diabetic mother, newborn has an abnormal asymmetric Moro reflex - what’s wrong?
    (a) Hypoxic Ischaemic Encephalopathy
    (b) Fracture of humerus
    (c) Brachial plexus injury - secondary to probably shoulder dystocia because of diabetes
A

(c) Brachial plexus injury - secondary to probably shoulder dystocia because of diabetes

431
Q
  • What’s the management for DVT risk in a pregnant woman or something like that who’s coming in for a planned cesarean? LMWH, LMWH and Ted stockings, Warfarin, Aspirin, Aspirin and Ted stockings
A

LMWH and Ted stockings

432
Q
  • What do you do in a woman in labour after 4 hours of checking, she is 4cm dilated? Prostaglandins, Cervical sweep, Oxytocin, Ergometrine, ARM
A

Cervical sweep

433
Q
  • What is a screening tool for Down’s syndrome risk?
A

Nuchal translucency

434
Q
  • Foreign woman has come in and is pregnant. What vaccine should she be offered? Hepatitis A, hepatitis B, whooping cough
A

hepatitis B

435
Q
  • What do you measure at booking for Hepatitis B?
A

Hep B surface antigen.
o If HepBsAg is positive, all babies should receive Hepatits B vaccination.
o Then HepBeAg should be checked too – if this is positive, also give baby Hepatitis B immunoglobulin
o Antibody response to the vaccination course should be checked at 12 months in high-risk infants, as 5% need further vaccination.

436
Q
  • Woman at term has just undergone spontaneously rupture of membranes, transverse lie clear liquid but fetal distress, why? Cord prolapse, vasa previa, placenta previa, placental abruption, uterine rupture
A

Cord prolapse

437
Q
  • Pregnant lady being domestically abused by husband and scared to go home. What do you do in GP? Send her home and ask her to come back with husband, give her a leaflet about domestic abuse, call and arrange emergency accommodation
A

call and arrange emergency accommodation

438
Q
  • Pregnant woman in early pregnancy with depression and anxiety on sertraline. What do you do? Stop sertraline, continue sertraline use, prescribe high-dose folic acid, reduce the dose, amitriptyline
A

continue sertraline use

439
Q
  • Effect of taking fluoxetine (paroxetine?) during pregnancy on baby?
A

Persistent pulmonary hypertension of the newborn

440
Q
  • Cervical os is open in a young woman early pregnancy and there is increasing amounts of bleeding, and cramping pain. What is it?
A

Inevitable miscarriage

Inevitable – bleeding is worsening, os is open. Cramping pain is present
Incomplete – bleeding may have finished. Os is open. No cramping pain
Threatened – os is closed

441
Q
  • Woman with mild pre-eclampsia, what drug do you give her first-line? Labetalol, Nifedipine, Methyldopa
A

Labetalol

if it was higher severity, and needed immediate relief then can give nifedipine

442
Q
  • At antenatal check, woman with BP 150/90, what would you do?
A

Admit and assess

443
Q
  • Mother had rupture of membranes at like 32 weeks. What do you give her?
A

IM dexamethasone

444
Q
  • Which describes a 20 or 23 week antenatal USS the best? [REPEAT]
    (a) Tests for nuchal thickness for Down’s
    (b) Tests for congenital heart disease
    (c) Predicts position of placenta at term
    (d) Least amount of radiation to foetus
    (e) 2D with foetal biometry
    (f) Looks for multiple pregnancies
A

(b) Tests for congenital heart disease

445
Q
  • What causes increased urinary volume and frequency in the first trimester? Increased GFR, pressure of uterus on bladder, glycosuria.
A

Increased GFR

446
Q
  • Woman with rupture of membranes, painless bleeding and something along those lines. What was the likely diagnosis? placental praevia, Vasa praevia, Placental abruption, chorioamnionitis
A

placental praevia

447
Q
  • Pregnant women with pain and bleeding, abdomen feels hard -
A

Placental abruption

448
Q
  • A pregnant Turkish lady has low haemoglobin, low MCV, normal MCHC. What is the appropriate next management?
    (a) Blood transfusion
    (b) Iron transfusion
    (c) Oral iron tablets – consider if at 16 weeks and Hb <11; or at 28 weeks and Hb <10.5
    (d) Do bloods again in 2 weeks
A

(c) Oral iron tablets – consider if at 16 weeks and Hb <11; or at 28 weeks and Hb <10.5

449
Q

When would you admit a patient with excessive vomiting?

A

Ketonuria

450
Q

Cause of macrosomnia =

A

gestational diabetes

451
Q

Women with previous GDM, what is the best way to investigate her blood glucose?

A

OGTT at 28 weeks

452
Q

Woman with APH (spotting), otherwise well. What is the most important thing to rule out?

A

Ectopic pregnancy

453
Q
  • Mother has a stillborn baby, generalised oedematous when it was born. Mother had a fever at 18 weeks with rash on trunk –
A

parvovirus

454
Q
  • Mother was being prepared for C-section, and got a sudden tingling around her mouth? What is the cause?
A

Spinal block

455
Q
  • Mother was being prepared for C-section, has a regional block. Sudden tachycardia, SOB, difficulty breathing. What is the diagnosis?
A

Anaphylaxis

456
Q
  • Mother had a previous C-section. Now presents with sudden abdominal pain and abnormal CTG during labour. What is the diagnosis?
A

Uterine rupture

457
Q
  • Post partum haemorrhage with high BP. What is given to manage this?
    (a) Carboprost
    (b) Misoprostol
    (c) Ergometrine
    (d) Syntocinon
A

(d) Syntocinon

458
Q
  • Woman 8 weeks after normal vaginal delivery and second degree tear, still bleeding and mild lower pelvic pain. Diagnosis? Normal menstruation, PID, endometritis, lochia
A

endometritis

459
Q
  • Woman with offensive smelly lochia 2 days post-partum, had had some high vaginal swabs? Reassure and discharge, await swab results, give antibiotics, abdominal USS
A

give antibiotics

460
Q
  • What is not a risk factor for primary PPH?

a. B thalassaemia trait
b. Retained products
c. Sepsis
d. Vaginal tear
e. Multiparity

A

a. B thalassaemia trait

461
Q

Med Ed translations:

Pale

A

Anaemic/shocked

462
Q

Med Ed translations:

Pale and jaundiced

A

Haemolytic anaemia

463
Q

Med Ed translations:

Drooling

A

Epiglottitis

464
Q

Med Ed translations:

Recently moved from abroad and don’t have a red book

A

Unimmunised

Measles/epiglot/meningitis

465
Q

Med Ed translations:

“mum has a new boyfriend”

A

Child abuse

466
Q

Med Ed translations:

‘Pins and needles’ in hands

A

Panic attack

467
Q

Med Ed translations:

6 days of fever

A

Kawasaki’s Disease

468
Q

Med Ed translations:

Mum unable to let go of hand

A

Myotonic dystrophy

469
Q

Med Ed translations:

Rash with amoxicillin

A

EBV

470
Q

Med Ed translations:

Irritability

A

Meningitis

471
Q

Med Ed translations:

Fat teenager

A

SUFE

472
Q

Med Ed translations:

Gower’s sign positive

A

Duchenne’s

473
Q

Med Ed translations:

Cap refil >3

A

Shocked

474
Q

Med Ed translations:

Recently moved from abroad and don’t have red book

A

No guthrie screening. Think thyroid/PKU/cystic fibrosis

475
Q

Med Ed translations:

Heart appears boot shaped on CXR

A

Tetralogy

476
Q

Med Ed translations:

Port wine stain

A

Sturge-Weber

477
Q

Med Ed translations:

Confusion

A

Encephalitis/opathy

478
Q

Med Ed translations:

Barking cough

A

Croup

479
Q

Med Ed translations:

Birthday party

A

Anaphylaxis

480
Q

A healthy 10 month old boy has a 3 day history of coryza and fever up to 38 degrees.
He has a barking cough and over the last few hours has developed inspiratory stridor. He has mild-moderate respiratory distress, and oxygen saturations of 99% on air. He is not drooling. His temperature in the department is 38.2 degrees.

A – Oral dexamethasone and observe in the day unit until respiratory distress settles.
B – Obtain IV access and take a blood culture, commence IV ceftriaxone
C – Adrenaline nebuliser and discharge from A+E if respiratory distress has resolved.
D – Give facial oxygen and a salbutamol nebuliser, and review after 20 minutes
E – Discharge home with 10 puffs 4 hrly inhaled salbutamol and advice to return if increasing respiratory distress

A

A – Oral dexamethasone and observe in the day unit until respiratory distress settles

481
Q

Lucy is 3 years old and presents with cough and stridor. She appears unwell and is drooling. She has subcostal, substernal recessions and tracheal tug.

What is the most appropriate initial management?

A) Cannulate the child as soon as possible and give IV Ceftriaxone
B) Do not approach the child and call for immediate anaesthetic and ENT support
C) Examine the child’s throat to confirm the diagnosis
D) Give an adrenaline nebuliser and prepare to cannulate
E) Give oral dexamethasone and facial O2

A

B) Do not approach the child and call for immediate anaesthetic and ENT support

482
Q

Lucy is 3 years old and presents with stridor and a spreading rash after returning from a playdate at a friend’s house.

She is given IM adrenaline in the ambulance and presents to A&E 10 minutes later.

She has loud stridor, swollen lips and respiratory distress. Her SATS are 92% on air and she is wheezy.

What is the most appropriate initial management?

A) 15L O2 via a non-rebreathe bag
B) IM adrenaline
C) Nebulised adrenaline
D) Nebulised salbutamol
E) Obtain IV access and give IV Hydrocortisone
A

B) IM adrenaline

483
Q

Caoimhe is 4 weeks old. She is brought to A&E with noisy inspiratory breathing which has been more prominent over the last few weeks.
She appears well on examination but has some inspiratory stridor, more obvious when she cries.
She is gaining weight and is afebrile

What is the most likely diagnosis?

A) Croup
B) Epiglottitis
C) Foreign body aspiration
D) Laryngomalacia
E) Subglottic stenosis
A

D) Laryngomalacia

484
Q

Sunayna is 1 year old and presents with abdominal pain and vomiting. You have been asked to write up full maintenance fluids for her. She weighs 12kg

Which fluid regime would be most appropriate?

A) 500ml bag of 0.9% NaCl followed by 500ml bag of 10% dextrose over 24 hours

B) 1100ml of NaCl 0.9% over 24 hours

C) 1100ml of NaCl 0.9% plus 5% dextrose over 24 hours

D) 1200ml of NaCl 0.9% over 24 hours

E) 1200ml of NaCl 0.9% plus 5% dextrose over 24 hours

A

C) 1100ml of NaCl 0.9% plus 5% dextrose over 24 hours

3 options
Maintenance fluids

0.9% NaCl + 5% Dextrose
100mls/kg/day for each of first 10kg (ie 0-10kg)
50ml/kg/day for each of next 10kg (ie 10-20kg)
20ml/kg/day for every further kg
Eg 24kg = (100 x 10) + (50 x 10) + (20 x 4)
= 1000 + 500 + 80 = 1580 over 24 h

Bolus fluids
20ml/kg 0.9% NaCl in most situations
10ml/kg when……
DKA, Trauma, Fluid overload or heart failure

Dehydration corrections:
Usually over 24 hours
Maintenance fluids PLUS %dehydration

Most of your body is water -> estimate the % lost

Weigh the child if possible.
1kg weight loss = 1000ml lost

Estimate clinically if not possible
3% weight loss in 20kg child.
20kg = 20000g = 20000mls fluid
1% = 200ml, therefore 3% = 600ml

ADD the correction onto maintenance fluids over 24h

485
Q

Maria is 4 years old and weighs 17kg. She presents with a non-blanching rash and a fever of 39 degrees

Her heart rate is 180 and her blood pressure is 64/40

What is the most appropriate initial fluid management?

A) 1350ml 0.9% NaCl and 5% dextrose over 24 hours

B) 170ml 0.9% NaCl bolus

C) 340ml 0.9% NaCl bolus

D) 340ml 0.9% NaCl and 5% dextrose bolus

E) 340ml ringer’s lactate bolus

A

C) 340ml 0.9% NaCl bolus

Maintenance fluids

0.9% NaCl + 5% Dextrose
100mls/kg/day for each of first 10kg (ie 0-10kg)
50ml/kg/day for each of next 10kg (ie 10-20kg)
20ml/kg/day for every further kg
Eg 24kg = (100 x 10) + (50 x 10) + (20 x 4)
= 1000 + 500 + 80 = 1580 over 24 h

Bolus fluids
20ml/kg 0.9% NaCl in most situations
10ml/kg when……
DKA, Trauma, Fluid overload or heart failure

Dehydration corrections:
Usually over 24 hours
Maintenance fluids PLUS %dehydration

Most of your body is water -> estimate the % lost

Weigh the child if possible.
1kg weight loss = 1000ml lost

Estimate clinically if not possible
3% weight loss in 20kg child.
20kg = 20000g = 20000mls fluid
1% = 200ml, therefore 3% = 600ml

ADD the correction onto maintenance fluids over 24h

486
Q

Marco presents following 2 days of profuse diarrhoea and vomiting. He has dry lips and is tachycardic, but is not shocked. He is not tolerating any oral intake. You estimate that he is 3% dehydrated. He weighs 10kg.

What would be the most appropriate fluid regimen to correct his dehydration?

A) 200ml 0.9% NaCl bolus, repeated if necessary

B) 200ml 5% dextrose bolus, repeated if necessary

C) 1000ml 0.9% NaCl + 5% dextrose over 24h

D) 1030ml 0.9% NaCl + 5% dextrose over 24 h

E) 1300ml 0.9% NaCl + 5% dextrose over 24 h

A

E) 1300ml 0.9% NaCl + 5% dextrose over 24 h

3 options
Maintenance fluids

0.9% NaCl + 5% Dextrose
100mls/kg/day for each of first 10kg (ie 0-10kg)
50ml/kg/day for each of next 10kg (ie 10-20kg)
20ml/kg/day for every further kg
Eg 24kg = (100 x 10) + (50 x 10) + (20 x 4)
= 1000 + 500 + 80 = 1580 over 24 h

Bolus fluids
20ml/kg 0.9% NaCl in most situations
10ml/kg when……
DKA, Trauma, Fluid overload or heart failure

Dehydration corrections:
Usually over 24 hours
Maintenance fluids PLUS %dehydration

Most of your body is water -> estimate the % lost

Weigh the child if possible.
1kg weight loss = 1000ml lost

Estimate clinically if not possible
3% weight loss in 20kg child.
20kg = 20000g = 20000mls fluid
1% = 200ml, therefore 3% = 600ml

ADD the correction onto maintenance fluids over 24h

487
Q

Abdul is 6 years old and presents with 24 hours of cough and difficulty in breathing. He is found to be wheezy and is given 3x nebulised salbutamol, 2x ipratropium bromide and oral prednisolone.

30 minutes later he is still wheezy and shows signs of respiratory distress. He appears to be shivering and has vomited twice.

What is the most appropriate next management step?

A) Magnesium Sulfate bolus IV

B) Montelukast orally

C) Salbutamol bolus IV

D) Salmeterol inhaled (long acting B agonist)

E) Salbutamol nebuliser

A

A) Magnesium Sulfate bolus IV

MED ED RECKONS IV STEPS:

1) MgSO4
2) Salbutamol
3) Aminophylline

I’m not so convinced…most textbooks say salbutamol first

488
Q

George has brittle asthma. He presents with a severe exacerbation.

He is currently on an aminophylline and salbutamol infusion.

His oxygen saturations are 92% on 15L O2 and he appears tired.

What is the most appropriate next management step for George?

A) Aminophylline level and adjust dose accordingly

B) Atrovent nebuliser

C) Hydrocortisone IV

D) Intubation and transfer to PICU

E) Magnesium Sulfate IV

A

D) Intubation and transfer to PICU

If tiring do safest option

489
Q

Josh is 3 years old and presents with a 24 hour history of cough and difficulty in breathing.

He received inhaled salbutamol and atrovent on presentation.

You review him one hour later. He appears comfortable with no wheeze and good air entry. What would be the most appropriate next step in management?

A) Admit to the ward

B) Magnesium sulfate IV

C) Montelukast orally

D) Prednisolone orally and reassess in 1 hour

E) Salbutamol via spacer 1 hourly

A

D) Prednisolone orally and reassess in 1 hour

490
Q

Katie is 3 years old. She has been admitted to hospital 4 times with wheezing and has required IV medications once. She has eczema and was started on very low dose inhaled corticosteroids after her 2nd admission

She presents to follow up clinic and her mother reports that she is using her blue inhaler at least once most days. She wakes frequently overnight coughing.

What is the most appropriate advice to offer?

A) Admit to hospital for a sleep study

B) Start a course of oral corticosteroids

C) Start inhaled long acting B agonist

D) Start oral montelukast

E) Use salbutamol more frequently

A

D) Start Montelukast once a night

491
Q

You are called to the post-natal ward to see Billie, a 3 day old baby. The nurse is concerned that he is breathing quickly and appears pale.

He was born at term to a low risk pregnancy and required no resuscitation. There were no risk factors for sepsis. Antenatal scans were normal.

His RR is 65, HR 180, Cap refil 4 seconds with cool peripheries. Peripheral pulses are difficult to feel. He appears pale. HS I+II+0 and the chest is clear. BM=6.

What is the single most likely diagnosis?

A – Cardiac Arrythmia

B – Co-arctation of the aorta

C – Congenital Pneumonia

D – Hypoplastic left heart syndrome

E – Inborn error of metabolism

A

B – Co-arctation of the aorta

First Few Hours
Pulmonary/aortic atresia/critical stenosis
Hypoplastic heart syndrome
First few days
Transposition, Tetralogy, large PDA in premature infants
Co-arctation
First few weeks
Aortic Stenosis
Co-arctation
First few months
Any left to right shunt as pulmonary resistance falls.

492
Q

Julia is a 6 month old who was born at 33 weeks.
She presents with 4/7 difficulty in breathing and a cough. She has a runny nose and a fever of 38.2 degrees.
On auscultation you hear wheeze and creps bilaterally. She has significant recessions.

Which investigation is most likely to reveal the diagnosis?

A) Blood culture
B) Chest X-ray
C) Naso-pharyngeal aspirate
D) Per-nasal swab
E) Throat swab
A

C) Naso-pharyngeal aspirate

493
Q

Julia is a 6 month old baby born at 33 weeks.
She presents with 4/7 difficulty in breathing and a cough. She has a runny nose and a fever of 38.2 degrees.
She has wheeze and creps on her chest with moderate subcostal recessions. Her O2 SATS are 89% on air. She is feeding around 40% of normal.

What is the most appropriate initial management plan?

A) CPAP and NG tube for feeds

B) Inhaled salbutamol, nasal cannulae O2 and encourage breast feeds

C) Nasal cannulae O2 and NG tube for feeds

D) Nasal cannulae O2, oral augmentin and encourage breast feeds

E) Nebulised salbutamol, nasal cannulae O2 and NG tube for feeds

A

C) Nasal cannulae O2 and NG tube for feeds

Bronchiolitis
Viral infection affecting the lungs in infants < 12 months old
Commonly Respiratory synctial virus, but many other viral pathogens
Common – 30% of infants get it, and 3% of infants admitted to hospital in first 12 months of life
Presentation – URTI, followed by cough, then development of respiratory distress over 3-4 days. Peaks day 5, lasts 10-14 days.
Death rate 8 per 100000

494
Q

Nana Kwame is 2 days old. He became jaundiced yesterday and is receiving phototherapy.

He was born at term following an uncomplicated pregnancy with no risk factors for sepsis. Mum’s blood group is O+ve and the family is originally from Ghana. He is exclusively breast fed.

What is the most likely cause of his jaundice?

A) ABO incompatibility

B) Physiological/breast milk jaundice

C) Rhesus incompatibility

D) Sepsis

E) Sickle cell disease

A

A) ABO incompatibility

495
Q

Baby Smith is 1 day old and has developed jaundice which is above the phototherapy line.

She was born at 37 weeks following an uncomplicated pregnancy. Mum’s blood group is A+ve

On examination she appears quiet. Her respiratory rate is 85 breaths per minute. She has not yet tolerated a feed.

What is the single most likely diagnosis?

A) ABO incompatibility

B) G6PD deficiency

C) Inborn error of metabolism

D) Neonatal sepsis

E) Resus incompatibility

A

D) Neonatal sepsis

496
Q

Abbey is 10 days old and is referred in by the midwife as she appears jaundiced in her body and sclera.

She was born at 36 weeks following a C-section and received 48 hours of antibiotics.

She has been well since and is exclusively breast fed.
Birth weight : 3.2kg Current weight 3.3kg

Abbey appears well on examination and is alert and active. Her observations are stable.

What is the most likely diagnosis?

A) Biliary Atresia

B) Gilbert’s disease

C) Hypothyroidism

D) Physiological/breast milk jaundice

E) Urinary tract infection

A

D) Physiological/breast milk jaundice

497
Q

Ibrahim, 4 year old boy, presents to A&E with a 1 day history shortness of breath and lethargy. He had presented 2 days earlier with a low grade fever, dysuria and urinary frequency and was sent home with oral antibiotics.
He has been previously well, has never been admitted to hospital and was born at term with no complications. He is on no regular medications but is allergic to penicillin. His mother is on iron supplements from her GP for anaemia, otherwise no FHx of note.
He is now afebrile. HR 148, RR 40, SATS 98%
His sclera are yellow, HS1+2+0, Chest clear and abdomen soft non tender.

Blood tests are performed and return as follows:
Hb 5.2 WCC 7.4 Plts 250 Retics high
Na 136 K 5.1 Urea 4.6 Creat 42
Bili 96 AST 40 ALP 240 CRP 4

You request a blood film. What abnormalities would you expect to see?

A – Auer Rods
B – Heinz bodies with bite cells
C – Howell-Jolly bodies with macrocytosis
D – Microcytic Hypochromic anaemia
E – Small round cells (Spherocytes)
A

B – Heinz bodies with bite cells

498
Q

Joshua is 2 months old and presents to A&E. His mum reports that he has been feeding around 50% of usual over the last 24 hours and she has struggled to settle him to sleep. She says he has felt quite hot. She denies any coryzal symptoms or D+V.
He was born at 37+2. PROM but otherwise uneventful pregnancy. He is otherwise well.
He cries inconsolably when examined. HR 175, CRT 2s, RR 50 (crying), SATS 98% Chest clear, abdo SNT. No rashes are seen. Temp 38.3 degrees.
What is the most appropriate management?

A) Admit for observation on the ward
C) Admit, start IV Amoxicillin and Cefotaxime, perform full septic screen including LP and start maintenance fluids.
B) Admit to the ward, start augmentin IV and perform septic screen including LP
D) Admit, start IV Cefotaxime, perform full septic screen including LP and give fluid bolus
E) Send home with advice to return if a non-blanching rash occurs

A

B) Admit, start IV Amoxicillin and Cefotaxime, perform full septic screen including LP and start maintenance fluids.

499
Q

Glory is an 8 year old girl who has found it difficult to walk over the last 24 hours. She is unable to weight-bear on her left leg and complains of a rash over her left shin where she ran into a table 3 days earlier. She has been more lethargic over the last 24 hours and has lost her appetite. Her mum has given her codeine and paracetamol at home which has not controlled her pain.
She has a background of sickle cell anaemia and takes Penicillin V and folic acid.
Her temperature is recorded at 38.5 degrees, HR 130, RR 30, SATS 99%. Cap refil <2 seconds

A – Co-amoxiclav orally and regular codeine
B – IV Ceftriaxone and blood transfusion
C – IV Ceftriaxone and paracetamol
D – IV Ceftriaxone, oramorph, regular simple analgesia and IV fluids
E – Observe on ward and double dose of Penicillin V

A

D – IV Ceftriaxone, oramorph, regular simple analgesia and IV fluids

Substitution of glutamic acid for Valine at position 6 on B-haemoglobin gene.
HbSS, HbSC (less severe), HbS-Thal.
CRISES – Caused by deoxygenated, sickled haemoglobin -> shorter life span, occlusion in small vessels.
Vaso-occulsive crises
Infective (most common cause of mortality)
Functionally asplenic by 1 year old
Susceptible to encapsulated organisms eg pneumococcus
Osteomyelitis can be unusual organisms like Salmonella.
Haemolytic
Aplastic (eg Parvovirus B19)
Sequestration (mainly spleen but can be liver or lungs)

500
Q
Which of the following medications is not likely to be required in a child with Cystic Fibrosis?
A) Creon
B) Hydroxcarbamide
C) Insulin
D) Tobramicin
E) Ursodeoxycholic acid
A

B) Hydroxycarbamide

R : Respiratory
Bacterial infection/colonization eg Pseudomonas, Staph Aureus, Haemophilus
Allergic bronchopulmonary aspergillosis (ABPA)
C : Cardiac
Signs of right heart failure if pulmonary hypertension present
H : Hepatic
Cholestasis in infancy causing obstructive jaundice.
Fatty liver
Cirrhosis
Biliary obstruction and gall stones.
I : Intestinal
Meconium ileus (think of with signs of respiratory distress and a laparotomy scar)
Rectal prolapse
Distal intestinal obstruction syndrome
P : Pancreatic
Insufficiency in 85% of patients
Pancreatitis
Impaired glucose tolerance in 10%, IDDM in 7% of patients (DKA is uncommon).
S : Salt deficiency state
Hyponatraemic, Hypochloraemic metabolic alkalosis

501
Q

A baby has been brought to NICU because of bilious vomiting. You’ve found out the baby was born at term after an uncomplicated delivery and nothing was picked up in the fetal anomaly scan. You examine the abdomen and find it is not distended or tender, there is a patent anus. You get an abdominal Xray and it shows a dilated stomach and another smaller bubble of air next to the stomach but otherwise no other gas pattern. There is no pneumatosis coli. What is the most likely diagnosis?

a) Colonic atresia
b) Oesophageal atresia
c) Pyloric stenosis
d) NEC
e) Duodenal atresia

A

e) Duodenal atresia

502
Q

Princess, 4, has sickle cell anaemia. She presents in a haemolytic crisis. Her Hb is found to be 2.3 and her GCS is 8. Her parents are Jehovahs Witnesses and refuse any blood products under any circumstances after long discussions.
What is the most appropriate course of action?

A) Ask a local Jehovah’s witness minister to attend tomorrow morning to discuss with the parents
B) Give the blood transfusion and remove the parents from the room
C) Expand her circulating volume with 0.9% NaCl boluses up to 60ml/kg/day
D) Have social services take away parental responsibility
E) If parents continue to refuse, apply for an emergency court order and treat in the child’s best interests

A

E) If parents continue to refuse, apply for an emergency court order and treat in the child’s best interests

503
Q
  • Alcoholic has cellulitis and was given chlordiazepoxide and antibiotics. What else would you give? Options: oral vitamin B1 [thiamine], pabrinex [thiamine, riboflavin, pyridoxine, ascorbic acid], vitamin B12 [cobalamin]
A

pabrinex

504
Q
  • Old man who was ex-accountant has like MMSE 28/30, getting forgetful and needs wife to do his finances as he gets confused, but manages all other daily activities. what’s the diagnosis?
A

Mild cognitive impairment

505
Q
  • Woman with learning difficulties in a care home with change in behaviour (she had a change in feeding time – rather than change in appetite), irritable because the carer that was looking after them has gone suddenly, what’s the diagnosis?
A

challenging behaviour, depressive episode

506
Q
  • What drug can you give for depression in a guy who had an MI –
A

sertraline

507
Q
  • What does clozapine cause?
A

neutropenia

508
Q
  • Guy on haloperidol gets muscle problems -
A

IM procyclidine

Tetrabenazine - treats moderate/severe tardive dyskinesia
Propranolol - treats akathisia (restlessness).
IM procyclidine and benztropine - treats acute dystonia (spasm of facial muscles)

509
Q
  • Suicide risk. What is the highest predictor of doing it again?
A

Feeling hopeless about future,

510
Q
  • 80-year-old man with new onset dementia, needs MRI but refuses. Who is best to make a decision for him? Options: talk to family with his permission, ask independent mental health advocate
A

independent mental health advocate

511
Q
  • Woman on SSRI wants to get pregnant, was wondering what risk it may have on her baby. Options: stillbirth, persistent pulmonary hypertension of newborn, delayed labour, neonatal hypoglycaemia
A

persistent pulmonary hypertension of newborn

512
Q
  • Woman 5 weeks postpartum feels very sad, unable to cope, teary. Dx?
A

Postpartum depression

513
Q
  • What is the risk of baby getting schizophrenia if the baby’s father has it?
    Options: 7-9%, 12-15%, 20-25% (parent = 10-15%, sibling = 10%, MZ twin = 50%)
A

12-15%

514
Q
-	40-year-old man with moderate learning disability has a Hb of ~6 (below normal range). He refuses blood transfusion but happy to take oral iron therapy. What law do you use to assess his decision?
o	Common Law
o	Mental Capacity Act
o	Mental Health Act
o	Disability Discrimination Act
o	European Convention on Human Rights
A

o Mental Capacity Act

515
Q
  • What is high in anorexia nervosa?
A

Growth hormone

516
Q
  • Man with treatment resistant schizophrenia on clozapine, recently stopped smoking. High level of clozapine now. Most likely consequence? Agranulocytosis, seizures
A

seizures

517
Q
  • Failed 2 antipsychotics, what is next line treatment?
A

Clozapine

518
Q

(dark brown line down the centre of the abdomen – fades after you have the baby)

A

o Linea nigra

519
Q

o Striae gravidarum

A

(stretch marks

520
Q

o Increased loudness of S1 and S2
o Increased splitting of mitral and tricuspid components of S1
o Loud S3 by 20 weeks gestation
o 5% develop S4
o 95% develop a systolic murmur that disappears after delivery
o 20% develop a transient diastolic murmur
o 10% develop continuous murmurs due to increased mammary bloodflow

A

• Pregnancy changes to heart sounds

521
Q

Changes to maternal coagulation in pregnancy?

A

 Factors V, VII, X increase
 Fibrinogen increases
 Fibrinolysis decreases
 Factors XI, Protein S decrease

522
Q

Pregnancy - Number of glandular ducts is increased by

A

Oestrogen

523
Q

Pregnancy, what increases the number of breast gland alveoli.

A

• Progesterone

524
Q

Pregnancy - what causes:
• Delay in gastric emptying
• Reduced colonic motility ( constipation).

A

Progesterone during pregnancy is the cause of constipation

525
Q

What causes increased urinary frequency in 1st trimester?

A
  • Urinary tract becomes dilated during pregnancy, and most women have some evidence of stasis or hydronephrosis (R>L).
  • These changes predispose pregnant women to ascending UTI (taken very seriously as more likely to get pyelonephritis with pyrexia  pre-term labour) therefore frequent MSU
  • GFR increases by 50%
  • Polyuria
  • Plasma creatinine, urate and urea decrease. NB: need to adjust reference ranges
  • Glycosuria may occur.
526
Q

What stimulates TSH receptor in pregnancy?

A

• Thyroid: BMR increases by 30%. TSH levels unchanged. T3/4 increase but overall free levels remain the same as there is a concomitant increase in TBP. Increased hbCG in the first trimester binds to TSH receptors and stimulates T3/4 production, and produces a negative feedback which suppresses and decreases TSH levels. The physiological high T3/4 and low TSH may be mistaken for hyperthyroidism; n.b. by the second trimester, serum TSH levels return to normal

527
Q

Diabetic wants to get pregnant. When is best to do so?

A

o Recent HbA1c – advise to continue contraception until HbA1c is at least under 8% (under 6.1% is ideal)

o Commence aspirin 75mg OD if the foetal heart is seen, due to the increased risk of pre-eclampsia
o If the patient conceives with an HbA1c >10%, consider termination

528
Q

Increase or decrease anti-epileptics in pregnancy?

A

Want lowest possible dose / number of drugs to control seizures HOWEVER:
o Increased metabolism, dilution and clearance of antiepileptic (e.g. due to increased GFR and hyperemesis) – may therefore need to increase dose of antiepileptic

o High risk times for seizures:
 1st trimester
 Labour
 Postnatal period

 Avoid sodium valproate, which carries a 25% risk of congenital abnormalities – causes neural tube defects and developmental delay (autism, poor concentration and poor fine motor control):

  • Sodium valproate is the only antiepileptic that has been shown to cause developmental delay.
  • Advise that women who breastfeed have much better sleep, and are therefore less likely to have a seizure (as breastfeeding at night does not require the mother to fully wake up, and releases oxytocin, meaning she will be able to get back to sleep more easily). Breastfeeding while taking anticonvulsants is generally regarded as being safe – occasionally, if feeding coincides with a peak in drug levels, the baby may become drowsy, but in this case all that is needed is to change the time of feeds in relation to tablet taking.

NB: often chronic conditions like asthma and epilepsy follow the rule of thirds: During pregnancy:
• 1/3 will get better
• 1/3 will get worse (typically those that are hardest to control even before pregnancy)
• 1/3 will stay the same

529
Q

Effect of graves on pregnancy

A

o Anti-TSH receptor stimulating antibody (causes Graves’ disease) – can cross the placenta and cause Graves’ disease in the foetus. Signs = foetus moves a lot, growth restriction, sustained foetal tachycardia.
o Anticholinesterase antibody (causes myasthenia gravis).
o TSH receptor stimulating antibodies cross the placenta and will cause thyrotoxic fetus (increased movement, sustained increased HR etc but the medication used to treat mother also crosses the placenta therefore if the mother is euthyroid, so will be the fetus)

Normal antenatal care

530
Q

Components of combined test.

What chance of downs would allow additional testing?

A

o Can also do combined screening test – nuchal translucency plus bloods (hCG, PAPP-A and oestradiol). The combined screening test can be done between 11 and 13+6 weeks, but is optional.

o If someone has a 1 in 150 or greater chance of having a baby with Down’s syndrome, send them for diagnostic tests:
 CVS (samples the placental tissue) – is not definitive, as the placenta may have a mosaicism that means its genetics are not the same as those of the foetus.
• From 11 weeks
• 1-2% risk of procedure-related miscarriage
 Amniocentesis (samples the amniotic fluid)
• From 15 weeks
• 1% risk of procedure-related miscarriage
o NB: if definitive diagnosis of Down’s from amniocentesis, then would perform an extra cardiac scan

531
Q

Management of high risk cervical incompetence?

At what cervical length should Rx be offered?

What is this Rx?

A

 if a woman was found to be high-risk of cervical incompetence e.g. previous surgery or previous PTL due to this, cervical length is checked, and it is usually done as an add-on to the dating and anomaly scan. In very high-risk women, frequency of scanning might be increased e.g. at 16 weeks as well and every 2-3 weeks thereafter. Cervical length becomes less accurate after 22 weeks.

For women with cervical length <25mm:
• Cervical suture (aims to close the internal os). Taken out at 36-37 weeks GA or earlier if she goes into labour.
• Vaginal progesterone (proposed that this causes uterine quiescence) – given 2x a day, either vaginally or rectally until 36 weeks from when shortened cervical length is found

532
Q

What additional aspects are assessed for on doppler if there is reduced growth?

A

If there is a problem with the growth, they will also do Doppler studies:
• Maternal:
o Uterine artery (marker for increased risk of PET or IUGR)
• Fetal:
o Umbilical artery – normal is forward flow through artery. If you start to get high resistance in the placenta this will manifest as changes in the umbilical artery, leading firstly to absent end-diastolic flow (stasis), and then as it worsens it will lead to reverse end-diastolic flow
o MCA – head-sparing (growth of brain is maintained at the cost of other organs) leads to increased velocity through MCA and lower pulsatility index (PI)
o Ductus venosus (from liver to IVC) – reflects the cardiac cycle, absent ‘a’ wave is abnormal

533
Q

List causes of asymmetrical / non

A
o	IUGR:
	Asymmetrical:		(Big head and small abdomen)
•	Uteroplacental insufficiency – the foetus preferentially diverts blood to the vital organs to combat this insufficiency. This means bloodflow to the kidneys and liver is reduced, resulting in reduced production of liquor, and less storage of glycogen in the liver which in turn results in a discrepancy between the growth of the foetal head and the abdomen. 
•	Pre-eclampsia 
	Symmetrical:
•	Pre-eclampsia 
•	Current maternal disease:
o	Renal disease (including renal transplantation)
o	Hypertension
o	Congenital heart disease
o	Severe anaemia
o	Sickle cell disease
o	SLE
o	Cystic fibrosis
o	HIV 

 if baby is <10th centile on GAP growth chart – deliver at 37 weeks

534
Q
  • This is an ultrasound finding, and happens after the first ultrasound scan.
  • You see two sacs on the first ultrasound scan, so say it is a twin pregnancy. However, you later do another scan and find that one sac has disappeared.
A

Vanishing twin syndrome:

535
Q
  • Intracardiac potassium chloride is given to one or more foetuses under ultrasound guidance, in order to reduce the number of foetuses in the pregnancy.
  • Aim is to improve the outcomes for the remaining foetuses by reducing the risk of miscarriage and preterm labour.
  • Is usually performed at 12-14 weeks gestation.
  • Procedure-related risk of miscarriage of wanted foetuses is around 6%.
A

Selective fetocide (multifetal pregnancy reduction):

536
Q

o Very rare complication of vaginal deliveries.
o Occurs when the first twin is presenting breech, and its head is prevented from entering the pelvis by the head of the cephalic-presenting second twin.
o If it is diagnosed during the first stage of labour, a caesarean section should be performed.
o If it is diagnosed during the second stage of labour, general anaesthesia is required to allow manipulation of the foetuses.

A

• Locked twins:

537
Q

Why is the incidence of ectopics rising?

A

• Incidence = 1 in 100 (1% of pregnancies), but this is rising due to the rising incidence of PID, asymptomatic chlamydial infection, and an increase in the number of IVF pregnancies (10-15% of ectopics occur following IVF).

Risk factors:
• PID (due to tubal damage and pelvic scarring) associated with STIs
• Endometriosis (due to tubal damage)
• Previous peritonitis or pelvic surgery, e.g. for appendicitis (due to pelvic scarring)
• Previous tubal surgery (e.g. for sterilisation, reversal of sterilisation, previous ectopic pregnancy)
• Previous ectopic pregnancy (due to tubal damage and pelvic scarring)
• Coil in situ (increases risk of abnormal implantation)
• Progesterone-only pill (increases risk of abnormal implantation because it works by reducing the motility of the cilia in the Fallopian tubes)
• IVF (increases risk of abnormal implantation)
• Age >40
 Heterotopic ectopic pregnancy – have ovulated twice, and get one intrauterine pregnancy and one ectopic pregnancy.

o You have to warn the patient that methotrexate is teratogenic, so they should not attempt to conceive for 3 months after treatment.

538
Q

Why should ectopics NOT get pregnant for 3 months?

A

o You have to warn the patient that methotrexate is teratogenic, so they should not attempt to conceive for 3 months after treatment.

So only if medical

539
Q

List reasons why CKD is dangerous in pregnancy?

A

• Impact of CKD on pregnancy:
o Women more likely to be chronically hypertensive, or if not already hypertensive, to develop gestational hypertension.
o Increased risk of pre-eclampsia (the risk in the general population is about 5%, but in people with CKD it is about 50%).
o Increased risk of preterm birth
o Increased risk of foetal growth restriction
o Drugs the mother is taking for her CKD may have an effect on the foetus.
o Deterioration in renal function (particularly in CKD stages 3-5) as the mum’s kidneys cannot cope with the physiologically increased GFR that occurs as a result of pregnancy. This means that pregnancy shortens the time to these women needing a kidney transplant/dialysis.
 In pregnant women, haemodialysis should be increased to 5 times a week (normal = 3 times a week) to improve outcomes – increases foetal growth etc.

540
Q

2 main causes of LATE miscarriage?

A
  • Infection – bacterial vaginosis (change in the natural flora of the vagina) has been linked to second trimester miscarriage, as have the causative organisms above.
  • Cervical incompetence (the cervix is slightly dilated, so the membranes bulge out through it. Can occur due to e.g. previous LLETZ procedure or multiple previous abortions)
541
Q

What two ABx are given as prophylaxis post surgical termination?

A

• Metronidazole – for prophylaxis against E. coli infection
 Oral erythromycin – for chlamydia prophylaxis

Histology/antiD/counselling on endometritis

542
Q

FETOPLACENTOMATERNAL

CAUSES OF STILLBIRTH

A
Causes:
•	Maternal conditions:
o	Diabetes
o	Pre-eclampsia
o	Sepsis
o	Obstetric cholestasis 
o	Acute fatty liver of pregnancy
o	Thrombophilias:
	Protein C and protein S resistance
	Factor V Leiden
	Antithrombin III deficiency
•	Foetal conditions:
o	Rhesus disease – causes severe anaemia. Gets more severe with subsequent pregnancies. 
o	Chromosomal abnormality 
o	Structural abnormalities
o	IUGR 
o	Twin-to-twin transfusion syndrome 
o	Alloimmune thrombocytopenia 
o	Infection:
	Listeria
	Toxoplasmosis
	Parvovirus 
•	Placental conditions:
o	Postmaturity – placenta starts to deteriorate 
o	Placental abruption
o	Placenta praevia causing significant bleed 
o	Recurrent antepartum haemorrhage – repeated small abruptions can compromise foetal wellbeing and cause IUGR 
o	Cord prolapse
543
Q

o “Woody uterus” – hard uterus that is tender throughout on palpation

A

P ABRUPT

544
Q

PP RFs

A
o	Placenta praevia in previous pregnancy
o	Previous caesarean section
o	Previous TOP
o	Multiparity 
o	Age >40
o	Multiple pregnancy (because they have a bigger bulk of placenta, so it is more likely that some of the placenta will go into the lower segment)
o	Smoking
o	Deficient endometrium 
o	IVF
545
Q

When should placentae praevia women be admitted?

A

o Admission at 36 weeks is recommended for women with major placenta praevia, because of the unpredictability of the timing of labour and the potential for heavy bleeding once labour starts and the cervix starts to dilate.

546
Q

Diabetes-related changes in pregnancy:

A
  • The woman becomes more resistant to her own body’s insulin, partly due to the placenta secreting “anti-insulin” hormones (human placental lactogen, glucagons and cortisol).
  • The body loses its ability to smoothly regulate glucose levels, due to changes in glucose handling. This results in lower fasting glucose levels and higher postprandial levels than when not pregnant.
  • Renal threshold for glucose changes, so most women will have glycosuria at some point in pregnancy.
  • These effects increase through the second and third trimesters.
  • In order to cope with these changes, a woman will have doubled her insulin production by the end of pregnancy.
547
Q

DM risks to baby

A
•	Foetal insulinaemia:
o	Macrosomia (accelerated fetal growth)
o	Organomegaly
o	Polycythaemia
o	Hypoglycaemia
o	Respiratory distress syndrome 
•	Neonatal hypoglycaemia  early feeding, ideally breastmilk + top up milk if necessary to prevent hypo 
•	Increased risk of shoulder dystocia  brachial plexus injury e.g. Erb’s palsy and fetal hypoxia 
•	Intrauterine death/stillbirth
548
Q

DM risks to mother

A
  • Complications of diabetes e.g. hypos, retinopathy, nephropathy
  • Difficult birth due to macrosomic baby  increased requirement for intervention e.g. assisted vaginal delivery (instrumental) or c-section
  • Increased risk of developing pre-eclampsia (greatest risk in women with pre-existing diabetes)
  • Increased risk of infections e.g. thrush
549
Q

Changes in pregnancy that are protective against anaemia:

A

o Increased production of red cells
o Increased iron absorption in gut
o Increased transferrin (therefore increasing the total iron binding capacity)

550
Q

T/F
o Red cell volume increases
o Plasma volume increases (haemodilution)
o Serum iron and serum ferritin levels decrease
o Renal clearance of folate increases
o These changes mean that you get a physiological anaemia in pregnancy anyway, but if your Hb falls below 100g/dl this is pathological.

A
T
T
T
T
T
551
Q

What iron do you Rx preggo ladies with if non-compliant with tablets?

A

• IV iron = monofer. This is a single dose iron injection, and is given when someone’s compliance with oral iron supplements is in doubt. Give a small test dose first to check that it does not cause an allergic reaction.

552
Q

platelet count falls during pregnancy and goes back to normal after delivery.

A

o Gestational thrombocytopenia

553
Q

Pathological platelet level in pregnancy

A
  • Normal platelets in pregnancy = 100-400. <100 is pathological.
  • Platelets are physiologically lower in pregnancy due to the dilutional effect of plasma volume expansion.
554
Q

What delivery should be avoided in ITP in pregnancy?

A

 Use of suction or forceps should be avoided, as the ITP can be transmitted to the baby, and performing suction could result in a suction haematoma

555
Q

Most common cause of DIRECT maternal death in UK?

A

• Pulmonary embolism is the most common cause of direct maternal death in the UK

556
Q

When does post partum thyroiditis typically present?

A

Postpartum thyroiditis:
• Affects 5-11% of women
• Usually occurs around 12 weeks postpartum

o Treatment is not always necessary, as the condition is usually self-limiting and treatment will not speed up recovery, although it will relieve troublesome symptoms.
o Hyperthyroid symptoms can be managed with beta blockers
o Hypothyroid symptoms can be managed with thyroxine replacement

o 3-4% of patients will remain permanently hypothyroid and will require long-term thyroxine replacement
o A third will be euthyroid for a few years, but will then go on to develop permanent hypothyroidism
o Recurrence in future pregnancies is common

557
Q

Mainstay of asthma monitoring in pregnancy?

A

• Peak flow and FEV1 are unaffected by pregnancy, so remain the mainstay of monitoring asthma.

558
Q

Methyldopa contraindication?

A

o Methyldopa is contraindicated in patients with depression, and is usually discontinued in the postnatal period due to the increased risk of postnatal depression.

559
Q

Other causes of proteinuria besides PET?

A
  • Contamination with blood
  • Contamination with liquor (e.g. if membranes have ruptured)
  • Contamination with vaginal discharge
  • UTI (should be suspected if leukocytes, nitrites and/or blood are also present in the urine) – however, pre-eclampsia should not be excluded if a UTI is found, as the two can occur simultaneously
560
Q

Left lateral tilt?

A

Avoid canal compression

561
Q

Obstetric cholestasis - what needs to be ruled out first?

A

Obstetric cholestasis is a diagnosis of exclusion – should perform serology for hepatitis and ultrasound scanning for gallstones before making a diagnosis of obstetric cholestasis

562
Q

Complications of obstetric cholestasis:

A

• Maternal:
o Constant itching – debilitating
o Insomnia
o Postpartum haemorrhage – due to malabsorption of vitamin K and deranged clotting if liver function is very abnormal
• Fetal:
o Foetal and neonatal intracranial haemorrhage – due to maternal malabsorption of vitamin K
o Premature labour – 40% of patients with obstetric cholestasis will deliver before 37 weeks.
o Foetal distress in labour – meconium-stained liquor is likely to occur
o Intrauterine death/stillbirth – bile acids are cardiotoxic to the baby, and can cause it to go into complete heart block. The higher the bile acids, the higher the risk of complete heart block and resultant stillbirth. It is therefore important to diagnose obstetric cholestasis due to this small risk of stillbirth.

563
Q

Management of obstetric cholestasis

A

• Antenatal:
o Symptomatic relief:
 Piriton
 Aqueous cream with menthol
 Ursodeoxycholic acid – lowers bile acids and reduces itching
o Vitamin K 10mg OD – from 32 weeks onwards, to counteract the malabsorption of vitamin K that this condition causes and prevent the resultant complications.
o Foetal monitoring in DAU:
 Growth scans
 Doppler
 Twice-weekly CTG monitoring
o Counselling about the risk of preterm delivery, foetal distress, induction of labour and stillbirth – may suggest elective caesarean section, because due to the increased risk of foetal distress during labour, women are more likely to end up needing an operative delivery anyway
o Advise patient to come straight to triage if there are reduced foetal movements
o Weekly LFTs, bile acids and prothrombin time
o Induction of labour at 37-38 weeks

o Check LFTs at 6 weeks post-delivery – in obstetric cholestasis, the woman will clear her bile acids and stop itching spontaneously in the days following delivery. If she still has itching at six weeks, you would therefore be worried that you have missed something else.

564
Q

Best pain relief for VBAC?

A

o Pain relief:
 Epidural is the best option so that c-section can be done immediately in case of scar rupture

• If a patient attempting VBAC needs induction, you should not use prostaglandins to induce, as they triple the risk of uterine rupture. Can break waters and use balloon.

565
Q

Induction of VBAC. What is contraindicated?

A

o Pain relief:
 Epidural is the best option so that c-section can be done immediately in case of scar rupture

• If a patient attempting VBAC needs induction, you should not use prostaglandins to induce, as they triple the risk of uterine rupture. Can break waters and use balloon.

566
Q

What is an unstable lie?

A

o An unstable lie is when the foetal lie is different at each palpation.
o At term, an unstable lie can be managed expectantly, because spontaneous version to a cephalic presentation often occurs due to the increase in uterine activity.
o If the lie becomes transverse, this can be corrected by ECV followed by induction of labour.
o If the lie remains unstable, caesarean section is indicated.

567
Q

Causes of face presentation?

A

o Congenital goitre
o Congenital tumours of the neck (rare)
o Anencephaly (however, this is now rare due to increased prenatal diagnosis and termination of foetuses with anencephaly)

568
Q

What is malposition?

A
  • The position is the alignment of the foetal head within the pelvis during delivery.
  • Under normal circumstances, the head usually engages in an occipito-transverse position. With descent, the head rotates to an occipitoanterior position in order to present the narrowest diameter for delivery.
  • Any position that does not follow this pattern is regarded as a malposition, with the most important two positions being the occipitoposterior position and deep transverse arrest.

• Occipitoposterior position:
o Occurs in approximately 20% of women in early labour.
o Risk factors:
 Anthropoid and android pelvises (have an anteroposterior diameter greater than or equal to the transverse diameter)
 Anterior placenta
o An OP position is the commonest cause of an unengaged head at term in a primagravida.

 During the second stage, an episiotomy is recommended if the OP position persists, because a wider diameter presents, and this increases the risk of a severe perineal tear occurring.
 If rotation fails to occur spontaneously, it can be achieved using the following techniques:
• Manual rotation
• Kielland’s rotational forceps
• Ventouse extraction with spontaneous rotation

569
Q

Once in active labour, how much should the cervix progress by?

A

• Dilatation of the cervix – should be 1cm per hour in a nulliparous woman, and 1-2cm per hour in a multiparous woman

570
Q

____ is bounded by the pubic crest, the iliopectinaeal line and the sacral promontory. It is oval in shape, and its widest diameter is transverse.

A

Pelvic inlet

571
Q

______ is bounded by the lower border of the pubic symphysis, the ischial spines and the tip of the sacrum. It is oval in shape, and its widest diameter is anteroposterior.

A

pelvic outlet

572
Q

Failure to progress management:

A

• First line = if someone is not progressing, first thing to do is perform ARM as this will cause release of endogenous prostaglandins to enhance labour
• Second line = can give syntocinon to stimulate contractions, and increase their strength and frequency:
o Regular contractions will improve descent of the foetus, and will also help to correct a foetal malposition by rotating the head against the pelvic floor muscles.
 NB: if contractions are already 5 in 10 then might not be suitable
o Has similar side effects to ADH, e.g. fluid overload.

  • The presence of good contractions over several hours without significant progression in terms of cervical dilation is an indication for caesarean section.
  • Failure to progress in the second stage of labour is an indication for instrumental delivery.
  • If the head is almost crowning, an episiotomy may be all that is necessary in order for vaginal delivery to occur.
573
Q

Normal acceleration

A

• Accelerations – these are increases in the foetal heartrate of 15bpm or more, lasting 15 seconds or more. They are normal.

574
Q

Definition of a deceleration

A

Decelerations – these are falls in the foetal heartrate below the baseline of 15bpm or more, lasting 15 seconds or more. Different patterns of decelerations can be seen, depending on their timing with the contractions

575
Q

What is an early deceleration?

What is it caused by?

A

o Early decelerations – the foetal heartrate slows at the same time as the onset of the contraction, and returns to the baseline at the end of the contraction.
 Caused by fetal head compression (high PSNS stimulation)

Single deceleration lasting 3 mins is pathological

576
Q

What is a late deceleration, and what is it caused by?

A

o Late decelerations – the foetal heartrate begins to fall during the contraction, with its trough more than 20 seconds after the peak of the contraction, and returning to baseline after the contraction.
 Occur due to decreased uterine blood flow to baby which is detected by the aortic arch chemoreceptors and leads to high PSNS stimulation. It is associated with hypoxia  fetal blood sample

Single deceleration lasting 3 mins is pathological

577
Q

What is a variable deceleration and what is it caused by?

A

o Variable decelerations – the timing of the slowing of the foetal heartrate in relation to the contraction varies, and can occur in isolation. There is typically rapid onset and recovery, but other features e.g. loss of the normal baseline variability may make this type of deceleration more suspicious.
 Occur due to cord compression. Vein compression leads to decreased venous return and arterial pressure leading to increased SNS to maintain blood pressure. Compression of arteries leads to increase in BP and therefore PSNS activity, lowering HR. When the compression of arteries is relieved it may cause hypotension leading to a reactive tachycardia transiently. If it is persistent, requires FBS.

Single deceleration lasting 3 mins is pathological

578
Q

What is a sinusoidal pattern

A

o Sinusoidal pattern – there is a regular oscillation of the baseline, with absent variability, lasting for at least 10 minutes and with an amplitude of 5-15bpm above and below the baseline.

579
Q

Big square (AKA 4 little ones) on CTG

A

1 min

580
Q

Helper management of shoulder dystocia

A

o H: help call
o E: evaluate episiotomy (this will now allow the shoulders to deliver but will allow manipulation of the baby to achieve delivery
o L: legs  McRobert’s position – take woman out of the lithotomy position, straighten her legs and then bend them, pushing her thighs as far into the abdomen as you can. This straightens the spine and also expands the pelvis so that the shoulder can be delivered more easily. Is successful in about 90% of shoulder dystocias.
o P: pressure  apply suprapubic pressure to dislodge the anterior shoulder.
o E: enter manoeuvres  internal rotation techniques to try and rotate the baby
o R: remove posterior arm by inserting hand into the vagina and aiding delivery
o R: roll on to all fours (if above measures fail)

581
Q

Management of uterine inversion?

A

• Complete = uterine fundus passes through the cervix.
• Incomplete = fundus is still above the cervix.
• Can occur spontaneously, e.g. in association with a fundal placental site or an unicornuate uterus, or may be a result of mismanagement of the third stage of labour.
• Delivering the placenta and membranes by controlled cord traction should help prevent this.
• May present as PV bleeding, or as severe lower abdominal pain, followed by collapse and haemorrhage. The pain is secondary to tension on the infundibulopelvic ligaments.
• Management:
o Resuscitate patient – ABC approach
o Put uterus back in its correct position, either manually or hydrostatically
o Oxytocin infusion

582
Q

Indications for forceps

A

• There are two main types of forceps:
o Traction forceps (aka non-rotational forceps – are only suitable for occipitoanterior and occipitoposterior positions) – Simpson’s, Anderson’s, Neville-Barnes’ or Wrigley’s
o Rotational forceps – Keilland’s
• Main maternal complication is trauma – worst kind is trauma all the way up to the top of the vagina.
• Indications for forceps delivery:
o Medical conditions complicating labour e.g. cardiovascular disease
o Unconscious mother – means she is unable to assist with pushing and therefore ventouse is not suitable
o Gestation <34 weeks
o Face presentation
o Known or suspected foetal bleeding disorder
o Breech presentation:
 Use long Piper’s forceps to deliver the head
 Assistant supports the baby’s legs horizontally
 The forceps are applied downwards and used to pull the head out

583
Q

2 Types of ventoux

A

• One of the complications of ventouse is retinal detachment or retinal haemorrhage
• Two types:
o Disposable = KIWI
o Machine
• Can be used with any position of foetal head, as they permit rotation of the foetal head during traction.
• Indications for ventouse delivery:
o Delay in 2nd stage of labour due to maternal exhaustion
o Delay in 2nd stage of labour due to foetal malposition (occipitoposterior or occipitotransverse position)
o Abnormal CTG
• Ventouse delivery should not be used for babies born before 34 weeks gestation.
• Use of the ventouse requires maternal effort and adequate contractions.
• Method:
o The cup is applied in the midline, over or just anterior to the occiput.
o Apply traction in time with the maternal contractions and maternal effort, along the pelvic curve, i.e. initially in a downward direction and then upwards as the head crowns.
o The procedure should be complete within around 15 minutes of cup application.
o CTG monitoring should be used throughout.

584
Q

Tear

just involves the superficial skin or the vaginal mucosa

A

o First-degree tear – just involves the superficial skin or the vaginal mucosa
o Second-degree tear – involves the superficial muscles e.g. the transverse perinea
o Third-degree tear – involves the external or internal anal sphincters:
 3a – into the external anal sphincter, less than 50% of the external sphincter fibres are involved
 3b – more than 50% of the external sphincter fibres are involved
 3c – into the internal anal sphincter
o Fourth-degree tear – into the anorectal mucosa
o Buttonhole tear – a full-thickness tear, but higher up (through the vagina and into the rectal mucosa), meaning the anal sphincters and anal canal are intact, but there is a tear going into the rectum.

585
Q

tear

involves the superficial muscles e.g. the transverse perinea

A

o First-degree tear – just involves the superficial skin or the vaginal mucosa
o Second-degree tear – involves the superficial muscles e.g. the transverse perinea
o Third-degree tear – involves the external or internal anal sphincters:
 3a – into the external anal sphincter, less than 50% of the external sphincter fibres are involved
 3b – more than 50% of the external sphincter fibres are involved
 3c – into the internal anal sphincter
o Fourth-degree tear – into the anorectal mucosa
o Buttonhole tear – a full-thickness tear, but higher up (through the vagina and into the rectal mucosa), meaning the anal sphincters and anal canal are intact, but there is a tear going into the rectum.

586
Q

Tear involves the external or internal anal sphincters:

A

o First-degree tear – just involves the superficial skin or the vaginal mucosa
o Second-degree tear – involves the superficial muscles e.g. the transverse perinea
o Third-degree tear – involves the external or internal anal sphincters:
 3a – into the external anal sphincter, less than 50% of the external sphincter fibres are involved
 3b – more than 50% of the external sphincter fibres are involved
 3c – into the internal anal sphincter
o Fourth-degree tear – into the anorectal mucosa
o Buttonhole tear – a full-thickness tear, but higher up (through the vagina and into the rectal mucosa), meaning the anal sphincters and anal canal are intact, but there is a tear going into the rectum.

587
Q

tear into the anorectal mucosa

A

o First-degree tear – just involves the superficial skin or the vaginal mucosa
o Second-degree tear – involves the superficial muscles e.g. the transverse perinea
o Third-degree tear – involves the external or internal anal sphincters:
 3a – into the external anal sphincter, less than 50% of the external sphincter fibres are involved
 3b – more than 50% of the external sphincter fibres are involved
 3c – into the internal anal sphincter
o Fourth-degree tear – into the anorectal mucosa
o Buttonhole tear – a full-thickness tear, but higher up (through the vagina and into the rectal mucosa), meaning the anal sphincters and anal canal are intact, but there is a tear going into the rectum.

588
Q

a full-thickness tear, but higher up (through the vagina and into the rectal mucosa), meaning the anal sphincters and anal canal are intact, but there is a tear going into the rectum.

A

o First-degree tear – just involves the superficial skin or the vaginal mucosa
o Second-degree tear – involves the superficial muscles e.g. the transverse perinea
o Third-degree tear – involves the external or internal anal sphincters:
 3a – into the external anal sphincter, less than 50% of the external sphincter fibres are involved
 3b – more than 50% of the external sphincter fibres are involved
 3c – into the internal anal sphincter
o Fourth-degree tear – into the anorectal mucosa
o Buttonhole tear – a full-thickness tear, but higher up (through the vagina and into the rectal mucosa), meaning the anal sphincters and anal canal are intact, but there is a tear going into the rectum.

589
Q

• Prostaglandin F2a

A

(carboprost) is given as IM, comes in viles of 250mg and can be given every 15 minutes (up to 8 times but rare to give this much)

590
Q

• Prostaglandin E2

A

(prostin/propess) not used to control bleeding as they do not cause retraction of muscles. Instead used to induce labour. Side effect profile of prostaglandins is to relax other spinchters

591
Q

• Prostaglandin E1

A

(misoprostol) is used to induce labour, medical ToP, or treatment of gastric ulcers. Typically given per rectum. Dose = 800-1000mg (4 or 5 tablets of 200mg)

592
Q

Commonest causes of maternal mortality in the UK:

A
•	Direct deaths:
o	Thromboembolism – commonest cause 
o	Hypertensive disorders (including eclampsia) 
o	Haemorrhage
o	Genital tract sepsis 
o	Amniotic fluid embolism
o	Early pregnancy deaths (deaths before 24 weeks’ gestation):
	Ectopic pregnancy
	Miscarriage 
	Termination of pregnancy 
•	Indirect deaths:
o	Cardiac disease
o	Psychiatric deaths (commonest cause in most recent report) 
  • Direct (a cause that you can only get while you are pregnant)
  • Indirect (a condition that you had before you were pregnant or developed while pregnant but that was not related to the pregnancy, but was made worse by the pregnancy)
593
Q

Managment of GBS?

What is it based on?

A

o Intrapartum IV antibiotics can be given prophylactically to the mother during delivery to try to prevent transmission of GBS to the newborn baby. In the UK, the choice of whether to administer these intrapartum antibiotics is done via a risk-based approach, with mothers with risk factors for infection being offered antibiotics:
 Those who have had GBS infection detected in a previous pregnancy
 Those with a previous baby with early or late-onset GBS disease
 Those in preterm labour
 Those with a pyrexia (>38oc) during labour
o Benzylpenicillin is the antibiotic of choice.

594
Q

o Originates at the uterus and inserts into the inguinal canal and continues to the labia majora
o May be mistakenly clipped during female sterilisation procedure

A

• Round ligament

595
Q

o Sits behind the uterus and attaches to the anterior sacrum.
o Can be affected n endometriosis this can be affected.
o Major support structure for the uterus
o Damage may result in uterine prolapse
o If abnormal, may be palpated through the posterior fornix

A

• Uterosacral ligament

596
Q

o From sacrum to ischial spines

o Important landmark for pudendal block (local anaesthetic in the perineum)

A

• Sacrospinous ligament

597
Q

o Peritoneal fold that drapes over and attaches the uterus, fallopian tubes and ovaries to the pelvis.

A

• Broad ligament

598
Q

o Contains the ovarian vessels
o Suspends the ovary
o It is only divided if the ovary is taken away (as this contains the blood supply)

A

• Infundibular pelvic ligament

599
Q

o Also suspends the ovary

A

• Ovarian ligament

600
Q

the space between the uterus and the rectum

A

• Pouch of Douglas = the space between the uterus and the rectum

601
Q

39 year-old woman undergoes sterilization. 6 months later she falls pregnant

A

round ligament clipped instead of fallopian tubes.

602
Q

A 60 year-old woman is undergoing a hysterectomy. The surgeon divides the structure containing the ovarian vessels

What ligament?

A

this is the infundibular pelvic ligament.

603
Q

A 33 year-old woman presents with dysmenorrhea and deep dyspareunia. Vaginal examination notes tenderness and nodularity in posterior fornix

A

uterosacral ligament

604
Q

Layers that are cut in order during a C section

A
•	Skin
•	Fat (Camper's fascia)
•	Rectus sheath
•	Scarpa's fascia (superficial)
•	Stretch muscle open (not cut through)
•	Peritoneum
•	N.B. Urachus = remnant of canal that drains the bladder in the fetus found superior to the bladder
 Avoid the inferior epigastric arteries during laparotomies (risk of haematoma)
605
Q

1st line ADHD

A

Psychoeducation is considered a first-line intervention for all patients

606
Q

Learning disability/ developmental tests:

A

Bayley’s Developmental Scales, Griffiths Mental Development Scales, and Kaufman Scales are widely used to assess the development of young children.

Specific cognitive assessments + IQ:
Wechsler Preschool and Primary Scale of Intelligence (WPPSI)
Wechsler Intelligence Scale for Children (WISC)

Reading, language, and mathematical ability can be assessed using the Wechsler Individual Achievement Test (WIAT); when children have normal IQ and are found to have a specific difficulty (e.g., reading and spelling), this points toward a specific diagnosis (e.g., dyslexia).

607
Q

Test for feto-maternal haemorrhage which detects fetal cells in maternal circulation. It estimates the volume of FMH to determine the amount of ADDITIONAL anti-D?

A

Kleihaur test

608
Q

What reverses respiratory depression caused by mag sulfate?

A

Calcium gluconate

609
Q

BMI over 30. What dose folic?

A

5mg high dose

610
Q
Neonatal sepsis
Hypothyroidism
Prader-Willi
Downs
Myaesthenai gravis
Maternal BDZ
A

Neonatal hypotonia

611
Q

What Rx should be used in frail elderly endometrial cancer PTs unfit for surgery?

A

High dose progestogens

612
Q

Does not cross suture lines

A

Cephalohaematoma

Caput succedanum does

613
Q

Child received another live vaccine 2 weeks ago. Can they get MMR?

A

NO

614
Q

MMR vaccine Sx

What is a contraindication?

A

Malaise
Fever
Rash
At 1 week

2nd dose 3/4
Live attenuated

Contraindications to MMR
severe immunosuppression
allergy to neomycin
children who have received another live vaccine by injection within 4 weeks
pregnancy should be avoided for at least 1 month following vaccination
immunoglobulin therapy within the past 3 months (there may be no immune response to the measles vaccine if antibodies are present)

615
Q

Contraindication to MMR?

A

Contraindications to MMR
severe immunosuppression
allergy to neomycin
children who have received another live vaccine by injection within 4 weeks
pregnancy should be avoided for at least 1 month following vaccination
immunoglobulin therapy within the past 3 months (there may be no immune response to the measles vaccine if antibodies are present)

616
Q

Safest antiepileptic in preg

A

Lamotrigine

617
Q

Liver in thoracic cavity

A

Poor prognostic factor for Congenital diaphragmatic hernia

618
Q

Review and refer for undescended testicle when?

A

3 months

619
Q

Continuous machinery murmer
systolic accentuation
Wide pulse pressure
bounding peripheral pulses are frequently present

A

PDA

620
Q

Downs
DM
Turners

What condition is associated with all?

A

Hypothyroidism

621
Q

Is there an increased risk of neonatal sepsis with HIV?

A

Actually the same as non

622
Q

Changes in Hb levels after birth?

A

Fall from 15-20g/dl to 10-11g/dl at 3 months

623
Q

What immune cell predominates in blood at birth?

What does this change to and when does this occur?

A

Lymphocyte

@ 4m -> neutrophil

624
Q

What percentage fetal Hb by 3m?

A

5%

625
Q

When does hand preference develop?

A

18m

626
Q

Reassuring signs in a child not walking by 18m?

A

Bottom shuffling

FHx

627
Q

Normal neuro exam but decreased tone
Associated in a delay in GROSS motor ONLY
Otherwise normal development

A

Benign central hypotonia

CP would also have poor fine motor

628
Q

No organisms on CSF

Inflammatory cells visualised

A

Brain abscess

629
Q
Hypotonia
Smooth skin
Soft cartilage
Undescended testes / big clitoris 
Lanugo hair
A

Prom baby Sx

630
Q

Test to visualise renal scarring

A

DSMA

631
Q

Renal Test for speed of draining and reflux

A

DTPA

632
Q

Test for assessing vesico-ureteric reflux

A

Micturating cysturethrogram

633
Q

Dec hip abduction
+ve trendlenburg
Asymmetry of skin creases
Reduced apparent leg length

A

Mixed congenital sublux of hip

634
Q

Defective proximal renal tubular function, phosphataemia and rickets

A

Cystinosis

635
Q

Hirschprungs associated with which men?

A

2A/B

636
Q
Vomiting
Diarrhoea
Digestive disorder 
Strenuous exercise 
Preg
Eating disorder
Low carb diet
A

Ketones

637
Q

MoA of clomifene?

A

SERM
Dec neg feedback of oestrogen on hypothalamus
Inc GnRH -> LH/FSH pulses -> ovulation

638
Q

RFs gastroschesis

A

Maternal age <20
Alcohol
Tobacco
Socio economic dep

639
Q

Number 1 cause of painless GI bleed in 1-2YO requiring transfusion?

A

Meckels

640
Q

Increased membrane permeability and fluid losss from intravascular compartment
Rarely seen with clomifene
Associated with HCG or GnRH

Abdo bloating 
NV
Ascites 
Hypoproteinuria
Oliguria 
ARDS
VTE
PE
A

Ovarian hyperstimulation syndrome

641
Q

High grade fever THEN rash

A

Roseola infantum

642
Q

Hand foot and mouth cause

A

Coxsackie A6

643
Q

Rise in clozapine level?

A

Recent stop smokin

644
Q

Only effective Rx for large fibroids if want to conserve fertility

A

Myomectomy

645
Q

COCP 2 pills missed

A

Take 2 that day and continue as normal

646
Q

Whooping cough Rx

A

Clarithromycin or azithromycin if <21 days from onset (possible after but unlikely to help)

647
Q

Treat muscle spasticity

A

Baclofen

648
Q

Child under 2, phimosis, whaddyado?

A

Normal

Review in 6month

649
Q

Weight gain
Myocarditis
Arrythmias
Excessive salivation

SEs of what drug?

A

Clozapine

Also agranulocytosis…

650
Q

Common complication of viral gastroenteritis

A

Lactose intolerance

651
Q

Most common cause gastroenteritis

A

Rotavirus

652
Q

CATFISH

Mneumonic for atypical UTI

A
Creatinine
Abdo/pelv mass
Terribly ill
Flow problems
Infected with non-e-coli organism
Septicaemia 
Halted response to ABx
653
Q

Most common cause headache in child

A

Migraine

654
Q

Prophylaxis for PPROM

A

10 day erythromycin

Day unit assessments

655
Q

HCG secreted by

A

Synctiotrophoblast

656
Q

Day 21 progesterone (OR 7 days from end of cycle)

A

Ovulation

657
Q

Any evidence of infection in miscarriage

A

Surgical evacuation

658
Q

Low plasma haptoglobin

A

Haemolysis

Haptoglobin binds haemoglobin in blood

659
Q

Stop COC for how long before surgery?

A

4 weeks - switch to POP

660
Q

Confirm pertussis?

A

Nasal swab

661
Q

Akathisia

A

Restlessness

662
Q

1st line nit Rx

A

Wet combing

663
Q

SE of hysterectomy?

A

Vag vault prolapse

Enterocele

664
Q

SUFE Rx

A

Internal fixation

665
Q

Stop methotrexate when before pregnancy?

A

Both man and women stop 3 months before trying

666
Q

Fever
Tonsillitis
Malaise
Sandpaper rash

A

Scarlet fever

667
Q

Endometrial hyperplasia
PM
Atypia

A

Total abode hysterectomy + bilateral salpingoophorectomy

668
Q

Hep B
Breastfeeding
Is it safe

A

Ayeeeees

669
Q

POP

How long to work

A

2 days

670
Q

1st line for premenstrual syndrome

A

COC

671
Q

Supravalvular aortic stenosis

A

Williams

672
Q

Contraception with weight gain

A

ONLY

Depo-provera injection

673
Q

Post partum thyroiditis diagnosis

Do you need to assess for TPO

A

No

TFTs alone enough

674
Q

Most common ovarian cancer

A

Serous carcinoma

675
Q

Fever >38 in <3 month

A

Admit

676
Q

Saddle shaped nose

Hutchinsons incisors

A

Syphilis

677
Q

Painless haematuria
Flank pain
Anorexia
Fever

A

Wilms

3-5yo

678
Q

Most common complication of TOP

A

Infection

679
Q

1st line anxiety disorder

A

SSRI

680
Q

SOB on feeding
Sweaty
Recurrent chest infections

A

HF

681
Q

POP missed pill

A

Take pill

Use condoms 48hrs

682
Q

COCP missed pill

A
1 = take asap and no action
2 = take asap and take next normal pill time
683
Q

Amiodarone and BFing

A

NO

684
Q

Bowed legs in a 3yo

A

Normal -> unless >4yo

685
Q

U+E seen in pyloric stenosis

A

Increased bicarb

Low K+ + Cl-

686
Q

Child drowsy post febrile seizure

A

Not a febrile seizure

687
Q

Rx for GBS

A

Intrapartum Ben Pen

688
Q

DNA vaccines induce

A

TH1 response

689
Q

Triple test

A

AFP
Oestriol
HCG

690
Q

Trimethroprim contraindicated in which stage preg

A

1 trim

691
Q

+ve hydrogen breath test

A

Lactose intolerance

692
Q

Peristent pulmonary hypertension

A

Fluoextine in 3rd trimester

693
Q

Failure to progress

A

20hrs reg contractions in primp

14 if multip

694
Q

Ix for retained products of conception

A

Exam under anaesthesia

boggy uterus

695
Q

Pierre robin Sx

A

Pierre rob in mouth
Cleft palate
Micrognathia
Posterior displacement of tongue

696
Q

Cleft palate
Micrognathia
Posterior displacement of tongue

A

Pierre rob in mouth

697
Q

WilliamS syndrome

A

Short, Struggles in School, Smiley, Supravalvular aortic StenoSiS

698
Q

Short, Struggles in School, Smiley, Supravalvular aortic StenoSiS

A

WilliamS syndrome

699
Q

Main side effect POP

A

Bleedin

700
Q

Hyper G Questionnaire?

A

PUQE scoring system

Pregnancy unique quantification of emesis scoring

701
Q

CRASH N BURN

A
Kawasaki
Conjunctivits 
Rash
Adenopathy
Strawberry tongue
Hand swelling/feet / peeling
BURN = 5 days fever
702
Q

Most ectopics located in

A

Ampulla

703
Q

Most ectopics rupture in

A

Isthmus

704
Q

Female
Amenorrhoea
Lumps inguinally
May have breast development

A

Androgen insensitivity

46XY

705
Q

1st line imbetigo

A

fusidic acid

706
Q

AGITATED PT ON WARD

1st action

A

Not lorazepam

Assess pt in a safe environment with another person

707
Q

Suboptimal SpO2 in first 10 mins of life

Is this reassuring

A

Its fine - expected

708
Q

Child does not want mole removed

Parents do

A

If you judge a child does not want a cosmetic procedure you must not perform it

709
Q

Moro should disappear at

A

4m

710
Q

Ullipristal given up to

A

5 days later

711
Q

How long should contraception continue after menopause

A

1 yr if over 50

2 yrs if under

712
Q

Increased AFP

A

Omphalocele

NT defects

713
Q

Mod severe tardive dyskinesia

A

Tetrabenzene

714
Q

Ebstein anomaly

A

TR + stenosis

715
Q

Lochia abnormal after

A

6w

716
Q

Breech need US when

A

6w even if normal B+O

717
Q

Perthe Rx under 6yo?

A

Observe
Consider Spica cast
Surgery at 6yo

718
Q

Precious puberty - what effect does TSH have

A

Stimulates FSH receptors

719
Q

Waterhouse freidrichson

A

Rare complication of men meningitis
Bilateral adrenal necrosis
Lack of adrenal hormones and worsening hypotension

720
Q

How long is VZIG useful?

A

10 days post exposure

Oral aciclovir given within 24 hrs of any rash

721
Q

Blocks A2 receptors and increases appetite

also causes sedation

A

Mirtazipine

722
Q

10-15% have febrile seizures

A

Roseola infantum

723
Q

Why does aripiprazole have reduced SEs

A

Partial antagonist of dopamine

724
Q

Continuous hum below clavicle

A

Venous hum

Innocent murmer

725
Q

How much should a 6w old baby eat?

A

150-180ml/kg/day

726
Q

+ve triple test result

A

> 1 in 250 chance downs

727
Q

Ergometrine MoA

A

alpha adrenrgic
Dopamine
Serotinergic binding

728
Q

Hyponatraemia. Which anti epileptic?

A

Carbemazipine

729
Q

Produced by theca cells

A

Testosterone

730
Q

Produced by ovary and secreted throughout cycle

A

Oestradiol

731
Q

Triggers first meiotic divisions

A

LH

732
Q

Early stage endometrial carcinoma

A

Laparoscopic hysterectomy (vs open)

733
Q

Ischiocarvernosis cut during episiotomy

A

False

734
Q

Pelvic outlet is formed by

A

Ishcial tuberosity
Pubic arch
Sacrum

735
Q

Broad ligament of uterus contains

A

Fallopian tubes and uterine artery

736
Q

What week does fetal heart start

A

Usually 5w

737
Q

Blastocyst divides in 3. What is formed

A

Embryo

Placentae

738
Q

Tetrabenazine

A

Tardive dyskinesia

739
Q
Gland like spaces
Call-exner bodies
Coffee bean nuclei
Oestrogen
PM bleed
A

Granulosa ovarian tumours

740
Q

Cremaster preserved but signs of torsion

Blue dot on transillumination

A

Torsion of the hydatid of morgagni
DIFFERENT to torsion
Still requires surgery
More common in younger males

741
Q

Prehns sign

Aka pain disappears on elevation of the testicle

A

Epidydimitis

742
Q

Small for dates = RF
Shock seen is out of proportion with bloodless (due to vagal stimulation)
Cannot palpate uterine fundus

A

Uterine inversion

743
Q
Excessive salivation
Mouth twitching
Grunting
Slurred speech
High amplitude spikes in left centrotemporal region
A

Benign rolandic seizures

Accounts for 20% of childhood epilepsy

744
Q

Epilepsy triggered by alcohol

Can occur on waking

A

Juvenile myoclonic epilepsy

745
Q

3 spike wave complex on EEG

A

Absence seizure

746
Q

Abnormal Poo pattern in child

A

<3 per week

747
Q

Slow spike waves

Very frequent seizures

A

Lennox-gaustaut

748
Q

Common organisms of ophthalmic neonatorum

A

Staph A, chlamidya, gonorrhoea

749
Q

Single dose cef + azithro

A

The gon is gone

750
Q

Transient synovitis gets better when

A

1-2 weeks

If not recall -> consider perthes

751
Q

How long must PTSD symptoms last before diagnosis

A

1 month

752
Q

Pseudomyxoma

A

Mucinous carcinoma

753
Q

Baby blues. When present, when disappear by

A

usually days 3-5

Gone by 10d

754
Q

PND. When present, when disappear by

A

Begins within 3m

Resolves by 6m usually

755
Q

PN psychoses. When present, when disappear by

A

70% present at day 3
Rapid onset
Always presents by 3 weeks

Prognosis variable

756
Q

75% of endometrial cancers are picked up in what stage

A

1

757
Q

MMR associated thrombocytopenia. Is it a thing.

A

Yes its a thing

758
Q
Vomiting
Lethargy
Jaundice
Cataracts
Hepatomegaly (3cm)
A

Galactosaemia

759
Q

Can cows milk protein intolerance occur with breast fed infants

A

Yeas

Trace amounts present in milk

760
Q

N+V in preg
When present
When better

A

usually 4-7w

Better by 12-14 in most

761
Q

When to admit hyperemesis

A

Ketones +2
U+E imbalance
Severe dehydration

762
Q

SEs lactulose

A

Atonic colon

Hypokalaemia

763
Q

Opiods and SSRI

A

List of serotonin syndrome

764
Q

Bronchiolitis

T1/2 resp failure

A

Begins as T1. T2 as child tires

T2 - hypoxia and hypercapnia

765
Q

Eisenmenger

A

R->L shunt due to pulmonary hypotension

766
Q

How long is urine pregnancy test +ve after termination

A

4 weeks

767
Q

How should endometrial hyperplasia be classified?

A

The revised 2014 World Health Organization (WHO) classification is recommended. This separates endometrial hyperplasia into two groups based upon the presence of cytological atypia: i.e. (i) hyperplasia without atypia and (ii) atypical hyperplasia.

768
Q

What should the initial management of hyperplasia without atypia be?

A

Women should be informed that the risk of endometrial hyperplasia without atypia progressing to endometrial cancer is less than 5% over 20 years and that the majority of cases of endometrial hyperplasia without atypia will regress spontaneously during follow-up.

Reversible risk factors such as obesity and the use of hormone replacement therapy (HRT) should be P identified and addressed if possible.

Observation alone with follow-up endometrial biopsies to ensure disease regression can be considered, especially when identifiable risk factors can be reversed. However, women should be informed that treatment with progestogens has a higher disease regression rate compared with observation alone.

Progestogen treatment is indicated in women who fail to regress following observation alone and in P symptomatic women with abnormal uterine bleeding.

Both continuous oral and local intrauterine (levonorgestrel-releasing intrauterine system [LNG-IUS]) progestogens are effective in achieving regression of endometrial hyperplasia without atypia.

769
Q

What should the duration of treatment and follow-up of hyperplasia without atypia be?

A

6 months and re-check

Hysterectomy is indicated in women not wanting to preserve their fertility when (i) progression to atypical hyperplasia occurs during follow-up, or (ii) there is no histological regression of hyperplasia despite 12 months of treatment, or (iii) there is relapse of endometrial hyperplasia after completing progestogen treatment, or (iv) there is persistence of bleeding symptoms, or (v) the woman declines to undergo endometrial surveillance or comply with medical treatment.

770
Q

What should the initial management of atypical hyperplasia be?

A

Women with atypical hyperplasia should undergo a total hysterectomy because of the risk of underlying malignancy or progression to cancer.

LAP rather than Abdominal

There is no benefit from intraoperative frozen section analysis of the endometrium or routine lymphadenectomy.

Postmenopausal women with atypical hyperplasia should be offered bilateral salpingo-oophorectomy P together with the total hysterectomy.

For premenopausal women, the decision to remove the ovaries should be individualised; however, bilateral salpingectomy should be considered as this may reduce the risk of a future ovarian malignancy.

Endometrial ablation is not recommended because complete and persistent endometrial destruction cannot be ensured and intrauterine adhesion formation may preclude endometrial histological surveillance.

If atypia and really want to stay fertile -> counsel on risk of cancer and ongoing surveillance + IUS

771
Q

F/U for atypia hyperplasia who doesn’t want hysterectomy

A

3 monthly

772
Q

When should anendometrial hyperplasia lady conceive?

A

Disease regression should be achieved on at least one endometrial sample before women attempt to P conceive.

Women with endometrial hyperplasia who wish to conceive should be referred to a fertility specialist to discuss the options for attempting conception, further assessment and appropriate treatment.

773
Q

Cyst <5 cm Pre menopausal

Whaddyado

A

Women with small (less than 50 mm diameter) simple ovarian cysts generally do not require follow-up as these cysts are very likely to be physiological and almost always resolve within 3 menstrual cycles.

774
Q

Cyst >1cm PM

Whaddya do

A

Ca 125 + RMI

RMI = U x M x CA-125.

775
Q

Chemo for ovarian cancer (stages 2-4)
NB chemo not routinely recommended for stage 1 - Consider three cycles of adjuvant carboplatin plus paclitaxel10 for women with high-risk stage I disease (grade 3 or stage Ic) if they are prepared to accept treatment of shorter duration but increased toxicity.

A

It is recommended that paclitaxel in combination with a platinum- based compound or platinum-based therapy alone (cisplatin or carboplatin) are offered as alternatives for first-line chemotherapy (usually following surgery) in the treatment of ovarian cancer.

776
Q

Do all postmenopausal women with ovarian cysts require surgical evaluation and is there a role for conservative management?

A

Asymptomatic, simple, unilateral, unilocular ovarian cysts, less than 5 cm in diameter, have a low risk of malignancy. In the presence of normal serum CA125 levels, these cysts can be managed conservatively, with a repeat evaluation in 4–6 months. It is reasonable to discharge these women from follow-up after 1 year if the cyst remains unchanged or reduces in size, with normal CA125, taking into consideration a woman’s wishes and surgical fitness.

777
Q

When should laparotomy be undertaken for PM cysts?

A

All ovarian cysts that are suspicious of malignancy in a postmenopausal woman, as indicated by a RMI I greater than or equal to 200, CT findings, clinical assessment or findings at laparoscopy, require a full laparotomy and staging procedure.

778
Q

PM cyst
Cysts fulfilling ALL of the following criteria: RMI<200
asymptomatic, simple cyst, < 5 cm, unilocular, unilateral

A

CONSERVATIVE

Repeat assessment in 4–6 months
CA125, TVS ± TAS

779
Q

PM
Cysts with ANY of the following features:
symptomatic, non-simple features, > 5 cm, multilocular, bilateral

A

Consider surgery salpingo-oophorectomy (usually bilateral)

780
Q

PM
RMI >200
Next step

A

CT abdo pelvis
MDT review
Either full staging procedure or
Pelvic clearance (TAH + BSO + omentectomy + peritoneal cytology) by a suitably trained gynaecologist

781
Q

What are the risks and benefits of planned VBAC versus ERCS from 39+0 weeks of gestation?

A

Women should be made aware that successful VBAC has the fewest complications and therefore the chance of VBAC success or failure is an important consideration when choosing the mode of delivery.
Women should be made aware that the greatest risk of adverse outcome occurs in a trial of VBAC resulting in emergency caesarean delivery.
Women should be informed that planned VBAC is associated with an approximately 1 in 200 (0.5%) risk of uterine rupture.
Women should be informed that the absolute risk of birth-related perinatal death associated with VBAC is extremely low and comparable to the risk for nulliparous women in labour.
Women should be informed that ERCS is associated with a small increased risk of placenta praevia and/or accreta in future pregnancies and of pelvic adhesions complicating any future abdominopelvic surgery.
The risk of perinatal death with ERCS is extremely low, but there is a small increase in neonatal respiratory morbidity when ERCS is performed before 39+0 weeks of gestation. The risk of respiratory morbidity can be reduced with a preoperative course of antenatal corticosteroids.

782
Q

VBAC success rate

A

Women should be informed that the success rate of planned VBAC is 72–75%.

783
Q

When to refer URGENTLY for suspected ovarian cancer?

A

Refer the woman urgently if physical examination identifies ascites and/or a pelvic or abdominal mass (where it is clear that this is not due to uterine fibroids).

Carry out tests in primary care if a woman (especially if 50 or over) reports having any of the following symptoms on a persistent or frequent basis — particularly more than 12 times per month:
Persistent abdominal distension (women often refer to this as 'bloating').
Feeling full (early satiety) and/or loss of appetite.
Pelvic or abdominal pain.
Increased urinary urgency and/or frequency.
Consider carrying out tests in primary care if a woman reports unexplained weight loss, fatigue or changes in bowel habit.
Advise any woman who is not suspected of having ovarian cancer to return to her GP if her symptoms become more frequent and/or persistent.
Carry out appropriate tests for ovarian cancer in any woman of 50 or over who has experienced symptoms within the last 12 months that suggest irritable bowel syndrome (IBS), because IBS rarely presents for the first time in women of this age.
Measure serum CA125 in primary care in women with symptoms that suggest ovarian cancer.
If serum CA125 is 35 IU/ml or greater, arrange an ultrasound scan of the abdomen and pelvis.
784
Q

STAGING of endometrial cancer

A

1 - confined to uterus
A = half

2 = cervix

3 = local/regional spread
A = Serosa of uterus/adnexae
B = vagina
C1 = positive pelvic LNs
C2 = para-aortic LNs
4A = bladder or bowel
4B = Distant mets / inguinal
785
Q

STAGING of cervical cancer

A
0 = Carcinoma in situ 
1 = cervix only
2 = uterus/parametrial/vagina
3 = pelvic wall / lower 3rd vagina
4a = bladder/bowel
4b = distant mets
786
Q

STAGING of ovarian cancer

A
1 = limited to ovaries
A = 1 ovary
B = both 
C = ruptured capsule
2 = pelvis limited
3 = Abdomen limited
4 = distant mets
787
Q

Vulval cancer Rx

A

Wide radical local excision of the primary tumour with a minimum margin of 15 mm of disease-free tissue is often sufficient.

Dissection of the groin nodes (unilateral or bilateral) should be performed when the depth of invasion is greater than 1 mm (FIGO stage Ib or worse) or the maximum diameter of the tumour is greater than 2 cm.

Groin node surgery should be undertaken through separate incisions (triple incision technique) to reduce morbidity. The incidence of skin bridge recurrence in early-stage disease is low.

In unifocal tumours of less than 4 cm maximum diameter where there is no clinical suspicion of lymph node involvement, patients can be safely managed by removal of the identified sentinel lymph nodes.

788
Q

Rapid tranquillisation guidelines:

A

In general IM lorazepam or haloperidol+promethiazine:

Insufficient evidence to guide choice / no prev antipsychotic -> LORAZEPAM

CV disease -> LORAZEPAM

No response to lorazepam -> HALOPERIDOL with PROMETHIAZINE

789
Q

Women in 1st stage of labour
No SROM
Failing to progress
Whaddjya do?

A

For women with intact membranes in whom delay in the established first stage of labour is confirmed, advise the woman to have an amniotomy, and to have a repeat vaginal examination 2 hours later whether her membranes are ruptured or intact.

790
Q

Do you measure HbA1c in GDM?

A

NICE: Measure HbA1c levels in all women with gestational diabetes at the time of diagnosis to identify those who may have pre‑existing type 2 diabetes

791
Q

When do you offer testing for GDM in a women with a Hx of GDM?

A

NICE: Offer self‑monitoring of blood glucose or a 75 g 2‑hour OGTT as soon as possible for women with a history of gestational diabetes who book in the first trimester.